Vous êtes sur la page 1sur 121

Copyright 2014 Delhi Academy of Medical Sciences, All Rights Reserved.

1/121
Test Information
Test Name GRAND TEST - 129 Total Questions 300

Test Type Examination Difficulty Level Difficult

Total Marks 1500 Duration 180minutes

Test Question Language:- ENGLISH

(1). Vertebral level at which renal artery originates is?

a. T12

b. Superior border of L1

c. Inferior border of L1

d. Inferior border of L2

Solution. Ans-1: (c) Inferior border of L1 Ref.: Read the text below Sol :
Renal artery arises from aorta just inferior to superior mesenteric artery, which arises at the level of L1-L2 vertebral disc

Correct Answer. c

(2). Elevation in a fully adducted eye is done by?

a. Inferior oblique

b. Inferior rectus

c. Superior oblique

d. Superior rectus

Solution. Ans-2: (a) Inferior oblique


Ref.: Read the text below
Sol :
Action of individual muscles of eye

Muscle In primary position Abducted eye Adducted eye

Depression
Abduction
Superior oblique Only Intorsion Only depression
Intorsion

Elevation
Abduction
Inferior oblique Only extortion Only elevation
Extortion

Depression
Adduction
Inferior rectus Only elevation Only Intorsion
Extortion

Elevation
Adduction
Superior rectus Only elevation Only Intorsion
Intorsion

Medial rectus Only adduction -- --

Lateral rectus Only abduction -- --

Correct Answer. a

Copyright 2014 Delhi Academy of Medical Sciences, All Rights Reserved. 2/121
(3). Standing on toe tips is due to?

a. Tibialis anterior

b. Gastrocnemius

c. Tibialis posterior

d. Gracilis

Solution. Ans-3: (b) Gastrocnemius Ref.: Read the text below Sol : GASTROCNEMIUS :
Action : Plantar flexes (points) foot at ankle joint. Assists in flexion of knee joint. It is a main propelling force in walking and running.
Basic functional movement : Standing on tip topes
This muscle of often chronically shortened/tight in several forms of cerebral palsy.
ACTIONS OF TRICEPS SURAE
The two heads of gastrocnemius, together with soleus, form a tripartite muscular mass sharing the calcaneal tendon and are sometimes
termed the triceps surae. These muscles are the chief plantar flexors of the foot.
Gastrocnemius is also a flexor of the knee.
Although they share a common tendon, the two muscles of the triceps surae are capable of acting alone, and often do so: You stroll with
the soleus but win the long jump with the gastrocnemius

Correct Answer. b

Copyright 2014 Delhi Academy of Medical Sciences, All Rights Reserved. 3/121
(4). All of the following are fibrous joints except:

a. Skull sutures

b. Symphysis pubis

c. Syndesmosis

d. Gomphosis

Solution. Ans-4: (b) Symphysis pubis Ref.: Read the text below Sol :
Fibrous joints are connected by dense connective tissue, consisting mainly of collagen.
Types
These joints are also called "fixed" or "immoveable" joints, because they do not move. These joints have no joint cavity and are connected
via fibrous connective tissue. The skull bones are connected by fibrous joints.
Sutures are found between bones of the skull. In fetal skulls the sutures are wide to allow slight movement during birth. They later
become rigid (synarthrodial).

Syndesmoses are found between long bones of the body, such as the radius and ulna in forearm and the distal tibio-fibular joint in leg.
Unlike other fibrous joints, syndesmoses are moveable (amphiarthrodial), albeit not to such degree as synovial joints.
Gomphosis is a joint between the root of a tooth and the sockets in the maxilla or mandible.

Correct Answer. b

Copyright 2014 Delhi Academy of Medical Sciences, All Rights Reserved. 4/121
(5). One of the following is not a branch of axillary artery

a. Superior thoracic

b. Lateral thoracic

c. Suprascapular

d. Subscapular

Solution. Ans 5: (c) Suprascapular Ref Read the text below Sol: Axillary artery is divided into three parts by Pectoralis minor. Branches
from the parts are as follows
First part Superior thoracic
Second part lateral thoracic & thoraco-acromial
Third part anterior circumflex humeral, posterior circumflex humeral & subscapular. Suprascapular artery is a branch of first part of
subclavian artery.

Correct Answer. c

(6). Which of the following arteries do not participate in anastamoses around the scapula:

a. Thoracodorsal

b. Deep branch of transverse cervical

c. Suprascapular

d. Subscapular

Solution. Ans 6: (a) Thoracodorsal Ref Read the text below Sol:
Anstamosis on the dorsal aspect of scapula is between 1st part of subclavian artery (Suprascapular) & third part of axillary
artery(subscapular artery), with a small contribution from dorsal scapular i.e. deep descending branch of transverse cervical.

Correct Answer. a

(7). Blood supply of the breast is by all the following except:

a. Internal thoracic artery

b. Lateral thoracic artery

c. Posterior intercostal artery

d. Thoracodorsal Branch of subscapular artery

Solution. Ans 7: (d) Thoracodorsal Branch of subscapular artery Ref Read the text below Sol:
The chief artery supplying the breast is Lateral thoracic, branch of second part of axillary.
The other arteries are posterior intercostals, superior thoracic & internal mammary.

Correct Answer. d

Copyright 2014 Delhi Academy of Medical Sciences, All Rights Reserved. 5/121
(8). Medial cord of brachial plexus gives rise to all except:

a. Nerve to supraspinatus

b. Medial cutaneous nerve of arm

c. Medial cutaneous nerve of forearm

d. Ulnar nerve

Solution. Ans 8: (a) Nerve to supraspinatus Ref Read the text below Sol: Medial cord of brachial plexus gives origin to the following
nerves.
a. Medial pectoral nerve
b. Medial root of median nerve
c. Medial cutaneous nerve of arm
d. Medial cutaneous nerve of forearm
e. Ulnar
Supraspinatus is supplied by the suprascapular nerve, which arises from the upper trunk of brachial plexus.

Correct Answer. a

(9). Muscle inserted into iliotibial tract is:

a. Tensor fascia lata

b. Gluteus minimus

c. Vastus medialis

d. Adductor magnus

Solution. Ans 9: (a) Tensor fascia lata Ref Read the text below Sol:
The deep fascia of thigh is known as Fascia lata.
On the lateral aspect of thigh it splits, to enclose the iliotibial tract.
Muscles inserted in the iliotibial tract are tensor fascia lata & gluteus maximus.
Adductor magnus is inserted on the adductor tubercle of femur.
Gluteus medius is inserted on the greater trochanter of femur.

Correct Answer. a

(10). The Anterior division of obturator nerve does not supply :

a. Adductor longus

b. Adductor brevis

c. Pectineus

d. Obturator externus

Solution. Ans 10: (d) Obturator externus Ref Read the text below Sol:
The anterior division of obturator nerve supplies adductor longus, adductor brevis, gracialis & medial half of pectineus.
The posterior division supplies the adductor brevis (if not supplied by the anterior division), adductor magnus, obturator externus &
gives a branch to supply the knee joint.

Correct Answer. d

Copyright 2014 Delhi Academy of Medical Sciences, All Rights Reserved. 6/121
(11). The Thickest nerve of the body is?

a. Radial

b. Median

c. Sciatic

d. Axillary

Solution. Ans-11: (c) Sciatic Ref.: Read the text below Sol :
The sciatic nerve (also known as the ischiadic nerve and the ischiatic nerve) is a large nerve in humans and other animals.
It begins in the lower back and runs through the buttock and down the lower limb. It is the longest and widest single nerve in the human
body, going from the top of the leg to the foot on the posterior aspect.
The sciatic nerve supplies nearly the whole of the skin of the leg, the muscles of the back of the thigh, and those of the leg and foot. It is
derived from spinal nerves L4 through S3. It contains fibres from both the anterior and posterior divisions of the lumbosacral plexus.
The nerve exits the sciatic notch and runs anterior (deep) to piriformis. It then lies posterior (superficial) to the short external rotators
(superior gemellus, inferior gemellus and obturator internus). It then runs down the posterior leg where it breaks into its three main
divisions at the level of the mid thigh. The terminal branches are the common peronealand tibial nerve.[2]

Correct Answer. c

(12). Arterial supply to palatine tonsils are all except:

a. Lingual artery

b. Facial artery

c. Maxillary artery

d. Mandibular artery

Solution. Ans-12: (d) mandibular artery Ref.: Read the text below Sol :
Blood supply is provided by tonsillar branches of five arteries: the dorsal lingual artery (of the lingual artery), ascending palatine artery
(of the facial artery), tonsillar branch (of the facial artery), ascending pharyngeal artery (of the external carotid artery), and the lesser
palatine artery (of the descending palatine artery).
The tonsils venous drainage is by the peritonsillar plexus, which drain into the lingual and pharyngeal veins, which in turn drain into the
internal jugular vein.

Correct Answer. d

Copyright 2014 Delhi Academy of Medical Sciences, All Rights Reserved. 7/121
(13). Posterior communicating artery is a branch of?

a. Posterior cerebral artery

b. Superior cerebral artery

c. Internal carotid artery

d. External carotid artery

Solution. Ans-13: (c) Internal carotid artery Ref.: Read the text below Sol :
The left and right posterior communicating arteries are arteries at the base of the brain that form part of the circle of Willis.
Each posterior communicating artery connects the three cerebral arteries of the same side. Anteriorly, it connects to the internal carotid
artery (ICA) prior to the terminal bifurcation of the ICA into the anterior cerebral artery and middle cerebral artery.
Posteriorly, it communicates with the posterior cerebral artery.

Correct Answer. c

Copyright 2014 Delhi Academy of Medical Sciences, All Rights Reserved. 8/121
(14). Among the following muscles, the one that is found in the first layer of the sole is?

a. Adductor hallucis

b. Flexor digitorum brevis

c. Flexor digiti minimi brevis

d. Flexor hallucis longus

Solution. Ans-14: (b) Flexor digitorum brevis Ref.: Read the text below Sol : The intrinsic muscles in the sole are grouped in four layers:
In the first layer, flexor digitorum brevis is the large central muscle located immediately below the plantar aponeurosis. It flexes the
second to fifth toes and is flanked by abductor hallucis and abductor digiti minimi.

In the second layer, the quadratus plantae, located below flexor digitorum brevis, inserts into the tendon of flexor digitorum longus on
which the lumbricals originates.
In the third layer, the oblique head of adductor hallucis joins the muscle's transversal head on the lateral side of the big toe. Medially to
adductor hallucis are the two heads of flexor hallucis brevis, below the tendon of flexor hallucis longus. The considerably smaller flexor
digiti minimi brevis on the lateral side can be mistaken for one of the interossei.
In the fourth layer. the dorsal and plantar interossei are located between and below the metatarsal bones and act as antagonists. The
central compartment is shared by the lumbricals, quadratus plantae, flexor digitorum brevis, and adductor hallucis; the medial
compartment by abductor hallucis, flexor hallucis brevis, abductor digiti minimi, flexor digiti minimi brevis, and opponens digiti minimi
(often considered part of the former muscle); whilst the lateral compartment is occupied by extensor digitorum brevis and extensor
hallucis brevis

Correct Answer. b

Copyright 2014 Delhi Academy of Medical Sciences, All Rights Reserved. 9/121
(15). Sternocostal triangle is related with all except ?

a. Larrey's) triangle

b. Foramina of Morgagni

c. Superior epigastric arteries

d. External thoracic arteries

Solution. Ans-15: (d) External thoracic arteries


Ref.: Read the text below
Sol :
The sternocostal triangle or foramina of Morgagni (sing.: foramen of Morgagni) are small zones lying between the costal and sternal
attachments of the thoracic diaphragm. Important vessels that pass through these bilateral foramina include the superior epigastric
arteries as terminations of the internal thoracic arteries, with accompanying veins and lymphatics.
Also known as sternocostal hiatus or (Larrey's) triangle

Correct Answer. d

(16). Cardiac catheterization was first attempted by:

a. Werner forssman

b. Willium Harvey

c. Andre Cournard

d. DickinsonRichards

Solution. Ans-16: (a) Werner forssman Ref: Read the text below Sol:
Cardiac catheterization was first attempted by Dr. Werner in 1929 who introduced the catheter in himself with a radiographer holding a
mirror in front of the screen.
He catheterized his heart safely nine times, till he had no peripheral veins left to try.

Correct Answer. a

(17). Hardening of the arterial walls tends to raise :

a. Arterial compliance

b. Diastolic arterial pressure

c. Arterial pulse wave velocity

d. Peripheral resistance

Solution. Ans-17: (c) Arterial pulse wave velocity Ref: Read the text below Sol:
Vibration travels faster in stiff than lax vessels hence the pulse wave velocity increases when the arterial wall becomes hard.
Poor elastic recoil in diastole allows diastolic pressure and compliance to fall.

Correct Answer. c

Copyright 2014 Delhi Academy of Medical Sciences, All Rights Reserved. 10/121
(18). The process of lateral inhibition :-

a. Increases the sensitivity of receptors

b. Promotes sensory adaptation

c. Increases sensory acuity

d. Prevents adjacent receptors from being stimulated

Solution. Ans-18: (c) Increases sensory acuity Ref: Read the text below Sol:
Lateral inhibition and the resultant sharpening of sensation occur within the central nervous system.
Those sensory neurons whose receptive fields are stimulated most strongly inhibitvia interneurons that pass "laterally" within the
CNSsensory neurons that serve neighboring receptive fields.
LATERAL INHIBITION (a) When an object touches the skin (b) Receptors in the central area of the touched skin are stimulated more
than neighboring receptors (c) Lateral inhibition within the central nervous system reduces the input from these neighboring sensory
neurons. Sensation as a result is sharpened within the area of skin that was stimulated the most.

Correct Answer. c

(19). The maximum blood flow per 100 gm of tissue goes to

a. Heart

b. Liver

c. Kidneys

d. Brain

Solution. Ans-19: (c) Kidneys Ref: Read the text Below Sol :
The maximum blood flow per min or the maximum percentage of cardiac output goes to the liver (30% of cardiac output; kidneys receive
25% of CO, brain- 13% ; heart- 4 to 5% of CO) BUT the maximum blood flow per 100 g of tissue goes to kidneys > heart > liver > brain)

Correct Answer. c

(20). Which of the following does not change with age

a. FEV1

b. Glomerular Filtration Rate

c. Glucose Tolerance Test

d. Haematocrit

Solution. Ans-20: (d) Haematocrit Ref: Read the text Below Sol :
Vital capacity, FEV1, MBC, diffusion capacity for oxygen are significantly decreased; FRC is increased by 50%, residual volume is
increased by 100%
Decreased GFR occurs in the elderly because of a 30-40% decrease in the number of glomeruli by the age of 80 years.
Impaired glucose homeostasis is often seen in the elderly. This is due to diminished sensitivity of tissues to insulin as plasma levels of
insulin are unaffected. About 2 hours after administration of a glucose load, plasma glucose is 30 mg% higher at the age of 70 years than
in young adults.
HCT is usually unaffected with age, though the hemopoietic marrow reserve is decreased.

Correct Answer. d

Copyright 2014 Delhi Academy of Medical Sciences, All Rights Reserved. 11/121
(21). Weber -fechner law deals with

a. Frequency discrimination

b. Receptive field organisation

c. Intensity discrimination

d. Two point discrimination

Solution. Ans-21: (c) Intensity discrimination Ref: Read the text Below Sol :
Weber- Fechner law states that sensation felt is directly proportional to the log of the intensity of stimulus

Correct Answer. c

(22). The neurotransmitter involved in the PGO spikes of REM sleep is

a. Norepinephrine

b. Epinephrine

c. Serotonin

d. Acetylcholine

Solution. Ans-22 : (d) Acetylcholine Ref: Read the text Below Sol :
The discharge of the cholinergic PGO- on cells in the lateral pontine tegmentum produces the PGO spikes characteristic of REM sleep.
The PGO spikes pass rapidly to the lateral geniculate body and thence to the occipital cortex.

Correct Answer. d

(23). Active reabsorption of glucose appears to occur in

a. Proximal tubule

b. Loop of henle

c. Distal tubule

d. All of the above

Solution. Ans-23: (a) Proximal tubule Ref: Read the text Below Sol :
Secondary active reabsortion of glucose by the SGLT2 mechanism in the apical membrane of the PT cells is responsible for 100%
absorption of the filtered glucose

Correct Answer. a

(24). In complete diabetes insipidus, the urine volume is

a. 5L

b. 10

c. 15

d. 40L

Solution. Ans-24: (c) 15 Ref: Read the text Below Sol :


ADH regulates water output by acting on the collecting ducts. It is responsible for a maximum of 14.7% (13-15%) of the total water
reabsorbed from the filtrate.
In the absence of ADH, the CD epithelium is relatively impermeable to water.
The impermeability of the distal portions of the nephron is not absolute; along with the salt that is pumped out of the collecting duct
fluid, about 2% of the filtered water is reabsorbed in the absence of ADH.
However, as much as 13% (or 23.3L) of the filtered water may be excreted, and the urine flow may reach 15 mL/min.

Correct Answer. c

Copyright 2014 Delhi Academy of Medical Sciences, All Rights Reserved. 12/121
(25). The aquaporin the thin descending limb of loop of henle is

a. Aquaporin 1

b. Aquaporin 2

c. Aquaporin 3

d. Aquaporin 4

Solution. Ans-25 : (a) Aquaporin 1 Ref: Read the text Below Sol :
Till date 13 aquaporins have been cloned.
Only 4 aquaporins play a key role in the kidney. Aquaporin 1 is present in both the apical and basolateral membrane of the PT cells.
The thin descending limb of the loop of henle reabsorbs water via aquaporin1.
The key to the action of ADH on the collecting ducts is aquaporin-2. Aquaporins- 3 & 4 are found on the basolateral membrane sof the
CD cells

Correct Answer. a

(26). Which of these hormones may have a primary role in many circadian rhythms ?

a. Estradiol

b. Insulin

c. Adrenocorticotropic hormone

d. Melatonin

Solution. Ans-26: (d) Melatonin Ref: Read the text below Sol:
The principal hormone of the pineal gland is melatonin.
Production and secretion of this hormone is stimulated by activity of the suprachiasmatic nucleus (SCN) in the hypothalamus of the
brain via activation of sympathetic neurons to the pineal gland.
The SCN is the primary center for circadian rhythms in the body : rhythms of physiological activity that follow a 24-hour pattern. The
circadian activity of the SCN is automatic, but environmental light/dark changes are required to entrain (synchronize) this activity to a
day/night cycle.
Activity of the SCN and thus secretion of melatonin, begins to increase with darkness and peaks by the middle of the night. During the
day, neural pathways from the retina of the eyes to the hypothalamus act to depress the activity of the SCN, reducing sympathetic
stimulation of the pineal and thus decreasing melatonin secretion.
The pattern of melatonin secretion is altered when a person works night shifts or flies across different time zones. There is evidence that
exogenous melatonin (taken as a pill) may be beneficial in the treatment of jet lag, but the optimum dosage is not currently known.
Phototherapy using bright fluorescent lamps, which act like sunlight to inhibit melatonin secretion, has been used effectively in the
treatment of seasonal affective disorder (SAD), or "winter depression".

Correct Answer. d

(27). The IKATP is a potassium channel that is inhibited by physiologic intracellular concentrations of ATP. How is this channel activated?

a. Physical opening of the channel pore by the N-terminal portion of the channel

b. Chemical ligand binding in response to depletion of ATP from ischemia

c. Conformational changes in channel structure

d. The channel is only inhibitory and is not activated

Solution. Ans-27: (b) Chemical ligand binding in response to depletion of ATP from ischemia Ref: Read the text below Sol:
The IKATP potassium channel is inactivated by chemical ligand binding in response to ischemia and depletion of ATP.
Ventricular myocytes have high densities of these channels and their activation accounts for the ST elevation on the ECG seen during a
MI.

Correct Answer. b

Copyright 2014 Delhi Academy of Medical Sciences, All Rights Reserved. 13/121
(28). The sinus node is predominantly characterized by depolarization in which phase of the action potential?

a. Phase 1

b. Phase 2

c. Phase 3

d. Phase 4

Solution. Ans-28: (d) Phase 4 Ref: Read the text below Sol:
The sinus node is predominantly characterized by its phase 4 depolarization, which accounts for the pacemaker activities.
There are few sodium channels and the upstroke is primarily mediated by ICa,L. There is no discernible phase 1.
The lack of IK1, which is active in phase 3, accounts for the relative depolarized state of the tissue.

Correct Answer. d

(29). All of the following statements regarding the AV node are true except:

a. Conduction through the node displays decremental behavior

b. It is positioned in the subendocardium at the base of the triangle of Koch

c. It is composed of nodal cells and transitional cells

d. It is a right atrial structure

Solution. Ans-29 : (b) It is positioned in the subendocardium at the base of the triangle of Koch Ref: Read the text below Sol:
The AV node is positioned in the low RA at the apex, rather than the base, of the triangle of Koch.
The triangle of Koch is comprised of the ostium of the CS, tendon of Todaro, and septal attachment of the tricuspid valve leaflet.
The region within the triangle is comprised of nodal and transitional cells.

Correct Answer. b

(30). In which of the following tissues is the upstroke of the action potential generated by ingoing calcium currents?

a. Atrial

b. AV node

c. His-Purkinje

d. Ventricular

Solution. Ans-30 : (b) AV node Ref: Read the text below Sol:
Both the AV and sinoatrial nodal cells lack INa.
Conduction is mediated in these tissues by ICa,L.

Correct Answer. b

Copyright 2014 Delhi Academy of Medical Sciences, All Rights Reserved. 14/121
(31). Most important source of reducing equivalents for fatty acid synthesis in liver is?

a. Glycolysis

b. TCA cycle

c. Uronic acid pathway

d. HMP pathway

Solution. Ans-31: (d) HMP pathway Ref.: Read the text below Sol :
HMP shunt path is most important path for production of NADPH. Fatty acid synthesis requires NADPH which is produced mainly by
HMP shunt pathway.
HMP shunt pathway occurs in cytosol of the cell and it produces ribose and NADPH.
Ribose is utilized for nucleotide production and NADPH is utilized for reduction biosynthesis

Correct Answer. d

(32). All the following statements are true of cellular phospholipid synthesis except :

a. Phosphatidylcholine is the primary phospholipid synthesized

b. Steps in lipid synthesis are catalyzed by ER enzymes that face the cytosol

c. Transport vesicles leave the ER and supply the mitochondria and peroxisomes with newly synthesized phospholipid

d. Newly formed phospholipid molecules are added to the cystolic half of the lipid bilayer

Solution. Ans-32: (c) Transport vesicles leave the ER and supply the mitochondria and peroxisomes with newly synthesized phospholipid
Ref.: Read the text below Sol :
Phospholipid synthesis occurs in the ER. Phosphatidylcholine (Lecithin) is the major phospholipid synthesized by the cell. Initial
formation of phosphatidic acid occurs in the cytosolic half of the ER membrane.
Additionally, enzymes required for the processing and synthesis of phosphatidic acid to form diacylglycerol and phosphatidylcholine are
also located on the cytosolic half of the membrane.
Although newly formed molecules of phospholipid are added to the cytosolic half of the ER membrane.
Enzymes called phospholipases are responsible for the flip-flop of phospholipid to the luminal side of the membrane.
These enzymes are specific for different head groups of the phospholipids. The mitochondria and peroxisomes are not part of the
continuous internal membrane-plasma membrane communication system.
Therefore, transport vesicles do not transport phospholipid to these organelles.
There is a ystem of water soluble carrier proteins, known as phospholipid exchange proteins, that is responsible for the transport of
phospholipid to the mitochondria and peroxisomes. This process is believed to occur through a random exchange mechanism based on
concentration gradients

Correct Answer. c

(33). Northern blot is used for the separation of?

a. mRNA

b. DNA

c. Protein

d. Protein DNA interaction

Solution. Ans-33: (a) mRNA Ref.: Read the text below Sol :
Northern blotting is used to estimate the gene expression via estimating the expression of mRNA.

Correct Answer. a

Copyright 2014 Delhi Academy of Medical Sciences, All Rights Reserved. 15/121
(34). Regarding cytosolic gene expression in eukaryotes false is?

a. Capping helps in attachment of mRNA to 40S ribosome

b. N formly methionine t RNA will be first RNA to come into action

c. EF2 shifts between GDP and GTP

d. Relasing factor relases the polypeptide chain from P site

Solution. Ans-34: (b) N formly methionine t RNA will be first RNA to come into action Ref.: Read the text below Sol :
It is methionine which acts as an initiating amino acid for the synthesis of polypeptide chain in eukaryotes.
Formyl methionine is the initiating amino acid for the synthesis of polypeptide chain in prokaryotes

Correct Answer. b

(35). Poly A tail is translated in to ?

a. Polysine

b. Polyalanine

c. Polyarginie

d. Polycysteine

Solution. Ans-35: (a) Polysine Ref.: Read the text below Sol :
Poly A tail is translated into the polylysine tail.
As AAA is the codon for lysine amino acid.

Correct Answer. a

(36). Telomerase : The correct statement is all except?

a. The RNA component acts as a template for the synthesis of segment of the DNA

b. It adds telomere to the 5 end of the DNA strand

c. Provides the mechanism for replicating the ends of the linear chromosome

d. It recognizes the G rich single strand of the DNA

Solution. Ans-36: (b) It adds telomere to the 5 end of the DNA strand Ref.: Read the text below Sol : Telomerase
It is a ribonuceoprotein complex with a short strand of RNA (3 CCCAATCCC5) and various proteins which is having reverse
transcriptase activity.
It adds 6 nucleotide telomere repeat at the 3 end of the DNA chain. Telomere is G rich sequence : TTAGGG
Telomerase binds to the end of 3 end, with part of telomerase RNA hydrogen bonded to last few nucleotide of the chromosome.
Telomere finally terminates in to a single stranded overhang that is roughly 150 nucleotide long.
Telomere is critical for maintaining the stability of the genome. Progressive shortening of the telomere is avoided by telomerase activity

Correct Answer. b

Copyright 2014 Delhi Academy of Medical Sciences, All Rights Reserved. 16/121
(37). The Enzyme ------------ is used in recombinant DNA technology :

a. Restriction endonuclease

b. Pyridoxine dehydrogenase

c. RNA polymerase II

d. DNA ligase

Solution. Ans-37 : (a) Restriction endonuclease Ref.: Read the text below Sol :
Werner Arber showed that certain enzymes of bacteria restrict the netry of phages into host bacteria. Hence, the name restriction
endonucleases. Hamilton Smith in 1970 isolated the first restriction enzyme beta Hindi.
Daniel Nathans in 1971 for the first time applied this restriction enzyme to cut SV 40 DNA. All the three scientists received Nobel prize
in 1978.
The Restriction endonucleases are named after the species and strains of bacteria and the order of discovery. For example, the enzyme
Eco RI is isolated from Escherichia Coli RY13 strain.
The Roman numberal one indicates the order discovery of an enzyme from that species. Restriction enzymes are isolated from
bacteria. Bacterial DNA is not broken by RE, because restriction sites are protected by site specific methylation.

Correct Answer. a

(38). First step in fatty acid synthesis involve?

a. Acetyl Co A carboxylase

b. hydroxy Co A dehydrogenase

c. Acetyl dehydrogenase

d. Pyruvate kinase

Solution. Ans-38: (a) Acetyl Co A carboxylase Ref.: Read the text below Sol :
Acetyl Co A carboxylase otherwise called malonyl Co A synthetase cartalyses conversion of acetyl Co A to malonyl Co A and this is the
rate limiting enzyme for fatty acid synthesis

Correct Answer. a

Copyright 2014 Delhi Academy of Medical Sciences, All Rights Reserved. 17/121
(39). The adenylate cyclase system is mediated by :

a. cAMP

b. Phosphodiesterase

c. GTP regulating proteins

d. Nuclear receptors

Solution. Ans-39 : (c) GTP regulating proteins Ref.: Read the text below Sol :
Adenylyl cyclase plays a very important role in such cellular communication pathways.
This enzyme catalyzes the conversion of ATP to cyclic AMP (cAMP). Adenylyl cyclase is an integral membrane protein.

The extra cellular messenger, (the hormone) combines with the specific receptor on the plasma membrane forming a complex.
This complex activates the regulatory component of the protein designated as G protein or nucleotide regulatory protein.
G proteins are so named, because they are bound to GTP.

Correct Answer. c

Copyright 2014 Delhi Academy of Medical Sciences, All Rights Reserved. 18/121
(40). Tyrosinemia type II is due to deficiency of?

a. Tyrosine transaminase

b. Fumaryl acetoacetyl hydrolase

c. Tyrosine hydroxylase

d. Tyrosinase

Solution. Ans-40 : (a) Tyrosine transaminase Ref.: Read the text below Sol :
Tyrosinemia type II is due to deficiency of tyrosine transaminase and tyrosinemia type I is due to deficiency of fumaryl acetoacetyl
hydrolase.

Correct Answer. a

(41). In Maple syrup urine disease the amino acids excreted in the urine are?

a. Leucine

b. Isoleucine

c. Valine

d. All of the above

Solution. Ans-41: (d) All of the above Ref.: Read the text below Sol :
Maple syrup urine disease is due to excretion of ketoacid of all branched amino acid (valine, leucine, isoleucine).
It is due to deficiency of ketoacid decarboxylase, which is a example of multifunctional enzyme having catalytic domain of
decarboxylase activity for all the three ketoacid.

Correct Answer. d

(42). Regarding nitrogen metabolism, all are false except :

a. Ammonia is the most important nitrogen containing substance in urine

b. There is breakdown and resynthesis of 56 gms of protein everyday

c. The enzymes aminotransferase and glutamate dehydrogenase are the main enzymes in the formation of urea

d. Excess of aminoacid are used in the synthesis of proteins

Solution. Ans-42 : (c) The enzymes aminotransferase and glutamate dehydrogenase are the main enzymes in the formation of urea Ref.:
Read the text below Sol : L-GLUTAMATE DEHYDROGENASE OCCUPIES A CENTRAL POSITION IN NITROGEN METABOLISM
The amino groups of most amino acids ultimately are transferred to ketoglutarate by transamination, forming L-glutamate.
Release of this nitrogen as ammonia is then catalyzed by L glutamate dehydrogenase, a ubiquitous enzyme of mammalian tissues that
uses either NAD or NADP as oxidant.
The net conversion of amino groups to ammonia thus requires the concerted action of glutamate transaminase (also called
aminotransferase) and glutamate dehydrogenase.

Correct Answer. c

Copyright 2014 Delhi Academy of Medical Sciences, All Rights Reserved. 19/121
(43). Methyl folate trap which occurs due to vitamin B12 deficiency is due to

a. Methionine metabolism is decreased

b. Conversion of glutamy CoA to Succinyl CoA

c. Conversion of methyl coA to succinyl CoA

d. Blockin conversion of N5-methyl THFA to THFA

Solution. Ans-43: (d) Blockin conversion of N5-methyl THFA to THFA Ref.: Read the text below Sol :
It is suggested that in man the methyl folate trap is a normal physiological response to impending methyl group deficiency resulting
from a very low supply of methionine. This decreases cellular S-adenosyl-methionine (SAM), which puts at risk important methylation
reactions, including those required to maintain myelin. In order to protect these methylation reactions, the cell has evolved two
mechanisms to maintain supplies of methionine and SAM as a first priority. (a) Decreased SAM causes the folate co-factors to be directed
through the cycle involving 5-methyl-tetrahydrofolate (5-methyl-THF) and methionine synthetase and away from the cycles that produce
purines and pyrimidines for DNA synthesis. This enhances the remethylation of homocysteine to methionine and SAM. In addition, by
restricting DNA biosynthesis and with it cell, division, competition for methionine for protein synthesis is reduced. Thus, whatever
methionine is available is conserved for the vital methylation reactions in the nerves, brain, and elsewhere.
5-methyl-THF, the form in which almost all folate is transported in human plasma, must react with intracellular homocysteine before it
can be retained by the cell as a polyglutamate. Since homocysteine is derived entirely from methionine, methionine deficiency will cause
intracellular folate deficiency, and the rate of mitosis of rapidly dividing cells will be reduced. although these two processes have evolved
as a response to methionine deficiency, they also occur in B12 deficiency, which the cell mistakenly interprets as lack of methionine. the
resulting response is inappropriate and gives rise to a potentially lethal anaemia. In these circumstances the methylation reactions are
also partly protected by the reduced rate of cell division. This explains why administration of folic acid, which induces cell division and
use of methionine in protein synthesis, impairs methylation of myelin and precipitates or exacerbates subacute combined degeneration
(SCD). During folate deficiency methionine biosynthesis is also diminished. As in methionine deficiency, the body responds to decreasing
availability of SAM by diverting folate away from DNA biosynthesis towards the remethylation of homocysteine to methionine and SAM.
The selective use pf available folate to conserve methionine, together with the ability of nerve tissue to concentrate folate form the
plasma, explains the absence of SCD in folate deficiency.

Correct Answer. d

(44). The sequence in respiratory chain in the mitochondria is

a. c-c1-b-a-a3

b. C1-c-b-b1-a3-a

c. b-c1-c-a-a3

d. a-a3-b-c-c1

Solution. Ans-44: (c) b-c1-c-a-a3


Ref.: Read the text below
Sol :
The chart of respiratory chain (ETC) is given below which is self explanatory:
FADH2

NADH Fe.SFMN Fe.S CoQ Cyt b Cyt c1 where
Cyt c Cyt a Cyt a3
Fe.s = Iron sulfur protein
FMN = Flavin mononucleotide
CoQ = Coenzyme Q
Cyt = Various cytochromes (b, c1, c, a, a3)

Correct Answer. c

Copyright 2014 Delhi Academy of Medical Sciences, All Rights Reserved. 20/121
(45). Vitamin B12 acts as a co-enzyme to which of the following enzyme?

a. Isocitrate dehydrogenase

b. Homocystein methyltransferase

c. Glycogen synthase

d. G-6- dehydrogenase

Solution. Ans-45: (b) Homocystein methyltransferase


Ref.: Read the text below
Sol :
This enzyme is needed for conversion of homocystein to the methionine. This is B12 dependent enzyme, which requires methyl THF as a methyl donor. Deficiency of vitamin
B12 leads to accumulation of methyl THF (Folate trap).
As free form of THF is required for the synthesis of N5 N10 methylene THF, a form of THF required for the synthesis of dTMP from dUMP by the enzyme thymidylate
synthetase, the deficiency of B12 leads to defective synthesis of the dTMP and thus that of the DNA and thus megaloblatic anemia.

Correct Answer. b

(46). All are autosomal recessive disorders except:

a. Hereditary spherocytosis

b. Sickle-Cell Disease

c. Tay-Sachs disease

d. Cystic fibrosis

Solution. Ans-46: (a) Hereditary spherocytosis Ref: Read the text below Sol :
Two copies of the gene must be mutated for a person to be affected by an autosomal recessive disorder.
An affected person usually has unaffected parents who each carry a single copy of the mutated gene (and are referred to as carriers).
Two unaffected people who each carry one copy of the mutated gene have a 25% chance with each pregnancy of having a child affected
by the disorder.
Examples of this type of disorder are cystic fibrosis, sickle-cell disease, Tay-Sachs disease, Niemann-Pick disease, spinal muscular
atrophy, and Roberts syndrome. Certain other phenotypes, such as wet versus dry earwax, are also determined in an autosomal recessive
fashion.

Correct Answer. a

(47). Lacunar cells are seen in which type of Hodgkins lymphoma?

a. Nodular sclerosis

b. Mixed cellularity

c. Lymphocyte predominant

d. Lymphocyte depleted

Solution. Ans-47: (a) Nodular sclerosis Ref: Read the text below Sol :
Nodular sclerosis (or "NSHL") is a form of Hodgkin's lymphoma that is the most common subtype of HL in developed countries. It affects
females and males equally and has a median age of onset at ~28 years. It is composed of large tumor nodules with lacunar Reed-
Sternberg cell (RS cells) surrounded by fibrotic collagen bands.
The British National Lymphoma Investigation further categorized NSHL based upon Reed-Sternberg cells into "nodular sclerosis type I"
(NS I) and "nodular sclerosis type II" (NS II), with the first subtype responding better to treatment

Correct Answer. a

Copyright 2014 Delhi Academy of Medical Sciences, All Rights Reserved. 21/121
(48). Malignant Melanoma is related with all except:

a. Chromosome arms 1

b. Chromosome arms 9

c. Chromosome arms 12

d. Chromosome arms 8

Solution. Ans-48: (d) Chromosome arms 8 Ref: Read the text below Sol :
Familial melanoma is genetically heterogeneous, and loci for familial melanoma have been identified on the chromosome arms 1p, 9p
and 12q. Multiple genetic events have been related to the pathogenesis (disease development) of melanoma
The multiple tumor suppressor 1 (CDKN2A/MTS1) gene encodes p16INK4a a low-molecular weight protein inhibitor of cyclin-
dependent protein kinases (CDKs) which has been localised to the p21 region of human chromosome

Correct Answer. d

(49). The predominant feature of a healing wound with separated edges are :

a. Tissue regeneration and restoration

b. Inflammation and granulation tissue formation

c. Wound infection and necrosis

d. Wound contraction and scar formation

Solution. Ans-49: (d) Wound contraction and scar formation Ref: Read the text below Sol : Healing by second intention (wounds with
separates edges) :
Secondary healing differs from primary healing in several respects :
Inevitably, large tissue defects generate a larger fibrin clot that fills the defect and more necrotic debris and exudate that must be
removed. Consequently the inflammatory reaction is more intense
Much larger amounts of granulation tissue are formed.
Perhaps the feature that most clearly differentiates primary from secondary healing is the phenomenon of wound contraction, which
occurs in large surface wounds.

Correct Answer. d

(50). While shaving one morning, a 23- year- old man nicks his lip with a razor. Second after the injury, the bleeding stops. Which of the
following mechanisms is most likely to reduce blood loss from a small dermal arteriole?

a. Protein C activation

b. Vasoconstriction

c. Platelet aggregation

d. Neutrophil chmotaxis

Solution. Ans 50: (b) Vasoconstriction Ref Read the text below Sol
The initial response to injury is arteriolar vasoconstriction , but this is transient and the coagulation mechanism must be initiated to
maintain hemostasis.

Correct Answer. b

Copyright 2014 Delhi Academy of Medical Sciences, All Rights Reserved. 22/121
(51). Polyuria occurs in :

a. Cardiac insufficiency

b. Renal ischemia

c. Prolonged vomiting

d. Diabetic ketoacidosis

Solution. Ans-51: (d) Diabetic ketoacidosis Ref: Read the text below Sol :
Diabetic Ketoacidosis : Diabetic ketoacidosis occurs in patients with Type l diabetes and may be the initial presentation of diabetes.
It is characterized by polyuria, polydipsia, dehydration, anorexia, nausea and vomiting, abdominal pain, tachypnea, obtundation and
coma.

Correct Answer. d

(52). A 53-year-old woman has had nausea, vomiting, and midepigastric pain for 5 months. On physical examination, there are no significant
findings. An upper gastrointestinal radiographic series shows gastric outlet obstruction. Upper gastrointestinal endoscopy shows an
ulcerated mass that is 2 X 4 cm at the pylotus. Which of the following neoplasms is most likely to be seen in a biopsy specimen of this
mass?

a. Non-Hodgkin lymphoma

b. Neuroendocrine carcinoma

c. Squamous cell carcinoma

d. Adenocarcinoma

Solution. Ans 52: (d) Adenocarcinoma Ref Read the text below Sol
The most likely cause of a large mass lesion in the stomach is a gastric carcinoma, and this lesion is an adenocarinoma.

Correct Answer. d

(53). Touton Giant cells are seen in

a. Cat scratch disease

b. Sarcoidosis

c. Xanthomas

d. Neurofibroma

Solution. Ans-53: (c) Xanthomas Ref.: Harshmohan 5th/e p. 148 Sol :


Tauton giant cells :
o These cells have a ring of nuclei that separates a peripheral clear cytoplasm from an eosinophilic central cytoplasm.
o These cells are seen in juvenile xanthogranuloma in skin or iris and in Xanthoma disseminatum

Correct Answer. c

Copyright 2014 Delhi Academy of Medical Sciences, All Rights Reserved. 23/121
(54). Which of the following cells in mucosal surfaces is most instrumental in transmitting HIV to CD4+ T lymphocytes?

a. CD8+ cells

b. Natural killer cells

c. Dendritic cells

d. Neutrophils

Solution. Ans 54: (c) Dendritic cells Ref Read the text below Sol
Three types of cells can carry HIV: dendritic cells, monocytes, and CD4 T cells. Dendritic cells(i.e., Langerhans cells) can bind to the
virus and transport it to CD4+ cells in the lymph nodes.

Correct Answer. c

(55). Epimembranous deposition is seen in

a. Good Pasteur syndrome

b. Membranous GN

c. MPGN

d. MCD

Solution. Ans-55: (b) Membranous GN Ref.: Robbins - 923 Sol :


Epimembranous (along the glomerular capillary wall) deposition is seen in membranous GN and Heymann glomerulonephritis.

Correct Answer. b

(56). All are amyloid stains except

a. Congo RED

b. Thioflavin T

c. PAS

d. Brilliant cresyl blue

Solution. Ans 56: (d) Brilliant cresyl blue Ref Read the text below Sol
Brilliant cresyl blue and new methylene blue are stains for reticulocytes

Correct Answer. d

(57). Hemodialysis associated amyloid is deposited in

a. Synovium

b. Liver

c. Kidney

d. Tongue

Solution. Ans 57: (a) Synovium Ref Read the text below Sol
Patients on long-term hemodialysis for renal failure develop amyloidosis as a result of deposition of 2-microglobulin
Patients often present with carpal tunnel syndrome because of 2- microglobulin deposition.
In some series, over half the patients on lont-term dialysis (>20 years ) developed amyloid deposits in the synovium, joints,or tendon
sheaths.

Correct Answer. a

Copyright 2014 Delhi Academy of Medical Sciences, All Rights Reserved. 24/121
(58). All of the following are true about temporal arteritis except

a. Polymyalgia rheumatica

b. Anemia

c. Low ESR

d. Sudden blindness

Solution. Ans-58 : (c) Low ESR Ref.: Robbins - 519 Sol :


Giant cell arteritis or temporal arteritis :
o It is the most common form of systemic vasculitis in adults.
o It is an acute and chronic, often granulomatous inflammation of arteries of large to small size.
o Giant cell arteritis affects principally the arteries in the head, especially the temporal arteries which is the extracranial branch of
carotid artery.
o Vertebral and ophthalamic arteries may also be involved.
o Segment of affected artery develops nodular thickening with narrowing of lumen.
o There is granulomatous inflammation in the vessel wall with foreign body and langhans type multinucleated giant cells and
fragmentation of internal elastic lamina.

Correct Answer. c

(59). A 63 year old woman experiences a buring sensation in her hands and feet. Two months ago, she had an episode of welling with
tenderness in the right leg, followed by dyspnea and then right-sided chest pain. CBC shows hemoglobin of 13.3g/dl, Hematocrit 40.1%,
MCV 91 um3, platelet count 657,000/mm3 and WBC count of 17,400/mm3. The peripheral blood smear shows abnormally large platelets.
Which of the following is the most likely diagnosis?

a. Essential thrombocythemia

b. Chronic myelogenous leukemia

c. Myelofibrosis with myeloid metaplasia

d. Polycythemia vera

Solution. Ans 59: (a) Essential thrombocythemia Ref Read the text below Sol
Essential thrombocythemia is a myeloproliferative disorder.
As with all myeloproliferative disease, the transformation occurs in a myeloid stem cell. In this form of myeloproliferative disease, the
dominant cell type affected is the megakaryocyte, and hence there is thrombocytosis.

Correct Answer. a

(60). A 3-day-old male infant is diagnosed with meconium ileus. You should be concerned that he has which of the following conditions?

a. Alkaptonuria

b. Cystic fibrosis

c. Hemophilia A

d. Phenylketonuria

e. WILSON disease

Solution. Ans 60: (b) Cystic fibrosis Ref Read the text below Sol
Meconium ileus is a very common early clinical expression of cystic fibrosis. Increased sweat sodium chloride level confirms the
diagnosis. About 70% of children with cystic fibrosis have a deletional abnormality of chromosome 7.
Pancreatic insufficiency, recurrent pulmonary infections, and biliary obstruction may complicate the disorder.
Alkaptonuria is an autosomal recessive disease that causes abnormal pigmentation and degeneration of cartilage.
Hemophilia A is an X-linked hereditary disease caused by a relative lack of coagulation factors VIII or IX.
Phenylketonuria is due to a hereditary lack of the enzyme phenylalanine hydroxylase. Mental retardation is the major clinical finding.

Correct Answer. b

Copyright 2014 Delhi Academy of Medical Sciences, All Rights Reserved. 25/121
(61). All of the following are examples of type II hypersensitivity reaction except:

a. ITP

b. Myasthenia gravis

c. Pernicious anemia

d. Urticaria

Solution. Ans-61: (d) Urticaria


Ref: Read the text below
Sol :

Correct Answer. d

Copyright 2014 Delhi Academy of Medical Sciences, All Rights Reserved. 26/121
(62). Mesothelioma is commonly associated with :

a. Lead

b. Arsenic

c. Asbestos

d. Silica

Solution. Ans-62: (c) Asbestos Ref: Read the text below Sol :
Asbestos enters the body by inhalation, and fine dust may be deposited in the alveoli.
The fibers are insoluble.
The dust deposited in the lungs causes pulmonary fibrosis leading to respiratory insufficiency and death; carcinoma of the bronchus;
mesothelioma of the pleura or peritoneum; and cancer of the gastro-inestinal tract.
In Great Britain, an association was reported between mesothelioma and living within 1 km of an asbestos factory.

Correct Answer. c

(63). Type of anemia in chronic diseases:

a. Microcytic hypochromic

b. Normocytic normochromic

c. Aplastic

d. Macrocytic hyperchromic

Solution. Ans-63: (b) Normocytic normochromic Ref: Read the text below Sol :
Normochromic is a form of anemia in which the concentration of hemoglobin in the red blood cells is within the standard range.
However, there are insufficient numbers of red blood cells.
This includes: aplastic, posthemorrhagic, and hemolytic anemias and anemia of chronic disease.

Correct Answer. b

(64). Basal cell carcinoma commonly spreads by :

a. Lymphatic

b. Haematogenous

c. Direct spread

d. All the above

Solution. Ans-64: (c) Direct spread Ref: Read the text below Sol :
Most common mode of spread of BCC is Direct.
Metastatic spread is most often to lymph nodes, lungs, and bones with lymphogenic and hematogenic spread equally frequent.
Age of sex of the patient seemed to have no influence on survival or way of metastatic spread.
Metastasis of basal cell carcinoma is extremely rare (1 in 1,000 to 35,000)

Correct Answer. c

Copyright 2014 Delhi Academy of Medical Sciences, All Rights Reserved. 27/121
(65). The following is not a feature of Alzheimers disease :

a. Neurofibrillary tangles

b. Senile (neuritic) plaques

c. Amyloid angiopathy

d. Lewy bodies

Solution. Ans-65: (d) Lewy bodies


Ref: Read the text below
Sol :
Disease Region affected Main features Predominant pathology

Cortical atrophy, senile plaques (neuritics), neurofibrillary


Alzheimers disease Cerebral cortex Progressive senile dementia
tangles, amyloid angiopathy

Correct Answer. d

(66). Which of the following drugs is used for the treatment of hemorrhagic stroke ?

a. Nifedipine

b. Nimodipine

c. Propanolol

d. Verapamil

Solution. Ans-66: (b) Nimodipine Ref: KDTs - 532 Sol :


It is used to prevent reflex vasospasm following subarachnoid hemorrhage.
Stroke severity, visible cerebral infarcts on CT, and early commencement of nimodipine treatment were associated.
Overall, high initial systolic and diastolic BP predicted a good functional outcome in patients on nimodipine.

Correct Answer. b

(67). Beta blockers are used in hyperthyroidism

a. As short term symptomatic therapy till effect of carbimazole develops

b. As long term therapy after subtotal thyroidectomy

c. In patient not responding to carbimazole

d. To potentiate the effect of radioactive iodine

Solution. Ans-67: (a) As short term symptomatic therapy till effect of carbimazole develops Ref: Read the text below Sol :
-blockers should be given to most hyperthyroid patients who do not have a contraindication to their use.
Beta blockers are relatively or, depending upon disease severity, absolutely contraindicated in patients with asthma or chronic
obstructive pulmonary disease, severe peripheral vascular disease, Raynaud phenomenon, bradycardia, second or third degree heart
block, and hypoglycemia-prone diabetics in whom the early warning symptoms of hypoglycemia may be masked.
In the absence of contraindications, beta blockers can be administered as soon as the diagnosis of hyperthyroidism is made, even before
obtaining a 24-hour radioiodine uptake.
In patients with Graves disease, -blockers are typically coadministered with a thionamide, but they can be given alone in patients
treated with radioiodine.

Correct Answer. a

Copyright 2014 Delhi Academy of Medical Sciences, All Rights Reserved. 28/121
(68). Which of the following hypnotic drugs facilitates the inhibitory actions of GABA but lacks anticonvulsant or muscle relaxing properties
and has minimal effect on sleep architecture ?

a. Buspirone

b. Diazepam

c. Phenobarbital

d. Zaleplon

Solution. Ans-68: (d) Zaleplon Ref: KDTs - 398 Sol :


Zolpidem, zaleplon and zopiclone are agonists at BZD receptors.
These are hypnotic drugs that lack muscle relaxant and anticonvulsant actions.
These have negligible effect on REM sleep and do not affect sleep architecture.
Risk of tolerance and dependence is much less than benzodiazepines.
Effects of these drugs can be blocked by flumazenil (antagonist of both .1 and .2 receptors.

Correct Answer. d

(69). In which of the following clinical conditions use of anticoagulants provide maximum benefit ?

a. Prevention of recurrences of myocardial infarction

b. Prevention of venous thrombosis and pulmonary embolism

c. Cerebrovascular accident

d. Retinal artery thrombosis

Solution. Ans-69: (b) Prevention of venous thrombosis and pulmonary embolism Ref: KDTs - 603 Sol :
Anticoagulants are mainly used for prophylaxis of venous thrombosis (DVT and pulmonary embolism)
Antiplatelet drugs are used to prevent arterial thrombosis (MI and stroke).

Correct Answer. b

(70). Physical withdrawal symptoms are absent in

a. Alcohol

b. Cannabis

c. Opium

d. Pethidine

Solution. Ans-70: (b) Cannabis Ref: Read the text below Sol :
Cannabis do not cause physical withdrawal symptoms as it does not cause physical dependence.
Cannabis, like cocaine and caffeine, cause only psychological dependence.

Correct Answer. b

(71). Which anesthetic agent should not be used with adrenaline or adrenaline like drugs

a. Thiopental

b. Halothane

c. Nitrous oxide

d. None

Solution. Ans-71: (b) Halothane Ref: Read the text below Sol :
Halothane sensitizes heart towards the arrhythmogenic action of adrenergic drugs.

Correct Answer. b

Copyright 2014 Delhi Academy of Medical Sciences, All Rights Reserved. 29/121
(72). True about Esmolol :

a. It is blocker

b. It has no intrinsic activity

c. It has longer half life

d. It cause tachycardia

Solution. Ans-72: (d) It is a cardioselective Ref: Read the text below Sol :
It is devoid of any intrinsic partial agonistic activity (which is seen in Pindolol & Acebutalol).
Esmolol has a small half life of even < 10 minutes due to rapid hydrolysis by plasma esterases.
It does not cause tachycardia and cause bradycardia like most other beta-blockers.

Correct Answer. d

(73). A patient undergoing general anesthesia experiences muscle rigidity upon intravenous injection of succinylcholine for tracheal
intubation. The body temperature rises precipitously. A drug that may be useful in this situation is

a. Carbamazepine

b. Valproic acid

c. Dantrolene

d. Clonazepam

Solution. Ans-73: (c) Dantrolene Ref: Read the text below Sol :
The patient has develop malignant hyperthermia, for which the drug of choice is dantrolene.
Dantrolene is an antispasticity drug, which works peripherally in the muscle by decreasing Ca2+ release from the sarcoplasmic
reticulum.
Carbamazepine is used for grand mal epilepsy. Valproic acid is useful in absence seizures.
Clonazepam is useful in the treatment of absence seizures and myolonic seizures in children.

Correct Answer. c

(74). A patient with renal failure is to undergo abdominal surgery. Which competitive neuromuscular blocking agent could be used in this
patient as adjunct to the surgery ?

a. Pipecuronium

b. Gallamine

c. Atracurium

d. Succinylcholine

Solution. Ans-74 : (c) Atracurium Ref: Read the text below Sol :
The competitive blocking agents, gallamine and pipecuronium are excreted via the kidney and, therefore, would be contraindicated in
patients with renal failure.
Atracurium, on the other hand, can be used in patients with renal failure since it does not rely on the kidney for excretion. It is largely
dependent on a nonenzymatic process of spontaneous molecular change known as Hoffmann elimination.
Succinylcholine is a depolarizing, not competitive, neuromuscular blocking agent.

Correct Answer. c

Copyright 2014 Delhi Academy of Medical Sciences, All Rights Reserved. 30/121
(75). Which of the following antibiotics is not effective against pseudomonas infections ?

a. Tobramycin

b. Ciprofloxacin

c. Cefoperazone

d. Vancomycin

Solution. Ans-75: (d) Vancomycin


Ref: KDTs - 732
Sol :
Drugs effective against pseudomonas are :
o Penicillins : Carbenicillin, Piperacillin, Ticarcillin
o Cephalosporins : Cefoperazone, Ceftazidime,
Cefepime, Moxalactam
o Other beta lactams : Imipenem, Aztreonam
o Aminoglycosides
o Quinolones : Ciprofloxacin, Pefloxacin

Correct Answer. d

(76). small amount of atropine is added to diphenoxylate in order to

a. Suppress associated vomiting of gastroenteritis

b. Augment the anti-motility action of diphenoxylate

c. Block side effects of diphenoxylate

d. Discourage overdose and abuse of diphenoxylate

Solution. Ans-76: (d) Discourage overdose and abuse of diphenoxylate Ref: KDTs 6th Ed. Pg. 664 Sol :
Although diphenoxylate is chemically related to narcotics, it does not have pain- relieving (analgesic) actions like most other narcotics.
In higher doses, however, like other narcotics, diphenoxylate can cause euphoria (elevation of mood) and physical dependence.
In order to prevent abuse of diphenoxylate for its mood-elevating effects, atropine is combined with diphenoxylate in small quantities.
As a result, if Lomotil is taken in greater than recommended doses unpleasant side effects from too much atropine will occur.

Correct Answer. d

(77). Resistance to acyclovir is most commonly due to mutation in a viral gene that encodes a protein that

a. Converts viral RNA into DNA

b. Phosphorylates acyclovir

c. Transports acyclovir into the cell

d. Transports acyclovir out of the cell

Solution. Ans-77: (b) Phosphorylates acyclovir Ref: KDTs - 768 Sol :


Nucleoside/tide analogues (like acyclovir) act by converting to NTPs. First phosphorylation step occurs inside the virus and resistance
occurs if there is mutation in this gene.

Correct Answer. b

Copyright 2014 Delhi Academy of Medical Sciences, All Rights Reserved. 31/121
(78). All of the following act by protein regulation through nuclear receptors, except

a. GnRH

b. Estrogen

c. Androgen

d. Progesterone

Solution. Ans-78: (a) GnRH Ref: Read the text below Sol :
GnRH, being a proteinaceous hormone; acts on surface receptors.
The steroidal hormones cross the cell membrane and act on cytoplasmic or nuclear receptors.

Correct Answer. a

(79). Which of the following does not bind to GABA receptor chloride channels ?

a. Ethanol

b. Alphaxolone

c. Zolpidem

d. Buspirone

Solution. Ans-79: (d) Buspirone Ref: Read the text below Sol :
Buspirone is a new non-sedative anti-anxiety drug. It binds to 5HT-1A receptors as a partial agonist.
Zolpidem and Zopiclone are non-benzodizepines acting on benzodiazepine receptors present on the GABA receptor linked chloride
channels.
Alcohol and Alphaxolone (an injectable anesthetic) have generalized effects but may specifically promote GABA receptor mediated
synaptic inhibition through chloride channel opening.

Correct Answer. d

(80). A female patient suffering from psychosis is taking phenothiazines. She now complains of sudden onset of high-grade fever, muscle
rigidity and altered sensorium. The most likely diagnosis is

a. Malignant hyperthermia

b. Neuroleptic malignant syndrome

c. Tardive dyskinesia

d. Akathesia

Solution. Ans-80: (b) Neuroleptic malignant syndrome Ref: Read the text below Sol :
Neuroleptic malignant syndrome has developed in this patient.
It may occur with high doses of phenothiazines. It lasts 5 10 days after drug withdrawal and may be fatal.
The neuroleptic must be stopped promptly and symptomatic treatment should be given.
Dantrolene and bromocriptine in large doses have been found to be useful.
Antiparkinsonian anticholinergics are of no help in this situation.

Correct Answer. b

Copyright 2014 Delhi Academy of Medical Sciences, All Rights Reserved. 32/121
(81). Exenatide is a newer drug proposed to be used in the treatment of

a. Osteoporosis

b. Diabetes mellitus

c. Hyperparathyroidism

d. Anovulatory infertility

Solution. Ans-81: (b) Diabetes mellitus Ref: KDTs -701 Sol :


Exenatide is a glucagon like peptide analogue which is proposed to be used in the treatment of post prandial hyperglycemia.
Exenatide is believed to facilitate glucose control in at least five ways:
1. Exenatide augments pancreas response (i.e. increases insulin secretion) in response to eating meals; the result is the release of a
higher, more appropriate amount of insulin that helps lower the rise in blood sugar from eating. Once blood sugar levels decrease closer
to normal values, the pancreas response to produce insulin is reduced; however, other drugs (like injectable insulin) are effective at
lowering blood sugar, but can "overshoot" their target and cause blood sugar to become too low, resulting in the dangerous condition of
hypoglycemia.
2. Exenatide also suppresses pancreatic release of glucagon in response to eating, which helps stop the liver from overproducing sugar
when it is unneeded, which prevents hyperglycemia (high blood sugar levels).
3. Exenatide helps slow down gastric emptying and thus decreases the rate at which meal-derived glucose appears in the bloodstream.
4. Exenatide has a subtle yet prolonged effect to reduce appetite, promote satiety via hypothalamic receptors (different receptors than
for amylin). Most people using exenatide slowly lose weight, and generally the greatest
weight loss is achieved by people who are the most overweight at the beginning of exenatide therapy. Clinical trials have demonstrated
the weight reducing effect continues at the same rate through 2.25 years of continued use. When separated into weight loss quartiles,
the highest 25% experience substantial weight loss, and the lowest 25% experience no loss or small weight gain.
5. Exenatide reduces liver fat content. Fat accumulation in the liver or nonalcoholic fatty liver disease (NAFLD) is strongly related with
several metabolic disorders, in particular low HDL cholesterol and high triglycerides, present in patients with type 2 diabetes. It became
apparent that exenatide reduced liver fat in mice and more recently in man.

Correct Answer. b

(82). All of these drugs can be used in the treatment of post menopausal osteoporosis except

a. Alendronate

b. Teriparatide

c. Calcium

d. Thyroxine

Solution. Ans-82: (d) Thyroxine Ref: KDTs - 327 Sol : The drugs used for the treatment of post-menopausal osteoporosis include :
Bisphosphonates : Alendronate, risedronate
Calcium
Vitamin D
Estrogen
Fluoride
SERMs: Raloxifene

Correct Answer. d

(83). Which of the following statements best explains the action of nitroglycerine on coronary vessels

a. It mitigates angina pectoris by increasing total coronary flow

b. It preferentially dilates conducting arteries without affecting resistance arterioles

c. It preferentially dilates autoregulatory arterioles without affecting the larger arteries

d. It mainly decreases the afterload

Solution. Ans-83: (b) It preferentially dilates conducting arteries without affecting resistance arterioles Ref: KDTs -523 Sol :
Nitrates cause redistribution of blood flow without affecting total coronary flow.
Autoregulatory small vessels remain unaffected whereas large conducting vessels are dilated.
Nitrates are predominantly venodilators, therefore decrease mainly preload.

Correct Answer. b

Copyright 2014 Delhi Academy of Medical Sciences, All Rights Reserved. 33/121
(84). A peptide that causes increased capillary permeability and edema is ?

a. Histamine

b. Agiotensin II

c. Bradykinin

d. Renin

Solution. Ans-84: (c) Bradykinin Ref: KDTs - 490, 491 Sol :


Although both histamine as well as bradykinin can cause edema and increase in permeability, histamine is an amine whereas bradykinin
is peptide.

Correct Answer. c

(85). All of the following agents are useful in acromegaly except

a. Bromocriptine

b. Somatostatin

c. Octreotide

d. Nafarelin

Solution. Ans-85: (d) Nafarelin Ref: KDTs - 239 Sol :


Nafarelin is a gonadotropin-releasing hormone agonist (GnRH agonist). By causing constant stimulation of the pituitary, it decreases
pituitary secretion of gonadotropins luteinizing hormone (LH) and follicle stimulating hormone (FSH). Nafarelin may be used in the
treatment of estrogen-dependent conditions (such as endometriosis or uterine fibroids), to treat central precocious puberty, and to
control ovarian stimulation in IVF.
It is normally delivered via a nasal spray.
Nafarelin is a Gn RH agonist. It has no role in acromegaly.

Correct Answer. d

(86). Fastest acting anti-thyroid drug is

a. Lugols iodine

b. Radioactive iodine

c. Propylthiouracil

d. Sodium thiocyanate

Solution. Ans-86: (a) Lugols iodine Ref: Read the text below Sol :
Lugol's iodine, also known as Lugol's solution, first made in 1829, is a solution of elemental iodine and potassium iodide in water
It was also used at one time as a first line treatment for hyperthyroidism, as the administration of pharmacologic amounts of iodine leads
to temporary inhibition of iodine organification in the thyroid gland, a phenomenon called the Wolff-Chaikoff effect.
However it is not used to treat certain autoimmune causes of thyroid disease as iodine-induced blockade of iodine organification may
result in hypothyroidism.
They are not considered as a first line therapy because of possible induction of resistant hyperthyroidism but may be considered as an
adjuvant therapy when used together with other hyperthyrodism medications.

Correct Answer. a

Copyright 2014 Delhi Academy of Medical Sciences, All Rights Reserved. 34/121
(87). A person returns to Delhi from Bangladesh after 2 days and has diarrhea. Stool examination shows RBCs in stool. The likely organism
causing is :

a. Enteropathogenic E. coli

b. Enterotoxigenic E. coli

c. Salmonella typhi

d. Shigella dysenteriae

Solution. Ans-87 : (d) Shigella dysenteriae


Ref: Read the text below
Sol :
Causes of bloody diarrhea :
Organism Incubation period

Shigella > 16h


> 16h
EHEC > 16h
EIEC 2-6 days
Campylobacter jejuni

Correct Answer. d

(88). Which toxin is mediated by C-AMP except :

a. V. cholera 01

b. Heat stable E. coli toxin

c. Heat labile E. coli toxin

d. V. cholera 0137

Solution. Ans-88: (b) Heat stable E. coli toxin


Ref: Ananthnarayan - 273
Sol :
Enterotoxins of E. coli
Stable Heat toxin Heat labile toxin Verocytotoxin = Shiga like toxin

Acts through
activation of CGMP Acts through activation of CAMP Inhibits protein synthesis

Correct Answer. b

(89). Which of the following statements about Pseudomonas aeruginosa is true?

a. It secretes an enterotoxin

b. It contains an endotoxin

c. It possesses both an enterotoxin and endotoxin

d. It possesses neither an enterotoxin nor an edotoxin

Solution. Ans-89 : (c) It possesses both an enterotoxin and endotoxin. Ref: Read the text below Sol :
Many if not all gram-negative bacteria contain an endotoxin.
In addition, many such as Pseudomonas, Shigella, Salmonella, and Vibrio also secrete an exotoxin.

Correct Answer. c

Copyright 2014 Delhi Academy of Medical Sciences, All Rights Reserved. 35/121
(90). Which one of the following statements about Francisella tularensis infections and their treatment is correct?

a. Culture of the suspected infection by the hospital laboratory is important.

b. An effective vaccine is available, but only for use in high-risk groups

c. F tularensis causes a noninvasive focal skin infection

d. Relapses do not occur after treatment

Solution. Ans-90: (b) An effective vaccine is available, but only for use in high-risk groups. Ref: Read the text below Sol :
A vaccine is recommended for high-risk groups, such as sheep handlers, fur trappers, and laboratory workers.
This organism is highly contagious after in vitro growth and only specially equipped laboratories (not hospitals) should attempt to
culture it.
When Francisella tularenis is implanted into skin, it is carried to lymph nodes, where it replicates and causes ulceroglandular tularemia.
These bacteria parasitize fixed macrophages and mononuclear phagocytes, and relapses may occur because of failure to eradicate
intracellular organisms.

Correct Answer. b

(91). A major step in the pathogenesis of listeriosis is :

a. The formation of antigen-antibody complex with resultant complement activation and tissue damage

b. The release of hyaluronidase by L. monocytogenes, which contributes to its dissemination from local sites

c. The antiphagocytic activity of the L. monocytogenes capsule

d. The survival and multiplication of L. monocytogenes within mononuclear phagocytes and host epithelial cells

Solution. Ans-91 : (d) The survival and multiplication of L. monocytogenes within mononuclear phagocytes and host epithelial cells Ref:
Read the text below Sol :
Listeria uses the cellular machinery to move around inside the host cell: It induces directed polymerization of actin by the ActA
transmembrane protein, thus pushing the bacterial cell around.
Listeria monocytogenes, for example, encodes virulence genes that are thermoregulated. The expression of virulence factor is optimal at
37C, and is controlled by a transcriptional activator, PrfA, whose expression is thermoregulated by the PrfA thermoregulator UTR
element. At low temperatures, the PrfA transcript is not translated due to structural elements near the ribosome binding site. As the
bacteria infect the host, the temperature of the host melts the structure and allows translation initiation for the virulent genes.
The majority of Listeria bacteria are targeted by the immune system before they are able to cause infection. Those that escape the
immune system's initial response, however, spread through intracellular mechanisms and are, therefore, guarded against circulating
immune factors (AMI).
To invade, Listeria induces macrophage phagocytic uptake by displaying D-galactose in their teichoic acids that are then bound by the
macrophage's polysaccharide receptors. Other important adhesins are the internalins.[8] Once phagocytosed, the bacterium is
encapsulated by the host cell's acidic phagolysosome organelle. Listeria, however, escapes the phagolysosome by lysing the vacuole's
entire membrane with secreted hemolysin, now characterized as the exotoxin listeriolysin O. The bacteria then replicate inside the host
cell's cytoplasm.

Correct Answer. d

Copyright 2014 Delhi Academy of Medical Sciences, All Rights Reserved. 36/121
(92). Tumbling motility is shown by :

a. Proteus vulgaris

b. Proteus mirabilis

c. Vibrio

d. Listeria

Solution. Ans-92 : (d) Listeria Ref: Read the text below Sol :
Listeria must then navigate to the cell's periphery to spread the infection to other cells. Outside the body, Listeria has flagellar-driven
motility, sometimes described as a "tumbling motility". However, at 37 C, flagella cease to develop and the bacterium instead usurps the
host cell's cytoskeleton to move.
Listeria, inventively, polymerizes an actin tail or "comet", from actin monomers in the host's cytoplasm with the promotion of virulence
factor ActA.
The comet forms in a polar manner and aids the bacteria's migration to the host cell's outer membrane. Gelsolin, an actin filament
severing protein, localizes at the tail of Listeria and accelerates the bacterium's motility.
Once at the cell surface, the actin-propelled Listeria pushes against the cell's membrane to form protrusions called filopods or "rockets".
The protrusions are guided by the cell's leading edge to contact adjacent cells, which then engulf the listeria rocket and the process is
repeated, perpetuating the infection

Correct Answer. d

(93). Following is the growth curve of E. coli growing in a nutrient medium at 35C with both O2 and added CO2 present.

The following descriptions are given for the phases of this bacterial growth curve:

a. Lag phase

b. Log phase

c. Stationary phase

d. Death phase

Solution. Ans 93: (a) Lag phase Ref :Read the text below Sol:
Bacterial growth curves are multiphasic. The lag phase is characterized by lack of growth but not necessarily metabolic activity.
The bacteria are adjusting to their new environment. Depending on the bacteria, the temperature, nutrients, and pH, the
microorganisms start dividing after a few hours and grow logarithmically for 12 to 18 h. Toxins accumulate in the medium and nutrients
become limiting.
Oxygen and CO2 are usually not limiting, as the gases freely diffuse into the growing culture.
When death and growth of cells are equal, the stationary phase occurs. The death phase is characterized by a death rate that is more
rapid than the growth rate. An antibiotic which inhibits protein synthesis would be optimally active in a rapidly dividing culture where
proteins are being rapidly synthesized, that is, the logarithmic phase.
Bacteria introduced into the human host may undergo similar phases of growth.
However, other factors such as host defenses play a major role in limiting logarithmic growth as does accumulation of toxic byproducts
as might occur in a closed-space infection such as an abscess.

Correct Answer. a

Copyright 2014 Delhi Academy of Medical Sciences, All Rights Reserved. 37/121
(94). Which of the following is the structure responsible for T-cell rosettes in the presence of sheep red blood cells?

a. CD2

b. CD3

c. CD4

d. CD8

Solution. Ans-94: (a) CD2 Ref: Read the text below Sol :
CD2 allows for T-cell rosettes of sheep red blood cells and allows T cells to bind to many other cells and surfaces.

Correct Answer. a

(95). Which one of the following statements best describes the rheumatoid factor?

a. It is the antigen initiating the rheumatoid inflammatory process

b. It is an antibody against cellular DNA

c. It consists primarily of DNA

d. It is an antibody against immunoglobulin

Solution. Ans-95: (d) It is an antibody against immunoglobulin. Ref: Read the text below Sol :
Rheumatoid factor is an immunoglobulin (Ig) M molecule with a specificity for the crystallizable fragment (Fc) portion of endogenous
IgG.

Correct Answer. d

(96). A severely neutropenic patient presents with pneumonia. Bronchial alveolar fluid shows dichotomously branching (generally with acute
angles), septate hyphae. What is the most likely causative agent?

a. Aspergillus

b. Cryptococcus

c. Candida

d. Malassezia

Solution. Ans-96: (a) Aspergillus Ref: Read the text below Sol :
Aspergillus spores are commonly airborne. Invasive infections with Aspergillus are controlled by phagocytic cells.
In severe neutropenia, risk of infection is high.

Correct Answer. a

(97). What is a mass of fungal filaments called?

a. Pseudohypha

b. Hypha

c. Mycelium

d. Septum

Solution. Ans-97 : (c) Mycelium Ref: Read the text below Sol :
A mycelium is a mass of hyphae (fungal filaments).

Correct Answer. c

Copyright 2014 Delhi Academy of Medical Sciences, All Rights Reserved. 38/121
(98). A 21-year-old man was bitten by a tick. Two years later, during the course of routine screening for an unknown ailment, a screening
Lyme disease test was performed, which was negative. A Western blot strip (IgG) showed the following pattern:

Which of the following is the correct interpretation of the test?

a. The patient has acute Lyme disease

b. The patient has chronic Lyme disease

c. The pattern may represent nonspecific reactivity

d. The screening test should be repeated

Solution. Ans 98: (c) The pattern may represent nonspecific reactivity Ref :Read the text below Sol:
The serologic diagnosis of Lyme disease is fraught with difficulty. Enzyme immunoassay (EIA) may be insensitive in the early stages of
disease and may lack specificity in advanced stages.
Western blot analysis of antibody is the confirmatory test for Lyme disease, but it, too, is not 100% sensitive and specific.
The Western blot test detects antibodies to proteins and glycoproteins of Borrelia burgdorferi.
Not all of these proteins are specific for the organism. For example, antibodies to Gp66 may reflect a cross-reaction, as many
Gramnegative bacteria have similar glycoproteins.
For this reason, a Western blot showing only antibodies to Gp66 is thought to be a nonspecific immune response.

Correct Answer. c

(99). Which of the following are acid fast positive with 20% sulphuric acid :

a. M. avium

b. M. leprae

c. M. tuberculosis

d. Nocardia

Solution. Ans-99: (c) M. tuberculosis


Ref: Read the text below
Sol :
Features M. tuberculosis M. leprae

Acid fastness
Resist decolorization by 5% H2SO4
Resist decolorization by 20% H2SO4 Absent
Alcohol fastness Present Not possible
Culture Possible Negative
Niacin Positive (also some strain of M. microti)
14-15 hours
12-13 days
Generation time

Correct Answer. c

Copyright 2014 Delhi Academy of Medical Sciences, All Rights Reserved. 39/121
(100). Which of the following is an example of a killed virus vaccine?

a. Jeryl Lynn mumps vaccine

b. Enders measles vaccine

c. Salk poliovirus vaccine

d. Oka varicella-zoster vaccine

Solution. Ans-100: (c) Salk poliovirus vaccine Ref: Read the text below Sol :
Although many of the childhood vaccines like measles, mumps, and chickenpox contain live, attenuated virus, the Salk poliovirus vaccine
contains killed virus.

Correct Answer. c

(101). A florist presents with a subcutaneous lesion on the hand, which she thinks resulted from a jab wound she received while she was
making a sphagnum moss-wire frame for a floral wreath. The lesion has not healed despite use of antibacterial cream and has begun to
spread up her arm with the lymph node raised and red and beginning to look like it might ulcerate, like the original lesion. The lymph
node above is also beginning to redden and is slightly raised. What is most likely to be an appropriate treatment for this infection?

a. Oral itraconazole or potassium iodide

b. Miconazole cream

c. Cortisone cream

d. Oral griseofulvin

Solution. Ans-101: (a) Oral itraconazole or potassium iodide Ref: Read the text below Sol :
This is a classic case of lymphocutaneous sporotrichosis in which a gardener or florist is infected via a puncture wound. The drug of
choice is either itraconazole or potassium iodide (administered orally in milk).
Topical antifungals are not effective, and the cortisone cream would probably enhance the spread of the disease.
Griseofulvin localizes in the keratinized tissues and would not halt the subcutaneous spread of this infection.
Penicillin would have no effect because Sporothrix is not a bacterium

Correct Answer. a

(102). Which of the following is an RNA virus that has a nuclear phase to its replication process?

a. Coronavirus

b. Rhabdovirus

c. Retrovirus

d. Togavirus

Solution. Ans-102: (c) Retrovirus Ref: Read the text below Sol :
The reverse transcriptase of retroviruses makes a DNA copy of the genomic RNA.
This DNA must be integrated into the host-cell DNA in the nucleus for the remaining steps in the replication process to occur.

Correct Answer. c

Copyright 2014 Delhi Academy of Medical Sciences, All Rights Reserved. 40/121
(103). True about salmonella gastroenteritis is:

a. Mainly diagnosed by serological tests

b. Blood and mucous are present in stool

c. Symptoms appear between 4-48 hours

d. Features are mainly due to exotoxin

Solution. Ans-103:(c) Symptoms appear between 4-48 hours Ref: Ananthnarayan - 299 Sol :
The bacterium induces responses in the animal it is infecting, and this is what typically causes the symptoms, rather than any direct
toxin produced.[3] Symptoms are usually gastrointestinal, including nausea, vomiting, abdominal cramps and bloody diarrhea with
mucus. Headache, fatigue and rose spots are also possible. These symptoms can be severe, especially in young children and the elderly.
Symptoms last generally up to a week, and can appear 12 to 72 hours after ingesting the bacterium.
After bacterial infections, reactive arthritis (Reiters syndrome) can develop.
In sickle-cell anemia, osteomyelitis due to Salmonella infection is much more common than in the general population. Though
salmonella infection is frequently the cause of osteomyelitis in sickle-cell anemia patients, it is not the most common cause. The most
common cause of osteomyelitis remains due to Staphylococcus infection.
By the end of the incubation period, the macro-organisms are poisoned by endotoxins that are released from the dead salmonellae. The
local response to the endotoxins is enteritis and gastrointestinal disorder. In the generalized form of the disease, salmonellae pass
through the lymphatic system of the intestine into the blood of the patients (typhoid form) and are carried to various organs (liver,
spleen, kidneys) to form secondary foci (septic form). Endotoxins first of all act on the vascular and nervous apparatus. This is manifested
by increased permeability and decreased tone of the vessels, upset thermal regulation, vomiting and diarrhea. In severe forms of the
disease, enough liquid and electrolytes are lost to upset the water-salt metabolism, to decrease the circulating blood volume and arterial
pressure, and to cause hypovolemic shock. Septic shock may develop. Shock of mixed character (with signs of both hypovolemic and
septic shock) are more common in severe salmonellosis. Oliguria and azotemia develop in severe cases as a result of renal involvement
due to hypoxia and toxemia.

Correct Answer. c

(104). Which of the following statement is true about : Vibrio cholera :

a. There is no natural reservoir

b. Transported in alkaline peptone water medium

c. Halophilic

d. Oxidase negative

Solution. Ans-104: (b) Transported in alkaline peptone water medium


Ref: Ananthnarayan - 303
Sol :
In holding or transport media, Vibrios do not multiply but remain viable.
Holding or transport media Plating media

1. VR medium
2. Caryblair medium 1. Alkaline bile salt age
3. Alkaline peptone water 2. GTTA
4. Monsurs taurocholate tellurite water 3. TCBS (best selective media)
3 and 4 are also enrichment media

Correct Answer. b

Copyright 2014 Delhi Academy of Medical Sciences, All Rights Reserved. 41/121
(105). Which toxin acts by ADP ribosylation :

a. Botulinum toxin

b. Shigella toxin

c. V. cholera

d. Diphtheria toxin

Solution. Ans-105 : (c) V. cholera Ref: Read the text below Sol :
V. cholerae pathogenicity genes code for proteins directly or indirectly involved in the virulence of the bacteria. During infection, V.
cholerae secretes cholera toxin, a protein that causes profuse, watery diarrhea. Colonization of the small intestine also requires the toxin
coregulated pilus (TCP), a thin, flexible, filamentous appendage on the surface of bacterial cells.
The toxin is an oligomeric protein composed of one A subunit and five B subunits (AB5). After entrance into intestinal epithelial cells
(enterocytes) via receptor-mediated endocytosis, the A subunit detaches and becomes activated by proteolytic cleavage, allowing it to
catalyze the ADP-ribosylation of the Gs subunit of the heterotrimeric G protein resulting in constitutive cAMP production. This in turn
leads to secretion of H2O, Na+, K+, Cl-, and HCO3- into the lumen of the small intestine. The entry of Na+ and consequently the entry of
water into enterocytes are diminished as well. The combined effects result in rapid fluid loss from the intestine, up to 2 liters per hour,
leading to severe dehydration and other factors associated with cholera, including a rice-water stool.
Interestingly, the pertussis toxin (also an AB5 protein) produced by Bordetella pertussis acts in a similar manner with the exception that
it ADP-ribosylates the Gi subunit, rendering it inactive and unable to inhibit adenylyl cyclase production of cAMP (leading to
constitutive production)

Correct Answer. c

(106). Perjury means :

a. Contempt of court

b. Cases tried with the help of jury

c. To act according to law

d. False evidence given by a witness after taking oath

Solution. Ans-106 : (d) False evidence given by a witness after taking oath Ref: Read the text below. Sol:
Perjury, also known as forswearing, lying under oath or lying on oath, is the willful act of swearing a false oath or affirmation to tell the
truth, whether spoken or in writing, concerning matters material to a judicial proceeding.
That is, the witness falsely promises to tell the truth about matters which affect the outcome of the case. For example, it is not
considered perjury to lie about one's age unless age is a factor in determining the legal result, such as eligibility for old age retirement
benefits.
Perjury is considered a serious offense as it can be used to usurp the power of the courts, resulting in miscarriages of justice. In the
United States, for example, the general perjury statute under Federal law defines perjury as a felony and provides for a prison sentence
of up to five years.

Correct Answer. d

(107). The defence witness in a murder trial is crossexamined by :

a. Public Prosecutor

b. Defence Counsel

c. Police officer

d. Judge

Solution. Ans-107: (a) Public Prosecutor Ref: Read the text below. Sol:
In India, we have a public prosecutor who acts in accordance with the directions of the judge.
The control of trial is in the hands of the trial judge. Investigation is the prerogative of the police.
The decision to prosecute function attributed to the procurator in continental countries taken in India by the magistrate on the report
submitted by the police. Again, the withdrawal of the prosecution can also be done only with the permission of the court. However, it is
generally believed that traditional right of nulle prosequi is available to the prosecutor.
The defence witness in a murder trial is cross-examined by public Prosecutor

Correct Answer. a

Copyright 2014 Delhi Academy of Medical Sciences, All Rights Reserved. 42/121
(108). Brachycephaly may result as a result of premature fusion of :

a. Sagittal suture

b. Coronal suture

c. Parietal suture

d. Lambdoid suture

Solution. Ans-108 : (b) Coronal suture Ref: Read the text below. Sol:
Normal skull growth occurs perpendicular to each suture. The primary factor that keeps sutures open is ongoing brain growth.
If one or more cranial sutures fuse prematurely (craniosynostois) it may result in an abnormal head shape.
Brachycephaly : Early bilateral coronal suture fusion.
Scaphocephaly : Early fusion of the sagittal suture.
Anterior plagiocephaly : Early fusion of 1 coronal suture.
Posterior plagiocephaly : Early closure of 1 lambdoid suture.
Trigonocephaly : Early fusion of the metopic suture.

Correct Answer. b

(109). Crual index is calculated as :

a. Length of radius 100


Length of humerus

b. Length of tibia 100 Length of femur

c. Length of tibia 100 Length of fibula

d. Length of (humerus + radius 100 Length of (femur + tibia)

Solution. Ans-109 : (b)


Length of tibia 100
Length of femur
Ref: Read the text below. Sol:
The word crural comes from latin crus meaning leg
In this index the two major leg bones-one representing the lower part and the other representing the upper part are compared with each
other.
All indices (cephalic index, medullary index, sterna index, ischiopubic index etc.) are basically comparisons between two measurements
(of bone, hair etc.) the shorter of which generally goes in the numerator.
Thus in cephalic index, breadth-the shorter of the two measurements goes in the numerator.

Correct Answer. b

(110). Professional death sentence can be passed by:

a. Supreme Court

b. Consumer Court

c. Medical Superintendent of a Hospital

d. State Medical Council

Solution. Ans-110: (d) State Medical Council Ref: Read the text below. Sol:
Professional death sentence can be passed by State Medical Council

Correct Answer. d

Copyright 2014 Delhi Academy of Medical Sciences, All Rights Reserved. 43/121
(111). A civil wrong is known as :

a. Mutatis mutandis

b. Pari passu

c. Tort

d. Ultra vires

Solution. Ans-111 : (c) Tort Ref: Read the text below. Sol:
All these four terms are legal terms, of which most relevant to forensic medicine is tort.
Tort means civil wrong, such as applying the plaster so tightly that gangrene sets up in foot.
The meaning of mutatis mutandis is with the necessary changes having been carried out.

Correct Answer. c

(112). Ewings postulates deals with relationship between:

a. Trauma and tumor

b. Trauma and MI

c. Trauma and SAH

d. Trauma and poisoning

Solution. Ans-112: (a) Trauma and tumor Ref: Read the text below. Sol:
Ewings postulates deals with relationship between Trauma and tumor.
Ewings postulate should be satisfied before accepting a relationship between trauma and new growth.

Correct Answer. a

(113). Head of humerus unites with the shaft at the age of:

a. 12 years

b. 14 years

c. 16 years

d. 18 years

Solution. Ans-113: (d) 18 years Ref: Read the text below. Sol:
Medial epicondyle unites with the shaft at 16 years.
Upper end of radius unites with the shaft at 16 years.
Lower end of radius unites with the shaft at 16 years. (These three centres fall upon the lowest line ossification centres around elbow
joint).
Head of femur unites with the shaft at 17 years. (This is the ossification center aroung hip joint it falls on the second line from below)
Lower end of tibia unites with the shaft at 17 years. Lower end of fibula unites with the shaft at 17 years. (These two are the ossification
centers around ankle joint it falls on the second line from below)
Head of humerus unites with the shaft at 18 years.

Correct Answer. d

Copyright 2014 Delhi Academy of Medical Sciences, All Rights Reserved. 44/121
(114). When a group of muscles of a dead body were in state of strong compaction immediately prior to death and remain so even after death,
the condition is termed as:

a. Gas stiffening

b. Rigor mortis

c. Cadaveric spasm

d. Cold stiffening

Solution. Ans-114: (c) Cadaveric spasm Ref: Read the text below. Sol: Cadaveric Spasm (Instantaneous rigor/cataleptic rigidity
Rare condition caused by stiffening of the muscles immediately after death without being preceded by the stages of primary relaxation.
The conditions necessary for its development are :
o Somatic death must occur with extreme rapidity.
o Person must be in state of great emotional tension.
o Muscles must be in physical activity at that time.

Correct Answer. c

(115). Hair grows at the rate of :

a. 0.004 mm/day

b. 0.04 mm/day

c. 0.4 mm/day

d. 4.0 mm/day

Solution. Ans-115: (c) 0.4 mm/day Ref: Read the text below. Sol:
Human hair typically grows at a rate of 0.4 mm/day, and this rate of growth is consistent across the head.
Several factors can influence both the rate and quality of hair growth, including diet, age, and general health

Correct Answer. c

(116). The recommended content of lodine in salt at the consumer level is

a. 10 ppm

b. 15 ppm

c. 20 ppm

d. 30 ppm

Solution. Ans-116: (b) 15 ppm Ref:Read the text below Sol:


The iodization of salt is now the most widely used prophylactic public health measure against endemic goitre.
In India the level of iodization is fixed under the Prevention of Food Adulteration (PFA) Act and is not less than 30 ppm at the production
point, and not less than 15 ppm of iodine at the consumer level (109).
Iodized salt is most economical, convenient and effective means of mass prophylaxis in endemic areas.
Under the national IDD control activities, the Govt. of India proposed to completely replace common salt with iodized salt in a phased
manner.

Correct Answer. b

Copyright 2014 Delhi Academy of Medical Sciences, All Rights Reserved. 45/121
(117). Sentinel surveillance, is done for identifying

a. Missed cases

b. Subclinical cases

c. Index cases

d. Prodromal infection cases

Solution. Ans-117: (a) Missed cases Ref:Park-37 Sol: SENTINEL SURVEILLANCE


No routine notification system can identify all cases of infection or disease. A method for identifying the missing cases and thereby
supplementing the notified cases is required. This is known as sentinel surveillance.
The sentinel data is extrapolated to the entire population to estimate the disease prevalence in the total population.
The advantages of such a system are that the reporting biases are minimized, and feed-back of information to the providers is simplified.

Correct Answer. a

(118). The number of degrees of freedom in a table of 4x4 is

a. 4

b. 6

c. 9

d. 12

Solution. Ans-118: (c) 9


Ref:Park19-705
Sol:
DEGREES OF FREEDOM = (c 1) (r 1)
c = Number of columns
r = Number of rows
(4 1) (4 1) = 9

Correct Answer. c

(119). Which of the following types of bias can be reduced by allowing equal interview time

a. Berkesonian bias

b. Recall bias

c. Selection bias

d. Interviewer bias

Solution. Ans-119: (d) Interviewer bias Ref:Park-69 Sol: Interviewers bias :


Bias may also occur when the interviewer know the hypothesis and also knows who the cases are.
This prior information may lead him to question the cases more thoroughly than controls regarding a positive history of the suspected
causal factor.
A useful check on this kind of bias can be made by noting the length of time taken to interview the average case and the average control.
This type of bias can be eliminated by double blinding

Correct Answer. d

Copyright 2014 Delhi Academy of Medical Sciences, All Rights Reserved. 46/121
(120). Infant mortality does not include

a. Early neonatal mortality

b. Perinatal mortality

c. Post neonatal mortality

d. Late neonatal mortality

Solution. Ans-120: (b) Perinatal mortality Ref:Park-449 Sol: The Eighth Revision of the International Classification of Diseases (ICD)
defined the perinatal period as lasting from the 28th week of gestation to the seventh day after birth. The Ninth Revision (1975) of ICD
added that :
(i) Babies chosen for inclusion in perinatal statistics (this means late total deaths, live births and early neonatal deaths) should be those
above a minimum birth weight, i.e. 1000 g at birth (A birth weight of 1000 g is considered equivalent to gestational age of 28 weeks)
(ii) If the birth weight is not available, a gestation period of at least 28 weeks should be used, and
(iii) Where (i) and (ii) are not available, body length (crown to heel) of at least 35 cm should be used. But the preferred criterion is birth
weight.)

Correct Answer. b

(121). Ring vacination is

a. Given by a ring shaped machine

b. Given to produced a ring lesion

c. Given around 100 yards of a case detected

d. Given around a mile of a case

Solution. Ans-121: (c) Given around 100 yards of a case detected Ref:Read the text below Sol:
Ring vaccination: The vaccination of all susceptible individuals in a prescribed area around 100 yards of a case detected or outbreak of
an infectious disease.
Ring vaccination controls an outbreak by vaccinating and monitoring a ring of people around each infected individual.
The idea is to form a buffer of immune individuals to prevent the spread of the disease.
Ring vaccination was used to control smallpox until the last naturally occurring case in 1977.

Correct Answer. c

(122). Longest incubation period, among the following is of

a. Malaria

b. Hepatitis

c. Leprosy

d. Filaria

Solution. Ans-122: (c) Leprosy Ref: Read the text below Sol:
Although the mode of transmission of Hansen's disease remains uncertain, most investigators think that M. leprae is usually spread from
person to person in respiratory droplets.
The minimum incubation period reported is as short as a few weeks and this is based on the very occasional occurrence of leprosy
among young infants.
The maximum incubation period reported is as long as 30 years, or over, as observed among war veterans known to have been exposed
for short periods in endemic areas but otherwise living in non-endemic areas. It is generally agreed that the average incubation period is
between three and five years.
Leprosy is now known to be neither sexually transmitted nor highly infectious after treatment.
Approximately 95% of people are naturally immune and sufferers are no longer infectious after as little as 2 weeks of treatment.

Correct Answer. c

Copyright 2014 Delhi Academy of Medical Sciences, All Rights Reserved. 47/121
(123). Reconstituted measles vaccine is used within how many hours.

a.

b. 1

c. 2

d. 8

Solution. Ans-123: (b) 1 Ref: Read the text below Sol:


Measles vaccine is usually given as soon as possible after 9 months of age.
Maternal antibodies against measles last longer than other antibodies, so immunization with measles vaccine is often not effective
before 9 months of age. However, in special situations, for instance in urban areas with high measles transmission or where children
below 9 months of age are getting measles, two doses may be given - at 6 months and 9 months of age.
Note: All children between 6 and 9 months of age who are admitted to hospital should be given a dose of measles vaccine. This should
NOT be marked on their immunization cards. Another dose should be given at 9 months of age.
Measles vaccine comes in powder form together with a diluent. Before it can be used it must be reconstituted.
Reconstituted measles vaccine must be used within one hour or disposed of.

Correct Answer. b

(124). Risk of the damage of fetus by maternal rubella is maximum if mother gets infected in

a. 6-12 weeks of pregnancy

b. 20-24 weeks of pregnancy

c. 21-25 weeks of pregnancy

d. 32-36 weeks of pregnancy

Solution. Ans-124: (a) 6-12 weeks of pregnancy Ref:Park19-131 Sol:


The first trimester of pregnancy is the most disastrous time for the foetus as the organs are developing.
If the infection is serious, spontaneous abortion and stillbirth may occur, or the infant may develop multiple defects such as the classical
triad of patent ductus arteriosus, cataract and deafness.
Infection in the second trimester may cause deafness, but those infected after week 16 suffer no major abnormalities.

Correct Answer. a

(125). Which of the following statements is true regarding pertussis

a. Neurological complication rate of DPT is 1 in 50000

b. Vaccine efficacy is more than 95%

c. Erythromycin prevents spread of disease between children

d. None

Solution. Ans-125: (c) Erythromycin prevents spread of disease between children Ref:Park19-138 Sol:
Neurological complication rate of DPT is 1 in 170000
Vaccine efficacy is more than 80 %
Erythromycin prevents spread of disease between children

Correct Answer. c

Copyright 2014 Delhi Academy of Medical Sciences, All Rights Reserved. 48/121
(126). T-cell functions are assessed by

a. Phagocytic index

b. T-cell count

c. Migration inhibition test

d. Immunoglobulin index

Solution. Ans-126: (c) Migration inhibition test Ref:Park19-271 Sol:


In recent years, newer in vitro tests such as lymphocyte transformation test (LTT) and leucocyte migration inhibition test (LMIT) have
been developed. They give a measure of the cell mediated immunity.
These tests have been used to detect subclinical infection.
A disadvantage of these highly sophisticated tests is that they cannot be applied on a mass scale under field conditions.

Correct Answer. c

(127). 0.1 ml of BCG contains

a. 0.050 mg moist weight

b. 0.025 mg moist weight

c. 0.075 mg moist weight

d. 0.100 mg moist weight

Solution. Ans-127: (d) 0.100 mg moist weight Ref:Park19-161 Sol: BCG VACCINATION
For vaccination, the usual strength is 0.1 mg in 0.1 ml volume.
The dose to newborn aged below 4 weeks is 0.05 ml.
This is because the skin of newborn is rather thin and an intradermal injection with full dose (0.1 ml) in some of them might penetrate
into deeper tissue and give rise to local abscess formation and enlarged regional (axillary) lymph nodes.

Correct Answer. d

(128). All of the following are contraindication of DPT except

a. Serious reaction to a previous injection of DPT

b. Parent or sibling with seizures

c. Brain problem that is getting worse

d. Inconsolable crying/screaming within 7 days of DPT immunization in the past

Solution. Ans-128: (d) Inconsolable crying/screaming within 7 days of DPT immunization in the past Ref: Read the text below Sol: The
following events or conditions may have an impact on the risk of a reaction with the DPT vaccine:
Serious reaction to a previous injection of DPT (or any of the components of the vaccine)
Moderate or severe illness
History of seizure(s)
Parent or sibling with seizures
Brain problem that is getting worse
Alterations of consciousness within 7 days of previous DPT vaccine
Inconsolable crying/screaming or collapse/shock-like state within 48 hours of DPT immunization in the past
High fever

Correct Answer. d

Copyright 2014 Delhi Academy of Medical Sciences, All Rights Reserved. 49/121
(129). The earliest indicator of PEM is :

a. Hair changes

b. Edema

c. Underweight for age

d. Restlessness

Solution. Ans-129: (c) Underweight for age


Ref: Read the text below
Sol:
The earliest indicator of PEM is Underweight for age.

Early detection of PEM

The first indicator of PEM is underweight for age.


The most practical method to detect this, which can be employed even by field health workers, is to maintain growth charts.

Weight for height is indicator of acute malnutrition.


Height for age is indicator of chronic malnutrition
Weight for age is indicator of both acute & chronic malnutrition.

Correct Answer. c

(130). The rate adjusted to allow for the age distribution of the population is

a. Perinatal mortality rate

b. Crude mortality rate

c. Fertility rate

d. Age standardized mortality rate

Solution. Ans-130: (d) Age standardized mortality rate. Ref:Park-54 Sol:


Age adjustment or age standardization, which removes the confounding effect of different age structures and yields a single
standardized or adjusted rate, by which the mortality experience can be compared directly.
The adjustment can be made not only for age but also sex, race, parity, etc. Thus one can generate age-sex, and race-adjusted rates.

Correct Answer. d

(131). What is Berkesonian bias

a. Mis-classification bias

b. Recall bias

c. Selection bias

d. Non-response bias

Solution. Ans-131: (a) Mis-classification bias Ref:Read the text below Sol:
In statistics, the term bias refers to several different concepts:
Selection bias, where individuals or groups are more likely to take part in a research project than others, resulting in biased samples.
This can also be termed Berksonian bias[.
o Spectrum bias arises from evaluating diagnostic tests on biased patient samples, leading to an overestimate of the sensitivity and
specificity of the test.
The bias of an estimator is the difference between an estimator's expectation and the true value of the parameter being estimated.
o Omitted-variable bias is the bias that appears in estimates of parameters in a regression analysis when the assumed specification is
incorrect, in that it omits an independent variable that should be in the model.

Correct Answer. a

Copyright 2014 Delhi Academy of Medical Sciences, All Rights Reserved. 50/121
(132). After administrating live vaccine, immunoglobulins are give after

a. 1 week

b. 2 weeks

c. 10 weeks

d. 12 weeks

Solution. Ans-132: (d) 12 weeks Ref:Park-99 Sol:


Normal human Ig is used to prevent measles in highly susceptible individuals and to provide temporary protection (upto 12 weeks)
against hepatitis A infection for travelers to endemic areas and to control institutional & household outbreaks of hepatitis A infection.
Live vaccines should not normally be given for 12 weeks after an injection of normal human Ig, and if a live vaccine has already been
given.
NHIg injection should be deferred for 2 weeks.

Correct Answer. d

(133). As per WHO guidelines, iodine deficiency disorders are endemic in the community when prevalence of goiter in school age children is
more than

a. 1%

b. 5%

c. 10%

d. 15%

Solution. Ans-133: (b) 5% Ref:Read the text below Sol:


It has been recommended that if more than 5% school age children (6-12 yrs) are suffering from goiter, the area should be classified as
endemic to iodine deficiency.
In the present study, a total goiter prevalence rate of 11.3% was found, signifying that in district Solan mild iodine deficiency existed. In
earlier study in 1956, a goiter prevalence of 39.9% was reported in the district.
The decreases in the prevalence of goiter could be possibly attributed to the availability of iodised salt to the beneficiaries.
The WHO/UNICEF/ICCIDD have recom-mended that no iodine deficiency is indicated in a population when median urinary iodine
excretion is 10 mcg/dl, i.e., more than 50% of the urine samples have UIE level of 10 mcg/dl, and not more than 20% of samples have UIE
level of 5 mcg/dl(7).
In the present study, the median urinary iodine excretion of the children studied was 15.0 mcg/dl and only 8.7% of children had UIE
level of less than 5mcg/dl. These findings indicated that there was no biochemical deficiency of iodine in the subjects studied.

Correct Answer. b

(134). Essential fatty acids are lowest in

a. Fish liver oil

b. Sunflower oil

c. Vegetable oils

d. Coconut oil

Solution. Ans-134: (d) Coconut oil Ref:Read the text below Sol:
Essential fatty acids, or EFAs, are fatty acids that humans and other animals must ingest because the body requires them for good
health but cannot synthesize them.
The term "essential fatty acid" refers to fatty acids required for biological processes, and not those that act only as fuel.
Almost all the polyunsaturated fat in the human diet is from EFA. Some of the food sources are fish and shellfish, flaxseed (linseed),
hemp oil, soya oil, canola (rapeseed) oil, chia seeds, pumpkin seeds, sunflower seeds, leafy vegetables, and walnuts.

Correct Answer. d

Copyright 2014 Delhi Academy of Medical Sciences, All Rights Reserved. 51/121
(135). Maximum amount of PUFA is in

a. Palmitic acid

b. Stearic acid

c. Oleic acid

d. Linoleic acid

Solution. Ans-135: (d) Linoleic acid Ref:Read the text below Sol:
Linoleic acid (LA) is an unsaturated n-6 fatty acid. It is a colorless liquid at room temperature. In physiological literature, it has a lipid
number of 18:2(n-6).
Linoleic acid is one of two essential fatty acids that humans and other animals must ingest for good health, because the body requires
them for various biological processes, but cannot synthesize them from other food components
Most of the omega-6 PUFAs consumed in the diet are from vegetable oils such as soybean oil and corn oil and consist of linoleic acid.
Linoleic acid is the precursor for the 20-carbon PUFA, arachidonic acid which is the principal omega-6 PUFA found in humans.
The significance of arachidonic acid is that it serves as the precursor molecule for the synthesis of a family of bioactive lipids called the
eicosanoids (eicosa means 20).

Correct Answer. d

(136). The eye of new born ism,

a. Hypermetropic with regular astigmatism

b. Hypermetropic

c. Hypermetropic with irregular astigmatism

d. Myopic

Solution. Ans-136: (b) Hypermetropic Ref.: Read the text below Sol :
Infant is hypermetropic by 2.5D to 3.0D

Correct Answer. b

(137). A lesion in the left temporal lobe is most likely to affect which visual field quadrant in the left eye:

a. Upper temporal

b. Lower temporal

c. Upper nasal

d. Lower nasal

Solution. Ans-137: (c) Upper nasal Ref.: Read the text below Sol :
A lesion in left temporal lobe will produce right homonymous upper quandrantopia due to the damage of the Meyers loop.
This will involve upper nasal field in the left eye and upper temporal field in the right eye

Correct Answer. c

(138). Mass treatment with azithromycin is indicated if prevalence of trachoma follicles (TF) in 1-9 years populations is more than-

a. 1.10%

b. 2.8%

c. 3.6%

d. 4.4%

Solution. Ans-138: (a) 1.10% Ref.: Read the text below Sol : According to WHO recommendation for initiating the treatment of
antibiotic in trachoma patient , it is indicated if the prevalence is more than 10%. Its a WHO program, S: surgery A: antibiotics F: facial
hygiene E : Enviromental cleanliness

Correct Answer. a

Copyright 2014 Delhi Academy of Medical Sciences, All Rights Reserved. 52/121
(139). All of following occur in herpes zoster ophthalmicus except

a. Pseudodendritic keratitis

b. Anterior stromal invasion

c. Sclero keratitis

d. Endothelitis

Solution. Ans-139: (c) Sclero keratitis Ref.: Read the text below Sol : All four options given in the question are correct but since
sclerokeratitis is least common ,we mark it as the answer

Correct Answer. c

(140). The given flourescein angiography finding of the patient confirms the diagnosis of:

a. ARMD

b. CME

c. CSR

d. DR

Solution. ns-140: (c) CSR


Ref.: Read the text below
Sol :
Central serous retinopathy is an eye condition where fluid accumulates underneath the retina causing distortion and visual loss.
Patients are typically 20-50 years old and often complain of a sudden, painless loss of vision. This condition occurs 8-10 times more commonly in men than in women and
is more common in Caucasians.
It is felt that this may be a stress-related condition as it is more common in patients with Type A personalities, and can induced in rabbits and monkeys with IV
injections of epinephrine.

- Patients may be asymptomatic unless the central part of the retina (macula) is affected. In these cases, they note decreased or blurred vision, distortion (metamorphopsia),
micropsia (small image size), and abnormal color vision (dyschromatopsia). Visual acuity can range from 20/20 to 20/200.
- Clinically, there is a localized retinal elevation (detachment). There may also be a detachment of the outermost layer of the retina called retinal pigment epithelium (RPE).
There may be some small yellow spots in the area of detachment.
- There may be other areas with changes in pigmentation which are believed to correspond to previous CSR episodes.

Correct Answer. c

Copyright 2014 Delhi Academy of Medical Sciences, All Rights Reserved. 53/121
(141). Bacteria that invades intact cornea

a. Staphylococcus aureus

b. Streptococcus pyogenes

c. Neisseria gonorrhea

d. Hemophilus influenzae

Solution. Ans-141: (c) Neisseria


th
gonorrhea
Ref.: Read the text below Kanski 6 edition pg 254
Sol :
L Listeria
H Haemophilus
G Gonorrhea
C - Corynbacterium
Both c and d are right but we will go by N,gonorrhea as it is more virulent

Correct Answer. c

(142). Size of incision in cataract surgery for an adult depends on

a. Stage of maturation of cortex

b. Type of IOL to be implanted

c. Age of the patient

d. Management of the nucleus

Solution. Ans-142: (d) Management of the nucleus


Ref.: Read the text below
Sol :
The 2 main factors are the type of surgery done and the type of IOL used.
The first one is more important factor in determining the incision size.
ECCE superior 6.5 mm
SICS 3.5 mm
Phacoemulsification 2.75 mm
Phaconit - 1.5 mm

Correct Answer. d

(143). What is percentage of endothelial cell loss during Descemts membrane stripping in automated penetrating keratoplasty-

a. 0-5%

b. 10-15%

c. 30-40%

d. 50-60%

Solution. Ans-143: (c) 30-40%


Ref: Read the text below
Sol :
DSAEK is a procedure which involves stripping of the diseased endothelium and descements membrane with the donor graft which comprises of endothelium,descements
membrane and a small strip of posterior stroma.
This donor graft is harvested with a automated keratome. Since no sutures are required ,postop refractive error is least and the rehabilitation time is less compared to
conventional penetrating keratoplasty[PK].
The endothelial loss is more compared to PK and ranges from more than 20% to 25%, hence we mark the answer C.
1. Cornea for keratoplasty is obtained from: Cadaveric eyes
2. Storage medium for cornea: MK medium ie Mc Kaufmann medium]

Correct Answer. c

Copyright 2014 Delhi Academy of Medical Sciences, All Rights Reserved. 54/121
(144). Corneal layer responsible for deturgescence :

a. Bowmans membrane

b. Endothelium

c. Collagen type I

d. Keratin sulphate

Solution. Ans-144: (b) Endothelium Ref.: Read the text below Sol : Relative dehydration (deturgescence) of the stroma which is
maintained by :
o Epithelium, which is largely impermeable to water.
o Endothelial transport system : The endothelial system pumps fluid from the corneal stroma to the aqueous by Na-K ATPase
mechanism.
o Special inter-cellular junction in the endothelium is also responsible to control the fluid traffic :
o The endothelium is responsible for maintaining deturgescence of the corneal stroma.
o These cells do not divide, so the number of endothelial cells is set a birth.
o This number actually decreases naturally as the body ages.

Correct Answer. b

(145). This layer of cornea has no regenerating capacity :

a. Stroma/Bowmans Membrane

b. Endothelium

c. Epithelium

d. Descemets Membrane

Solution. Ans-145: (d) Descemets Membrane Ref.: Read the text below Sol :
Descemets membrane lies between the stroma and the endothelium. The endothelium is just underneath descemets and is only one cell
layper thick.
This layer pumps water from the cornea, keeping it clear. If damaged or disease, these cells will not regenerate.

Correct Answer. d

(146). Posterior ethmoidal sinus drains into :

a. Sphenoethmoidal recess

b. Superior meatus

c. Inferior meatus

d. Middle meatus

Solution. Ans-146: (b) Superior meatus Ref.: Read the text below Sol : The groups of the ethmoidal air cells:
The posterior group (sometimes the posterior ethmoid sinus) drains into the superior meatus above the middle nasal concha; sometimes
one or more opens into the sphenoidal sinus.
The middle group (sometimes the middle ethmoid sinus) drains into the middle meatus of the nose on or above the bulla ethmoidalis.
The anterior group (sometimes the anterior ethmoid sinus) drains into the middle meatus of the nose by way of the infundibulum.

Correct Answer. b

Copyright 2014 Delhi Academy of Medical Sciences, All Rights Reserved. 55/121
(147). Most common causes of inspiratory stridor in newborn is :

a. Subglottic stengois

b. Tracheo-oesophageal fistula

c. Laryngomalacia

d. Dysphagia lusoria

Solution. Ans-147: (c) Laryngomalacia Ref: Read the text below Sol :
Inspiratory stridor is a syndrome of upper airway obstruction, characterized by a harsh sound on inspiration.
Laryngomalacia is the most common cause of inspiratory stridor in the neonatal period and early infancy and accounts for upto 75% of
all cases of stridor.

Correct Answer. c

(148). Sluders neuralgia originates through :

a. Anterior ethmoid nerve

b. Posterior ethmoid nerve

c. Sphenopalatine nerve

d. Trigeminal nerve

Solution. Ans-148 : (a) Anterior ethmoid nerve Ref: Read the text below Sol :
Sluders neuralgia is characterized by neuralgic pain in lower half of face with nasal congestion, rhinorrhea and increased lacrimation.
It is due to neuralgia of sphenopalatine ganglion

Correct Answer. a

(149). Regarding antrochonal polyp which one is true :

a. Origin from maxillary sinus and goes to nasopharnx

b. Present with severe bleeding

c. Bilateral

d. Seen in elderly person

Solution. Ans-149: (a) Origin from maxillary sinus and goes to nasopharnx
Ref: Read the text below
Sol:
Differences between antrochoanal polyp and ethmoidal polyp
Ethmoidal polypi Antrochoanal polyp
Seen in adults Seen in children and adolescents
Males: Females =2:1 to 4:1 Males> Female
Allergy is the common cause Infection is the common cause
Multiple (bunch of grapes) Unilateral
Seen easily on anterior rhinoscopy Seen commonly in post nasal exam
X ray PNS shows hazy maxillary antrum
X ray PNS may show hazy ethmoids and normal maxillary sinuses

Mostly bilateral Usually unilateral


Recurrence is common Recurrence is uncommon
Polypectomy Caldwel luc surgery in recurrent cases

Correct Answer. a

Copyright 2014 Delhi Academy of Medical Sciences, All Rights Reserved. 56/121
(150). MC cause of epistaxis in 3 years old child :

a. Nasal polyp

b. Foreign body

c. Upper respiratory catarrh

d. Trauma

Solution. Ans-150 : (d) Trauma


Ref: Read the text below
Sol:
Trauma is one of the common local causes of epistaxis. It is commonly caused by the act of nose picking in the Little's area of the nose.
This is commonly seen in young children. Acute facial trauma may also lead to epistaxis.
Patients undergoing nasal surgeries may have temporary episodes of epistaxis.

Correct Answer. d

(151). Poorly formed antihelix is seen in

a. Bat Ear

b. Wildermth's Ear

c. Mozart's Ear

d. None.

Solution. Ans-151: (a) Bat Ear


Ref: Read the text below
Sol:
Bat Ear Antihelix poorly formed
Wildermth's Ear helix under developed, antihelix is prominent
Mozart's Ear helix and antihelix are found fused

Correct Answer. a

(152). Sphenoid sinusitis pain is referred most commonly to

a. Occiput

b. Vertex

c. Frontal

d. Temporal region

Solution. Ans-152: (a) Occiput


Ref: Read the text below
Sol:
Sphenoid sinusitis typically causes pain in the occipital region. Although the sphenoid sinuses are less frequently affected, infection in these sinuses can cause
earaches, neck pain, and deep aching at the top of your head.
Acute sphenoid sinusitis is uncommon and usually seen as part of a pansinusitis, an inflammation of all the sinuses on one or both sides of the nose. Chronic sphenoid
sinusitis is also an uncommon condition and is typically treated as part of a combined treatment or surgery for sinusitis in multiple areas.
Treatment for sphenoid sinusitis may be medical or surgical, with initial treatment usually involving antibiotic administration plus painkillers and
antihistamines.

Correct Answer. a

Copyright 2014 Delhi Academy of Medical Sciences, All Rights Reserved. 57/121
(153). CSF Rhinorrhea occurs due to fracture of :

a. Roof of orbit

b. Cribriform plate

c. Frontal sinus

d. Sphenoid bone

Solution. Ans-153: (b) Cribriform


Ref: Read the text below
Sol:
CSF discharging from the nose is known as CSF rhinorrhoea. Cerebrospinal fluid is a clear colorless fluid that bathes the brain and spinal cord, cushioning
them against trauma.
In fact in literal terms the brain and spinal cord floats in the cerebrospinal fluid. The specific gravity of brain is only 4% of that of CSF, hence it could float
easily in the CSF.
The high pressure leaks are commonly encountered in the cribriform area. This is due to the fagility and unique anatomy in this area i.e. (prolongation of the
subarachnoid space along the olfactory filaments).
The leak during these conditions functions as a safety valve alleviating the increased intracranial pressure. These high pressure leaks are associated with slow
growing tumors and 1/4 of them have hydrocephalus.
Pituitary neoplasms are the most common type of intracranial tumor found, next common are the posterior cranial fossa lesions. Direct invasion of the skull base is not the
usual mechanism of this leak.
Closure of these leaks may worsen the condition of the patient if the causative lesion is left untreated.

Correct Answer. b

(154). Chevallet fracture of nasal septum is:

a. Horizontal backwards

b. Vertical backwards

c. Transverse backwards

d. Oblique backwards

Solution. Ans-154: (b) Vertical backwards


Ref: Read the text below
Sol :
Chevallet fracture of septal cartilage results from blows from below.
It runs vertically from the anterior nasal spine upwards to the junction of bony and cartilagenous dorsum of nose.

Correct Answer. b

Copyright 2014 Delhi Academy of Medical Sciences, All Rights Reserved. 58/121
(155). Turban Shaped Epiglottis is seen in -

a. Acute laryngitis

b. Allergic laryngitis

c. Laryngeal tuberculosis

d. Carcinoma larynx

Solution. Ans-155: (c) Laryngeal tuberculosis


Ref: Read the text below
Sol :

Correct Answer. c

(156). A 16-year-old boy has recurrent episodes of facial swelling without urticaria. Family history reveals that two siblings and both parents
have similar symptoms. Which of the following is the most likely diagnosis?

a. Familial C1 inhibitor deficiency

b. Cystic fibrosis

c. Exacerbation of asthma

d. Acquired C1 inhibitor deficiency

Solution. Ans-156: (a) Familial C1 inhibitor deficiency


Ref:Read the text below
Sol:
Hereditary angioedema is an autosomal dominant disease due to a deficiency of C1 inhibitor (C1INH).
The family history and the lack of urticaria suggest the diagnosis. Acquired C1 inhibitor deficiency has the
same clinical manifestations as the inherited form but is associated with lymphoproliferative disorders and lacks
the family history.
Serum sickness is due to the deposition of drug-antibody complexes causing complement activation and
subsequent urticaria, arthralgias, lymphadenopathy, glomerulonephritis, and cerebritis.
Most cases of serum sickness are due to penicillin. Asthma and cystic fibrosis are not associated with facial
swelling.

Correct Answer. a

(157). Which of the following is the most sensitive part of an examiners hand when assessing for vibration?

a. Fingertips

b. Palmar surface of the finger pads

c. Radial surface of the hand

d. Ulnar surface of the hand

Solution. Ans-157: (d) Ulnar surface of the hand


Ref:Read the text below
Sol:
The palmar surfaces of the fingers are most sensitive to texture, size, position, consistency, crepitus, fluid, and
masses. This is why the palm is used for light (1 cm) and deep (4 cm) palpation.
The ulnar surface of the hand is best for vibration and the radialsurface of the hand is best to determine
temperature.

Correct Answer. d

Copyright 2014 Delhi Academy of Medical Sciences, All Rights Reserved. 59/121
(158). Following a lung transplant the hematoxylineosin stained lung biopsy reveals basophilic staining intranunclear inclusions in alveolar
cells. Which of the following is the most likely pathogen?

a. Pneumocystis carinii

b. Cytomegalovirus (CMV)

c. Cryptococcus neoformans

d. Blastomyces dermatitidis

Solution. Ans-158: (b) Cytomegalovirus (CMV)


Ref:Read the text below
Sol:
The patient has the classic basophilic intranuclear inclusion of cytomegalovirus (CMV).
CMV pneumonitis is particularly common in organ transplantation patients, especially bone marrow allograft
recipients (10%-15%) and those with acquired immune deficiency syndrome (AIDS).
CMV hyperimmune globulin given to seronegative bone marrow recipients provides some protection from
contracting the infection.
Ganciclovir is sometimes given to patients before transplantation as a preventive measure.
It can also be used in treatment of CMV. The mortality rate is 85%.

Correct Answer. b

(159). The repeated regurgitation of stomach content is called:

a. Eructation

b. Vomiting

c. Rumination

d. Ingestion

Solution. Ans-159: (c) Rumination


(Ref: Harrisons Principles of Internal Medicine, 17th edn, Page 240)
Sol:
Nausea is a feeling to vomit. Vomiting is an effortless passage of gastric contents into the mouth.
Rumination is repeated regurgitation of stomach contents, which are often rechewed and then reswallowed.

Correct Answer. c

(160). Chronic constipation is not a symptom of:

a. Depression

b. Parkinsonism

c. Hyperthyroidism

d. Spinal cord injury

Solution. Ans-160: (c) Hyperthyroidism


Ref. Harrisons Principles of Internal Medicine, 17th edn, Page 253)
Sol:
There are many causes for chronic constipation. They are:
Medications antidepressants
Disorders of rectum
Hypothyroidism
Depression
Parkinsonism
Note: Hyperthyroidism induces increased motility of the intestines and such patients have diarrhea and not constipation

Correct Answer. c

Copyright 2014 Delhi Academy of Medical Sciences, All Rights Reserved. 60/121
(161). Most of the amount of bilirubin is derived from heme in:

a. Red blood cells

b. Bone marrow

c. Muscles

d. Other tissues

Solution. Ans-161: (a) Red blood cells (Ref. Harrisons Principles of Internal Medicine- 261) Sol.
Most of the bilirubin is produced from the breakdown of red blood cells.
The rest comes from prematurely destroyed erythroid cells in the bone marrow, from hemoproteins such as myoglobin and cytochromes
in the tissues throughout the body.

Correct Answer. a

(162). Alports syndrome is characterized by the following except :

a. Autosomal dominant transmission

b. Collagen type IV defect

c. Lenticonus

d. Sensori neural deafness

Solution. Ans-162: (a) Autosomal dominant transmission Ref.: Harrisons Principles of Internal Medicine- 2469 Sol :
Alports syndrome is characterized by hematuria, thinning or splitting of GBMs, mild proteinuria and chronic glomerulonephritis leading
to renal failure. Other features like sensorinural deafness, lenticonus of anterior lens capsule, rarely mental retardation, or
leiomyometosis.
Aprox 85% are X-linked inheritance.
Most patients have mutation of type IV collagen
Hematuria and lenticonus, together pathonomonic of Alports syndrome. Sometime hematuria may progress to end stage renal failure.
Renal transplantation is usually successful.

Correct Answer. a

Copyright 2014 Delhi Academy of Medical Sciences, All Rights Reserved. 61/121
(163). Newborn 7 days old with vomiting and dehydration clinical examination was normal except for hyper pigmentation of nipple. Electrolytes
Na: 120 mEq, K:9 mEq. Most likely diagnosis

a. Primary hypothyroidism

b. Congenital Adrenal Hyperplasia

c. Panhypopitaitarism

d. Pyloric stenosis

Solution. Ans-163: b) Congenital Adrenal Hyperplasia


Ref.: Read the text below
Sol :
Congenital adrenal hyperplasia (CAH) refers to any of several autosomal recessive diseases resulting from mutations of genes for
enzymes mediating the biochemical steps of production of cortisol from cholesterol by the adrenal glands (steroidogenesis).
Most of these conditions involve excessive or deficient production of sex steroids and can alter development of primary or secondary sex
characteristics in some affected infants, children, or adults
Signs and symptoms
The symptoms of CAH vary depending upon the form of CAH and the sex of the patient. Symptoms can include:
Due to inadequate mineralocorticoids:
vomiting due to salt-wasting leading to dehydration and death
Due to excess androgens:
functional and average sized penis in cases involving extreme virilization (but no sperm)
ambiguous genitalia, in some females, such that it can be initially difficult to determine sex
early pubic hair and rapid growth in childhood
precocious puberty or failure of puberty to occur (sexual infantilism: absent or delayed puberty)
excessive facial hair, virilization, and/or menstrual irregularity in adolescence
infertility due to anovulation
enlarged clitoris and shallow vagina
Genetics
The gene for 21-hydroxylase is found on 6p21.3 as part of the HLA complex. 21-hydroxylase deficiency results in a unique mutation with
two highly homologous near-copies in series consisting of an active gene (CYP21A) and an inactive pseudogene (CYP21P). Mutant alleles
result from recombination between the active and pseudo genes (gene conversion).
Penetrance
Further variability is introduced by the degree of enzyme inefficiency produced by the specific alleles each patient has. Some alleles
result in more severe degrees of enzyme inefficiency. In general, severe degrees of inefficiency produce changes in the fetus and
problems in prenatal or perinatal life. Milder degrees of inefficiency are usually associated with excessive or deficient sex hormone
effects in childhood or adolescence, while the mildest form of CAH interferes with ovulation and fertility in adults.

Correct Answer. b

Copyright 2014 Delhi Academy of Medical Sciences, All Rights Reserved. 62/121
(164). Which one of the following is associated with shift of potassium into cells?

a. Calcium gluconate administration

b. Intravenous furosemide

c. Dialysis

d. Salbutamol inhalation

Solution. Ans-164: (d) Salbutamol inhalation


Ref.: CMDT -899
Sol :
Some important potassium lowering agents used in the treatment of hyperkalemia :
Substance Mechanism of action

Calcium Antagonize cardiac conduction abnormalities

Sodium bicarbonate Distributes potassium into cells

Insulin Distributes potassium into cells

Salbutamol Distributes potassium into cells

Loop diuretics Increase renal potassium excretion

Sodium polystyrene sulphonate Ion exchange resin binds potassium

Hemodialysis Extracorporeal potassium removal

Peritoneal dialysis Peritoneal potassium removal

Correct Answer. d

(165). Translocation of bcr-able gene is characteristically seen in :

a. Chronic myeloid leukemia

b. Acute myeloid leukemia

c. Chronic lymphatic leukemia

d. Acute lymphatic leukemia

Solution. Ans-165: (a) Chronic myeloid leukemia Ref.: CMDT 2007 516 Sol :
CML is characterized by specific chromosomal abnormality, Philadelphia chromosome, a reciprocal translocation between long arm of
chromosome 9 and 22.
The portion of 9q that is translocated contains abl a proto oncogene.
The able gene receives by specific site on 22q that is bcr. The effusion gene bcr/abl produces a novel gene and it differs from normal
transcript of the able gene in that it possesses tyrosine kinase.
Evidence of bcr/able gene is patholgenic of leukemia.

Correct Answer. a

Copyright 2014 Delhi Academy of Medical Sciences, All Rights Reserved. 63/121
(166). Which one of the following is the drug of choice for control of exercise induced bronchial asthma?

a. Theophylline

b. Ipratropium

c. Cromoglycan sodium

d. Prednisolone

Solution. Ans-166: (c) Cromoglycan sodiu Ref.: CMDT -231 Sol : EXERCISED INDUCED ASTHMA :
Exercise induced bronchoconstriction usually begins within 3 minutes after the end of exercise peaks within 10-15 minutes, then
resolve by 60 minutes.
2 Agonist administration before exercise effectively prevents exercise induced bronchoconstriction.
Cromoglycan sodium and nodochromil are long term medications and improve symptom in patients with exercise-induced asthma.
Long term agonist prevention of exercise induced asthma.

Correct Answer. c

(167). Which of the following is the first-line therapy for fulminant ulcerative colitis (UC) ?

a. Cyclosproine

b. Sulfasalazine

c. Methotrexate

d. Adrenal steroids

Solution. Ans-167: (d) Adrenal steroids Ref: Read the text below Sol :
With fulminant ulcerative colitis the goal is to decrease inflammation as soon as possible.
Oral prednisolone may be used or IV prednisolone can be used if poor absorption is a problem.
Sulfasalazine is used to control mild to moderate disease and should NOT be used for severe UC.
Cyclosporine may be used for refractory UC, but it is not a first-line therapy.

Correct Answer. d

(168). Most common cause of sudden death in myocardial infarction is?

a. Ventricular fibrillation

b. Atrial fibrillation

c. Cardiac rupture

d. Cardiac tamponade

Solution. Ans-168: (a) Ventricular fibrillation Ref.: Read the text below Sol :
Most common cause of sudden death in myocardial infarction is arrthmia, mainly ventricular fibrillation.
It occurs mainly in first few house of MI.
The occurrence of Ventricular fibrillation can be reduced by prophylactic administration of intravenous lidocaine.

Correct Answer. a

Copyright 2014 Delhi Academy of Medical Sciences, All Rights Reserved. 64/121
(169). Classical liver lobule is present around :

a. Portal vein

b. Hepatic artery

c. Central vein

d. Bile duct

Solution. Ans-169: (c) Central vein Ref.: Liver and Biliary tract : pg. 834 Sol :
Central vein is present in the centre of hepatic lobule while the six portal triads will be present in the corners.
Portal triad consists of portal vein, hepatic artery and bile duct; these portal triad will be seen in the corners of hepatic lobule.

Correct Answer. c

(170). Metaplasia occurs in all of the following except :

a. Stem cells

b. Squamous cells

c. Columnar cells

d. Transitional cells

Solution. Ans-170: (a) Stem cells Ref.: Cellular responses to stress and toxic insults; page 11; Robbins and cotran Pathologic Basis of
Disease : 8th ed. Sol :
Metaplasia is the results of reprogramming of stem cells that are known to exist in normal tissues, or of undifferentiated mesenchymal
cells present in the connective tissue.
Metaplasia is a reversible adaptive change in which one differentiated epithelial cell type is replaced by another epithelial cell type of
mature type.
Example of columnar to Squamous Metaplasia : in chronic smokers, the normal ciliated columnar epitherlial cells of trachea and bronchi
are replaced by squamous cells.
Example of Squamous to columnar Metaplasia; in Barrett esophagus, in which esophageal Squamous epithelium is replaced by intestinal
like columnar cells under the influence of refluxed gastric acid.
Transitional /uro-epithelium can also undergo Squamous Metaplasia by chronic irritation like in urinary bladder stones

Correct Answer. a

(171). This type of Kaposi sarcoma was predominantly found in South African children.

a. Classical type

b. Lymphadenopathic type

c. AIDS related type

d. Transplant associated type

Solution. Ans-171: (b) Lymphadenopathic type Ref.: Blood vessels : pg. 523 Sol : There are four types of KS :
Classic KS (European KS) not associated with HIV infection
Lymphadenopathic type (Also known as African or Endemic KS) not associated with HIV infection ; more common in African Bantu
children; skin lesions are sparse and usually presents with lymph node enlargement. It is most common with lymph node enlargement. It
is most common in Central Africa.
AIDS related (epidemic) type : Associated with HIV infection; more common in male homosexuals.
Transplant associated type : due to long term immunosuppression therapy

Correct Answer. d

Copyright 2014 Delhi Academy of Medical Sciences, All Rights Reserved. 65/121
(172). Most common cause of acute tubular necrosis is?

a. Snake Bite

b. Diuretics

c. Hypovolumia

d. None

Solution. Ans-172: (c) Hypovolumia Ref.: Read the text below Sol :
Acute tubular necrosis is the most common cause of acute renal failure.
Acute tubular necrosis is most commonly precipitated by renal ischemia or exposure to nephrotoxic substances, including radiographic
contrast agents

Correct Answer. c

(173). Idiopathic episodes of intractable nausea and vomiting is called:

a. Cyclic vomiting

b. Hyperemesis

c. Rumination

d. All of the above

Solution. Ans-173: (a) Cyclic vomiting (Ref. Harrisons Principles of Internal Medicine, 17th edn, Page 241) Sol.
Cyclic vomiting syndrome is a rare disorder of unknown etiology that produces episodes of intractable nausea and vomiting, usually in
children.
This has a close association with migraine headaches.

Correct Answer. a

(174). A 55-year-old man being treated for diabetes, hypertension, and degenerative joint disease becomes depressed. Which of the following is
most likely to be associated with his altered mood?

a. Oral hypoglycemic drugs

b. Antiparasitic drugs

c. Nonsteroidal anti-inflammatory drugs (NSAIDs)

d. Antihypertensive drugs

Solution. Ans:174: (d) Antihypertensive drugs. Ref: Read the text below Sol:
Of the drugs listed, antihypertensive drugs are most likely to cause depression. Other drugs that are strongly associated with depression
are steroids and hypnotic medications.
Psychostimulants can also precipitate depression during drug withdrawal.
Oral hypoglycemic drugs, antiparasitic drugs, nonsteroidal anti-inflammatory drugs (NSAIDs), and selective serotonin reuptake
inhibitory drugs are not commonly associated with depression

Correct Answer. d

Copyright 2014 Delhi Academy of Medical Sciences, All Rights Reserved. 66/121
(175). Single painless ulcer on genitalia is due to

a. Treponema pallidum

b. H. ducreyi

c. LGV

d. Donovanosis

Solution. Ans 175: (a) Treponema pallidum Refernce Read the text below Sol:
Mucosal ulcers affect the genitals, mouth, pharynx & larynx
Early lesions are superficial; alter, they develop a white base & a red margin & coalesce to form snail track ulcers
Single lesion, relatively painless, regular edged and base hard button like ulceration.

Correct Answer. a

(176). Components of metabolic syndrome include all except :

a. Central obesity

b. Type-2 diabetes mellitus

c. Insulin sensitivity

d. Hypertension

Solution. Ans 176: (c) Insulin sensitivity Refernce Read the text below Sol:
In metabolic syndrome or syndrome X there is hyperinsulinemia due to insulin resistance.

Correct Answer. c

(177). A 23-year-old female was brought with pulseless electrical activity (PEA) with self-inflicted wounds. Cardiac monitor shows sinus
tachycardia with a rate of 120 beats per minute, wide QRS complexes, and a prolonged QT interval. The patient received oxygenation by
bag-valve mask, chest compressions, 2 L of normal saline, and 1 mg of intravenous epinephrine. What therapy is indicated next?

a. Atropine

b. Calcium gluconate

c. Sodium bicarbonate

d. Amiodarone

Solution. Ans 177: (c) Sodium bicarbonate. Reference Read the text below Sol:
This patient is presenting with pulseless electrical activity (PEA). PEA is defined by an organized rhythm without a pulse or signs of
perfusion.
More specifically, ventricular tachycardia (VT), ventricular fibrillation, and asystole are excluded from this definition.
Patients with PEA rarely survive to hospital discharge. PEA can be caused by hypovolemia, hypoxia, cardiac tamponade, pulmonary
embolism, tension, pheumothorax, massive myocardial infarction, or ingestion.
Treatment of PEA should focus not only on following an algorithmic approach to treatment and appropriate pharmacologic intervention
with epinephrine and atropine but also on identifying an underlying etiology.
The physical finding of healing, self-inflicted wounds in conjunction with PEA should raise concern for a possible ingestion. Atropine is
indicated in PEA only when a bradycardic rhythm is present.
Calcium would be indicated in the setting of calcium channel blocker toxicity, but the presence of tachycardia makes this ingestion
unlikely. Amiodarone is indicated as a second-line agent for VT after epinephrine, but although these QRS complexes are wide, this
rhythm is sinus in origin.
This patient ingested amitriptyline, a tricyclic antidepressant (TCA). A TCA should be suspected from the clinical presentation and the
characteristic electrocardiogram findings. Sodium bicarbonate along with supportive care is the appropriate intervention.

Correct Answer. c

Copyright 2014 Delhi Academy of Medical Sciences, All Rights Reserved. 67/121
(178). A patient presents with acute rheumatic carditis with fever. True statement is

a. Increased troponin T

b. Reduced myocardial contractility

c. Signs of inflammation and necrosis

d. Valve replacement will ameliorate CCF

Solution. Ans-178: (c) Signs of inflammation and necrosis Ref: Braumwald Heart disease 8th ed Sol: Carditis
Rheumatic carditis is associated with virtually all the major sequelae of RF, including mortality.
Approximately 40 to 60 percent of RF episodes result in RHD.
Carditis typically manifests as valvulitis, detected by the presence of mitral regurgitation (MR) or, less commonly, aortic regurgitation
on auscultation.
It can be responsible for acute and chronic myocardial dysfunction and acute, although not chronic, pericardial disease.

An important consideration is that neither myocarditis nor pericarditis can be expected to occur in the absence of valvulitis and, if
evidence of valvular involvement is not present, alternate nonrheumatic causes must be considered.
An important consideration is that neither myocarditis nor pericarditis can be expected to occur in the absence of valvulitis and, if
evidence of valvular involvement is not present, alternate nonrheumatic causes must be considered.
PHYSICAL EXAMINATION
There are a number of hallmarks of acute carditis noted on physical examination of a patient with an initial episode of RF.
The first heart sound may vary from normal to diminished intensity, either because of MR, prolonged PR interval, or both.
The second heart sound is normally or widely or variably split, depending on the degree of the MR.
The pulmonary component of the second sound is accentuated with the presence of pulmonary hypertension caused by severe MR.
Although classically the aortic second sound is diminished in chronic aortic insufficiency, in acute RF it is usually normal, even with
significant aortic regurgitation, because mobility of the aortic valve is not affected early.
A third heart sound is common, and cannot be used as a marker for severity of MR, because children frequently have an S3 heart sound
without associated pathology.
The soft, blowing, pansystolic murmur of MR is a hallmark of carditis in RF. The murmur is best heard at the apex and selectively
conducted to the axilla and back; the latter suggests severe MR.
The severity of left ventricular dysfunction, even in the acute setting, appears to correlate with the extent of valvulitis rather than with
any myocardial injury.
Rheumatic myocarditis, in the setting of preserved LV function, is not associated with the troponin level elevation seen in viral
myocarditides.
Both echocardiographic data and postmortem pathology findings are consistent with severe heart failure in acute RF, being secondary to
altered myocardial mechanics caused by MR rather than secondary to myocarditis.
Therapeutic Modalities Salicylates and nonsteroidals have no specific role in rheumatic carditis, with the exception of treatment of
concomitant pericarditis.
It has generally been accepted that acute carditis benefits from aggressive antiinflammatory therapy with steroids.
Nevertheless, in the setting of severe, potentially life-threatening heart failure, steroid administration is widespread but empirical;
randomized trials using modern diagnostic tools to assess the effect on cardiac function are required to address the issue.
Treatment of cardiac manifestations otherwise follows established guidelines, including management of congestive heart failure and
severe valvular regurgitation .
Unless valvular regurgitation and severe congestive heart failure are refractory to drug therapy, valve surgery is avoided for acute RF
patients.
Surgical morbidity and mortality have been significant and failed repair leading to valve replacement frequent, although postoperative
ventricular function generally improves significantly, consistent with regurgitation rather than myocardial dysfunction being the primary
mechanism leading to heart failure.
The natural course of the disease without secondary prophylaxis was extensively studied in the preantibiotic era in RHD hospitals where
patients were treated for late manifestations of the disease, virtually all cardiac.
Patients with recurrent episodes of carditis developed severe mitral and aortic valve disease and end-stage congestive heart failure,
often with severe pulmonary hypertension, right heart failure and, occasionally, end-stage liver disease secondary to passive congestion.

Correct Answer. c

Copyright 2014 Delhi Academy of Medical Sciences, All Rights Reserved. 68/121
(179). Chang staging is used for

a. Retinoblastoma

b. Medulloblastoma

c. Ewings sarcoma

d. Rhabdomyosarcoma

Solution. Ans-179: (b) Medulloblastoma Ref: Devita principle and practice of oncology 8th ed P.2009 Sol: Primitive
Neuroectodermal/Embryonal Central Nervous System Neoplasms
The CNS is the second most common site of malignancy in children behind leukemia, with the majority of tumors located infratentorially.
Neoplasms of embryonal origin are found in several locations.
These neoplasms are characterized by sheets of small, round, blue cells with scant cytoplasm.
Medulloblastoma Epidemiology
Medulloblastomas comprise 15% to 30% of CNS tumors in children.
There is almost a 1.5:1 male-to-female predominance, and 70% of cases are diagnosed by age 20.
Medulloblastomas become progressively rarer with increasing age, with few cases seen beyond the age of 50.
Both Gorlin syndrome and Turcot syndrome have increased rates of medulloblastoma, but together account for at most 1% to 2% of
medulloblastoma cases, with the remaining vast majority being sporadic in nature.
Pathology
Medulloblastomas are dense and cellular and classically have Homer-Wright (neuroblastic) rosettes, although these are found in less
than 40% of the cases.
Mitoses are frequent, representing a high proliferative index.
According to the current WHO classification, medulloblastomas are histologically grade 4.
Immunohistochemical analysis is positive for synaptophysin, most prominent in nodules and within the centers of the Homer-Wright
rosettes, correlating with a presumed neuronal progenitor origin.
Besides the classic medulloblastoma two important pathologic variants exist.
The desmoplastic subtype has collagen bundles interspersed with the densely packed undifferentiated cells of the classic subtype as well
as nodular, reticulin-free pale islands, or follicles.
Inactivation mutations of the PTCH gene, which is the underlying genetic anomaly in Gorlin syndrome, are displayed in the desmoplastic
subtype and this variant is seen more frequently in adults than children.
Large cell anaplastic medulloblastoma is relatively rare, present in approximately 4% of cases, and has large nuclei with prominent
nucleoli and more abundant cytoplasm, increased apoptosis, and large areas of necrosis.
This subtype displays a high rate of amplification of the myc oncogene, a known negative prognostic factor, and correlates with a worse
clinical outcome.
Staging/Risk Groups
Staging is critical in treatment planning for patients with medulloblastoma. A modified version of the Chang staging system is currently
used.
T stage has been made less relevant than the extent of residual disease, consequential to advances in neurosurgical techniques, and
therefore is not used in current clinical trials and other treatment determinations.
M stage remains a crucial staging component. M0 is designated as no tumor dissemination, whereas M1 represents the presence of
tumor cells in the CSF. M2 is presence of gross tumor nodules in the intracranial, subarachnoid, or ventricular space, and M3 represents
gross tumor nodules in the spinal subarachnoid space. M4 represents systemic metastasis.

Correct Answer. b

(180). Cause of vasodilatation in spider nevi

a. Estrogen

b. Testosterone

c. Hepatotoxins

d. FSH

Solution. Ans-180: (a) Estrogen Ref: Goodman and Gillman Sol:


Estrogen actions on the vascular wall include increased production of NO, which occurs within minutes via a mechanism involving
activation of Akt (also known as protein kinase B), and induction of inducible NO synthase and increased production of prostacyclin.
All of these changes promote vasodilation

Correct Answer. a

Copyright 2014 Delhi Academy of Medical Sciences, All Rights Reserved. 69/121
(181). A 20 year old man is hit on the eye with a ball.On examination there is restriction of lateral and upward gaze and diplopia. There is no
obvious visible sign of injury to eye ball, but there is some enophthalmos, the most likely diagnosis is :-

a. Zygoma fracture

b. Maxillary fracture

c. Blow out fracture of the orbit

d. Injury to lateral rectus

Solution. Ans 181: (c) Blow out fracture of the orbit Ref Read the text below Sol:
In blow our fracture of orbit, the weakest plate of bone, commonly infraorbital plate, rupture & orbital content herniate downward into
maxillary antrum.
The small fractured piece of bone may entrap the orbital contents particularly inferior oblique & inferior rectus muscle, leading to
failure of eye to rotate upwardLL
Enopthalmos & profound diplopia can follow.
Pain on movement of eye.
Anaesthesia over distribution of infra orbital nerve may be an important clue to the blow - out fracture.
Traction test is done to diagnose entrapment of ocular muscle.

Correct Answer. c

(182). In a case of blunt thoracic trauma thoracotomy will be performed if there is :

a. 500 ml. of blood at the time of initial drainage

b. Continued bleeding at the rate of 15 ml per 15 minutes from the intercostal drains

c. 800ml of blood at the time of initial drainage

d. Rupture of the bronchus

Solution. Ans 182: (d) Rupture of the bronchus


Ref Read the text below
Sol:
Indication for thoracotomy following blunt thoracic trauma are:
1. 1000ml blood at the time initial drainage
2. Continued brisk bleeding (>100ml/15minutes)
3. Continued bleeding of >200 ml/hour from the intercostal drains for three or more hours.
4. Rupture of bronchus, aorta, esophagus or diaphragm
5. Cardiac tamponade

Correct Answer. d

(183). Lymphoedema that starts at puberty is?

a. Milroy disease

b. Lymphoedema congenital

c. Lymphoedema tarda

d. Lymphoedema Praecox

Solution. Ans 183: (d) Lymphoedema Praecox.


Ref Read the text below
Sol:
Lymphedema may be inherited (primary) or caused by injury to the lymphatic vessels (secondary).
Lymphedema may be due to poor development of lymphatics (Milroy disease) presenting at birth (Lymphoedema congenita), develop at the onset of puberty (praecox),
or into adulthood (tarda).

Correct Answer. d

Copyright 2014 Delhi Academy of Medical Sciences, All Rights Reserved. 70/121
(184). Indication for surgery in benign prostatic hypertrophy are all except :

a. Prostatism

b. Chronic retention

c. Hemorrhage

d. Enlarged prostate

Solution. Ans-184: (d) Enlarged prostate


Ref: Bailey and Love- 1349.
Sol :
Strong indications for surgery (Prostatectomy) in Enlarged Prostate:
Acute or chronic urinary retention and renal impairment (Difficulty urinating caused by benign enlargement of the prostate gland is called prostatism)
Complications of bladder outflow obstruction; Persistent or recurrent urinary tract infections, stones and diverticulum formation.
Hemorrhage : Significant hemorrhage or recurrent hematuria.
Elective prostatectomy for severe symptoms; this accounts for 60% of prostatectomies, Increasing difficulty in micturition with considerable frequency day and night, delay in
starting and poor stream are the usual symptoms for which prostatectomy is advised. Frequency alone is not a strong indication for prostatectomy.

Correct Answer. d

(185). Most common cause of acute intestinal obstruction is

a. Adhesions

b. Volvulus

c. Inguinal hernia

d. Internal hernia

Solution. Ans-185: (a) Adhesions


Ref: Bailey and Love-1188
Sol :

Correct Answer. a

Copyright 2014 Delhi Academy of Medical Sciences, All Rights Reserved. 71/121
(186). All of the following abnormalities are related to persistence of the omphalomesenteric duct except?

a. Meckels diverticulum

b. Omphalocele.

c. Enterocutaneous fistula

d. Umbilical sinus

Solution. Ans-186: (b) Omphalocele.


Ref: Read the text below
Sol :
Omphalocele results from abnormal closure of the abdominal wall and is not related to the persistence of the omphalomesenteric duct. During normal development of the
human embryo, the midgut herniates outward through the umbilical ring and continues to grow. By the 11th week of gestation, the midgut returns back into the abdominal cavity
and undergoes normal rotation and fixation, along with closure of the umbilical ring. If the intestine fails to return, the infant is born with abdominal contents
protruding directly through the umbilical ring, termed an omphalocele.
Most commonly, a sac is still covering the bowel, thus protecting it from the surrounding amniotic fluid. In contrast with gastroschisis, karyotype abnormalities are present
in roughly 30% of infants, including trisomies 13, 18, and 21.
More than half of infants with omphalocele have other major or minor malformations, with cardiac being the most common, followed by musculoskeletal,
gastrointestinal, and genitourinary. There is also a close association with Beckwith-Wiedemann syndrome (omphalocele, hyperinsulinemia, and macroglossia). In contrast with
omphalocele, the defect seen with gastroschisis is always on the right side of the umbilical ring with an intact umbilical cord, and there is never a sac covering the abdominal
contents.
Meckels diverticulum occurs when there is persistence of intestinal end of the duct while persistence of duct throughout its course produces and
enterocutaneous fistula.

Correct Answer. b

Copyright 2014 Delhi Academy of Medical Sciences, All Rights Reserved. 72/121
(187). Sac in cloquet hernia is?

a. Below psoas major

b. Below femoral vessels

c. Below pectenius fascia

d. Below arcuate line

Solution. Ans-187: (c) Below pectenius fascia.


Ref: Read the text below
Sol :

Abdominal Examination Signs


SIGN DESCRIPTION DIAGNOSIS/CONDITION
Pain or pressure in epigastrium or anterior chest with persistent firm
Aaron sign Acute appendicitis
pressure applied to McBurney's point
Sharp pain created by compressing appendix between abdominal wall and
Bassler sign Chronic appendicitis
iliacus
Blumberg's sign Transient abdominal wall rebound tenderness Peritoneal inflammation
Loss of abdominal tenderness when abdominal wall muscles are
Carnett's sign Intra-abdominal source of abdominal pain
contracted
Chandelier sign Extreme lower abdominal and pelvic pain with movement of cervix Pelvic inflammatory disease
Charcot's sign Intermittent right upper abdominal pain, jaundice, and fever Choledocholithiasis
Claybrook sign Accentuation of breath and cardiac sounds through abdominal wall Ruptured abdominal viscus
Courvoisier's sign Palpable gallbladder in presence of painless jaundice Periampullary tumor
Cruveilhier sign Varicose veins at umbilicus (caput medusae) Portal hypertension
Cullen's sign Periumbilical bruising Hemoperitoneum
Danforth sign Shoulder pain on inspiration Hemoperitoneum
Abdominal wall mass that does not cross midline and remains palpable
Fothergill's sign Rectus muscle hematomas
when rectus contracted
Grey Turner's sign Local areas of discoloration around umbilicus and flanks Acute hemorrhagic pancreatitis
Iliopsoas sign Elevation and extension of leg against resistance creates pain Appendicitis with retrocecal abscess
Left shoulder pain when supine and pressure placed on left upper
Kehr's sign Hemoperitoneum (especially from splenic origin)
abdomen
Mannkopf's sign Increased pulse when painful abdomen palpated Absent if malingering
Pain caused by inspiration while applying pressure to right upper
Murphy's sign Acute cholecystitis
abdomen
Flexion and external rotation of right thigh while supine creates
Obturator sign Pelvic abscess or inflammatory mass in pelvis
hypogastric pain
Ransohoff sign Yellow discoloration of umbilical region Ruptured common bile duct
Rovsing's sign Pain at McBurney's point when compressing the left lower abdomen Acute appendicitis
Ten Horn sign Pain caused by gentle traction of right testicle Acute appendicitis

Correct Answer. c

Copyright 2014 Delhi Academy of Medical Sciences, All Rights Reserved. 73/121
(188). Surgery for ranula involves

a. Incision and drainage

b. Excision of cyst

c. Excision of affected sublingual salivary gland

d. Excision of cyst and affected sublingual salivary gland

Solution. Ans-188: (d)thExcision of cyst and affected sublingual salivary gland


Ref: Bailey and Love, 25 edn. Pg. 754
Sol :
A ranula is either a mucus retention cyst or a mucus extravasation pseudocyst arising from an obstructed sublingual gland.
A simple ranula is confined to the oral cavity as a cystic unilateral mass of the floor of the mouth.
A plunging ranula may pierce the mylohyoid and present as a paramedian or lateral neck mass with or without an obvious oral cavity ranula.
They should be differentiated from dermoid cysts and lymphangiomas.
Cyst aspiration consistently reveals fluid with high levels of protein and salivary amylase.
CT or MRI will demonstrate a uniloculated cystic mass arising from the sublingual space with extension into the submental, submandibular, or parapharyngeal
space.
Marsupialization has led to a high rate of recurrence of these lesions.
Surgical treatment includes excision in continuity with the sublingual gland of origin.

Correct Answer. d

(189). In chronic lymphocytic thyroiditis

a. Surgery is indicated in a majority of cases

b. Thyroid antibody estimation is useful in a few cases

c. FNAC is the most appropriate investigation

d. There is no role for steroid therapy in the management

Solution. Ans-189: (c) FNAC is the most appropriate investigation


Ref: Bailey and Love- 799-800.
Sol :
Chronic lymphocytic thyroiditis = Hashimotos thyroiditis
FNAC is the most appropriate investigation although abundant lymphocytes may make the cytological distinction between autoimmune thyroiditis and lymphoma
difficult.

Correct Answer. c

(190). In the adult patient with pleural effusion, the most appropriate site for pleurisentesis done by inserting a needle in:

a. 5th intercostals space in midclavicular line

b. 5th intercostals space in midaxillarys line

c. 2nd intercostals space adjacent to the sternum

d. 10th intercostals space adjacent to the vertebral column


th
Solution. Ans-190: (b) 5 intercostals space in midaxillarys line
Ref.: Bailey - 887
Sol :
The safest site for insertion of a drain is in the triangle which lies :
Anterior to mid axillary line
Above the level of nipple
Below and lateral to pectoralis major muscle
This will usually lie in fifth space

Correct Answer. b

Copyright 2014 Delhi Academy of Medical Sciences, All Rights Reserved. 74/121
(191). Measurements of intravascular pressure by a pulmonary artery catheter should be done :

a. At end expiration

b. At peak of inspiration

c. During mid inspiration

d. During mid expiration

Solution. Ans-191: (a) At end expiration


Ref.: CSDT 12/e p 357
Sol :
CVP and pulmonary artery pressure should be recorded at end expiration, because onset of inspiration causes a reduction in
intrathoracic pressure that is transmitted to both CVP and pulmonary artery pressure wave forms.
During spontanceous breathing, inspiration causes a decrease in pleural and juxtracardiac pressure that is transmitted in part to the
right atrium and lowers CVP.

Correct Answer. a

(192). The most common cause of acquired arterio venous fistula is :

a. Bacterial infection

b. Fungal infection

c. Blunt trauma

d. Penetrating trauma

Solution. Ans-192: (d) Penetrating trauma


Ref.: Read the text below
Sol :
Arteriovenous fistula
Abnormal communication between artery and vein either congenital or acquired
Most common cause of AV fistula is penetrating trauma
Through AV fistula high pressure arterial blood flows to vein.

Correct Answer. d

(193). Treatment of high grade mucoepidermoid tumour of salivary gland is?

a. Wide local excision

b. Wide local excision + Regional node dissection

c. Wide local excision + Regional node dissection + Radiotherapy.

d. Wide local excision + Regional node dissection + Radiotherapy + Chemotherapy

Solution. Ans-193: (c) Wide local excision + Regional node dissection + Radiotherapy.
Ref: Sabistom 18/e, p 1273
Sol :
For Low grade malignant tumours excision of the tumor with complete parotid (with clear margins) is the aim.
No adjuvant chemotherapy is required.
For high grade cancers after generous primary excision, for clinically palpable nodes radical or modified radical neck dissection is
performed followed by radiotherapy in selected group.

Correct Answer. c

Copyright 2014 Delhi Academy of Medical Sciences, All Rights Reserved. 75/121
(194). Most common malignant tumour of minor salivary gland is?

a. Mucoepidermoid carcinoma

b. Adenocystic carcinoma

c. Adenocarcinoma.

d. Malignant mixed tumour

Solution. Ans-194: (b) Adenocystic carcinoma.


Ref: Sabistom 18/e, p 1273
Sol :
Adenoid cystic carcinoma:
It is rare in parotid but commonest in minor salivary gland tumour, thus one fourth of malignant salivary gland tumours. 70% of all minor
salivary gland tumor occur in oral cavity, mainly in hard palate. Adenoid cystic carcinoma has propensity for perineural invasion

Correct Answer. b

(195). Treatment of Benign pleomorphic adenoma causing medial deviation of tonsil on oral cavity examination is?

a. Superficial Parotidectomy

b. Total Parotidectomy

c. Enucleation.

d. Total Conservative Parotidectomy

Solution. Ans-195: (d) Total Conservative Parotidectomy


Ref: Sabistom 18/e, p 1273
Sol :
Generally, benign tumors of the parotid gland are treated with superficial(Patey's operation) or total parotidectomy with the latter being
the more commonly practiced due to high incidence of recurrence.
The facial nerve should be preserved whenever possible. The benign tumors of the submandibular gland is treated by simple excision
with preservation of mandibular branch of the trigeminal nerve, the hypoglossal nerve, and the lingual nerve. Other benign tumors of
minor salivary glands are treated similarly.
Malignant salivary tumors usually require wide local resection of the primary tumor. However, if complete resection cannot be achieved,
adjuvant radiotherapy should be added to improve local control.

Correct Answer. d

Copyright 2014 Delhi Academy of Medical Sciences, All Rights Reserved. 76/121
(196). CABG is done for all of the following indications except :

a. To reduces symptoms

b. To prevent further catastrophes

c. To prolong life

d. To prevent progress of native blood vessel disease

Solution. Ans-196: (d) To prevent progress of native blood vessel disease


Ref.: Harrison 17/e, p 1525
Sol :
Revascularization procedure

Percutaneous coronary intervention Coronary artery bypass grafting


Feature

Anastomosis of one or both of the internal mammary artery


or a radial artery to the coronary artery distal to obstructive
Involves dilation of blocked artery by stent lesion is carried out.
For additional obstruction saphenous vein may be used

Advantage

Less invasive
Shorter hospital stay Effective in relieving symptoms
Lower initial cost Improved survival in certain subsets
Easily repeated Ability to achieve complete revascularization
Effective in relieving symptoms

Disadvantage

Rest enosis
High incidence of incomplete revascularization
Unknown effect on outcomes in patients with severe left Cost
ventricular dysfunction Increased risk of a repeat procedure due to late graft
Limited to specific anatomic subsets closure
Poor outcome in diabetics with 2- or 3-vessel coronary Morbidity and mortality of major surgery
disease

Ideal candidate for CABG is a male < 80 years of age who has no other complicating disease, has severe stenosis of two or three epicardial coromary artery with objective
evidence of MI as a cause of chest discomfort.
Occlusion rate in CABG is higher in saphenous vein graft than internal mammary or radial artery graft.

Correct Answer. d

Copyright 2014 Delhi Academy of Medical Sciences, All Rights Reserved. 77/121
(197). A patient undergoing surgery suddenly develops hypotension. The monitor shows that the end tidal CO2 has decreased abruptly by 155
m Hg. What is the probable diagnosis?

a. Hypothermia

b. Pulmonary embolism

c. Massive fluid deficit

d. Myocardial depression due to anesthetic agent

Solution. Ans-197: (b) Pulmonary embolism


Ref.: Millers Anesthesia 6/e 1461
Sol :
The concentration of CO2 in last 10 ml of tidal volume is called ETCO2
Why last 10 ml
Gas exhaled in initial phase of tidal volume represent dead space air
After this a mixture of dead space and alveolar air is exhaled
Finally alveolar air is exhaled.
Capnography provides information about CO2 production, pulmonary (lung) perfusion, alveolar ventilation, respiratory patterns, and
elimination of CO2 from the anaesthesia breathing circuit and ventilator. The shape of the curve is affected by some forms of lung
disease; in general there are obstructive conditions such as bronchitis, emphysema and asthma, in which the mixing of gases within the
lung is affected.
Conditions such as pulmonary embolism and congenital heart disease, which affect perfusion of the lung, do not, in themselves, affect the
shape of the curve, but greatly affect the relationship between expired CO2 and arterial blood CO2. Capnography can also be used to
measure carbon dioxide production, a measure of metabolism. Increased CO2 production is seen during fever and shivering. Reduced
production is seen during anaesthesia and hypothermia.

Correct Answer. b

(198). Most important prognostic factor for carcinoma esophagus is

a. Cellular differentiation

b. Depth of esophagus involvement

c. Length of esophagus involvement

d. Age of the patient

Solution. Ans 198: (b) Depth of esophagus involvement


Ref Read the text below
Sol:
Most important is depth of involvement of wall of esophgus and lymph node involvement of the surrounding esophageal tissue.
Length of esophagus involvement is not that important because esophagus has extensive submucosal lymph supply and for complete cure 10 cm excision margin would
mean removal of almost total esophagus.

Correct Answer. b

Copyright 2014 Delhi Academy of Medical Sciences, All Rights Reserved. 78/121
(199). A 52 yrs old alcoholic with known cirrhosis presents to emergency department with hematemesis after resuscitation and stabilization
which procedure should be the next step suspecting acute variceal bleed.

a. TIPS

b. Emergency porto- caval shunt

c. Splenectomy

d. Sclerotherapy.

Solution. Ans 199: (d) Sclerotherapy.


Ref Read the text below
Sol:
UGI endoscopy is the most rapid way of making the diagnosis. Once diagnosis is made sclerotherapy is the preferred method of managing acute variceal bleed. It success rate in
90%.
TIPS- Transjugular Intrahepatic porto systemic shunting is placing a metallic stent between the two veins with a success rate of 90% and lowering of porto hepatic
venous gradient to 12mm Hg
Indications
1. As a 2nd line of treatment in Acute variceal bleeding when medical management including drugs and Endoscopy fails
In such cases TIPS controls bleeding in 90-100% cases. It can also be used in acute gastric variceal bleed
2. Role in prevention of rebleed after primary management
The 1st line therapy is EVL (Endoscopic Variceal Ligation) and beta blockers. In these cases too rebleed can be as high as 20%
When compared with medical treatment in the primary prevention of rebleed TIPS markedly decreases the rebleed rate to around 19% as compared to 45% in medical group
although at a higher rate of encepahlopathy
3. Intractable ascites
Intractable ascites is one in which the ascites can not be mobilized with salt restriction and diuretics.Treatment options include repeated large-volume paracentesis,
peritoneovenous shunts, total portosystemic shunts, and liver transplantation
TIPS should not be the 1st choice therapy and used when repeated paracentesis is required or when ascites is associated with hydrothorax
4. Hepatorenal syndrome
5. Refractory Hydrothorax
6. Budd Chiari Syndrome and hepatic venous occlusive diseases
Contraindications
Pulmonary Hypertension and heptaopulmonary syndrome
Prehepatic portal hypertension

Correct Answer. d

(200). All are indication of Chenodeoxycholate treatment of gall stone except?

a. Radio-lucent stones

b. Functioning gall bladder

c. A 1.5 cm stone

d. Severe dyspeptic symptoms

Solution. Ans 200: (d) Severe dyspeptic symptoms.


Ref Read the text below
Sol:
Radiolucent stones, size < 2cm, Functioning gall bladder, minimal symptoms, single stone- medical management can be tried.

Correct Answer. d

Copyright 2014 Delhi Academy of Medical Sciences, All Rights Reserved. 79/121
(201). Bethesda HSIL includes?

a. CIN 1

b. CIN 2

c. CIN 1 and 2

d. CIN 2 and 3

Solution. Ans-201: (d) CIN 2 and 3


Ref.: Read the text below
Sol :
The Bethesda system(FIX) (TBS) is a system for reporting cervical or vaginal cytologic diagnoses,used for reporting Pap smear results. It
was introduced in 1988, and revised in 1991 and 2001.
High grade squamous intraepithelial lesion (HSIL) includes :
Moderate dysplasia CIN2
Severe dysplasia CIN3
Carcinoma in situ

Correct Answer. d

(202). Embryo transfer can be performed after various durations of embryo culture, conferring different stages in embryogenesis. Embryo
transfer is performed at ?

a. 1 day

b. 6 days

c. 10 days

d. 14 days

Solution. Ans-202: (b) 6 days


Ref.: Read the text below
Sol :
Embryo transfer can be performed after various durations of embryo culture, conferring different stages in embryogenesis. The main
stages at which embryo transfer is performed are cleavage stage (day 2 to 4 after co-incubation) or the blastocyst stage (day 5 or 6 after
co-incubation).
Embryos who reach the day 3 cell stage can be tested for chromosomal or specific genetic defects prior to possible transfer by
preimplantation genetic diagnosis (PGD). Transferring at the blastocyst stage confers a significant increase in live birth rate per transfer,
but also confers a decreased number of embryos available for transfer and embryo cryopreservation, so the cumulative clinical
pregnancy rates are increased with cleavage stage transfer.Transfer day 2 instead of day 3 after fertilization has no differences in live
birth rate.

Correct Answer. b

(203). Hyperplacentosis is seen in all except :

a. DM

b. Syphilis

c. Hypertension

d. Rh incompatibility

Solution. Ans-203: (c) Hypertension


Ref.: Read the text below
Sol :
Hyperplacentosis is a condition of heightened trophoblastic activity characterized by increased placental weight and circulating hCG
levels higher than those associated with normal pregnancy.
Hyperplacentosis is a risk factor for Gestational hypertension due to excessive exposure to chorionic vill.
Causes of hyperplacentosis :
Multiple pregnancy
Hydrops fetalis (Immune- Rh incompitability & Non immune syphilis, CMV, Parvovirus B19 etc.)
Diabetes mellitus
Large fetus

Correct Answer. c

Copyright 2014 Delhi Academy of Medical Sciences, All Rights Reserved. 80/121
(204). In poly cystic ovarian disease, which is not a feature?

a. Hirsutism

b. Polymenorrhea

c. Obesity

d. Infertility

Solution. Ans-204: (b) Polymenorrhea


Ref.: Read the text below
Sol :

Diagnostic Criteria (national Institutes of health and child health and human development

Major Minor

Chronic anovulation Insulin resistance


Hyperandrogenemia Perimenarchal onset of hirsutism and obesity
Clinical signs of hyperandrogenism Elevated LH-to-FSH ratio
Other etiologies excluded Intermittent anvulation associated with hyperandrogenemia (free testosterone, DHEAS)

Rotterdam criteria for PCOD at least 2 of the following should be present :

Oligo or anovulation
Clinical or biochemical signs of hyperandrogenism
Polycystic ovaries, with exclusion of other etiologies

Correct Answer. b

(205). Which is decreased in postmenopausal woman?

a. FSH

b. Estrogen

c. Both

d. None

Solution. Ans-205: (b) Estrogen


Ref.: Read the text below
Sol :
Estrogen levels are decreased leading to increased FSH levels in postmenopausal women.

Correct Answer. b

Copyright 2014 Delhi Academy of Medical Sciences, All Rights Reserved. 81/121
(206). Placenta develops from?

a. Chorion frondosum and deciduas basalis

b. Chorion frondosum and deciduas capsularis

c. Chorion

d. Amnion

Solution. Ans-206: (a) Chorion frondosum and deciduas basalis


Ref.: Read the text below
Sol :
The placenta functions as a fetomaternal organ with two components: the fetal placenta (Chorion frondosum), which develops from the
same blastocyst that forms the fetus, and the maternal placenta (Decidua basalis), which develops from the maternal uterine tissue
In humans, the placenta averages 22 cm (9 inch) in length and 22.5 cm (0.81 inch) in thickness, with the center being the thickest, and
the edges being the thinnest. It typically weighs approximately 500 grams (just over 1 lb).
It has a dark reddish-blue or crimson color. It connects to the fetus by an umbilical cord of approximately 5560 cm (2224 inch) in
length, which contains two umbilical arteries and one umbilical vein

Correct Answer. a

(207). Deep transverse arrest is found in which type of pelvis?

a. Android

b. Gynaecoid

c. Platypelloid

d. Anthropoid

Solution. Ans-207: (a) Android Ref.: Read the text below Sol : In Deep transverse arrest most common type of pelvis found is Android
pelvis as it is the commonest type of abnormal pelvis found. It is found in 1/3rd of women and platypelloid is found in only 3% If we have
100 women with DTA, maximum will have android component and few will have platypelloid, as android is much more common.

Correct Answer. a

(208). Commonest type of ectopic pregnancy with rupture is?

a. Isthmic

b. Ampulla

c. Interstitial

d. Infundibular

Solution. Ans-208: (a) Isthmic Ref.: Read the text below Sol : There is a significant increase in the number of ectopic pregnancies in the
past 20 years. Factors that increase the risk for tubal pregnancy:
a. Previous laparoscopically proven PID
b. Previous tubal pregnancy
c. Current IUD use
d. Previous tubal surgery for infertility
e. Prior Surgery.
f. Previous Pregnancies
g. Pelvic infection
The isthmus is the narrowest portion of the tube, least distensible, and tend to rupture early and violently, with massive abdominal
hemorrhage

Correct Answer. a

Copyright 2014 Delhi Academy of Medical Sciences, All Rights Reserved. 82/121
(209). Most common fungal infection in 3rd trimester of pregnancy is?

a. Epidermophyton

b. Aspergillus

c. Candida albicans

d. Tinea

Solution. Ans-209: (c) Candida albicans Ref.: Read the text below Sol :
There is an increased vaginal discharge in pregnancy-only personal hygiene required.
Candida albican growth favoured by high pH and presence of sugar in urine.
Treated by miconazole cream with one applicator full, high up in vagina, for 7 nights.

Correct Answer. c

(210). Tip test is done for?

a. Stress incontinence

b. Urge incontinence

c. Vesico vaginal fistula

d. Utero vesical fistula

Solution. Ans-210: (a) Stress incontinence Ref.: Read the text below Sol : Q-tipped cotton swab test :
A Q-tipped cotton swab dipped in Xylocaine jelly is placed in the urethra.
The patient is asked to strain & cough. Initially, the cotton swab-stick will be parallel to the floor.
In pts with no genuine stress incontinence (GSI), the cotton swab stick will normally reach an angle not exceading 10-50 above
horizontal.
This angle increases by 20 or more, commonly 50-70 in positive cases.
A positive test indicates sufficient degree of bladder neck decent.
All pts with GSI may not have a positive test.
This test, if positive, obviates the need for a head chain cystourethrogram

Correct Answer. a

(211). Bonneys test is used to demonstrate?

a. Stress incontinence

b. Urge incontinence

c. Fibroids

d. True incontinence

Solution. Ans-211: (a) Stress incontinence


Ref.: Read the text below
Sol :
COUGH STRESS TEST (BONEYS TEST)
When stress urinary incontinence is suspected, this test (Bonney 1923) can be conducted.
This is performed by placing the index and middle finger on each side of the urethra at the level of the bladder neck and supporting it,
with care taken not to cause compression of the urethra against the pubic bone.
The recumbent patient is requested to cough with bladder neck pushed up.
If urine leakage is controlled with the lifting motion, it is positive result and suggest stress incontinence due to bladder neck descent,
urethral hypermobility and laxity of paraurethral support.

Correct Answer. a

Copyright 2014 Delhi Academy of Medical Sciences, All Rights Reserved. 83/121
(212). A patient present with Ca endometrium, more than 50% myometrium and vagina involved, pelvic and para aortic nodes negative and
positive peritoneal cytology. Stage the disease :

a. II B

b. III B

c. III C1

d. III C2

Solution. Ans-212: (b) III B


Ref.: Read the text below
Sol :

FIGO Staging of CA Endometrium

Stage 1 Tumor confined to corpus uterus

1A No or less than myometrial invasion

1A Invasion equal to more than of myometrium

Stage II* Tumor invades cervical stroma, but does nto extend beyond the uterus**

Stage III* Local and/or regional spread of the tumor

III A* Tumor invades the serosa of corpus uteri and/or adenexa#

III B* Vaginal and/or parametrial involvement#

III C* Metastasis to pelvic and/or para-aortic involvement#

III C 1* Positive pelvic nodes

III C 2* Positive para-aortic lymph nodes with or without positive pelvic lymph nodes

Stage IV * Tumor invades bladder and/or bowel mucosa, and/or distant metastasis

IV A* Tumor invasion of bladder and/or bowel mucosa

IV B* Distant metastasis, including intra- abdominal metastasis and/or inguinal lymph nodes

Correct Answer. b

(213). The corpus luteum of menstruation persists for?

a. 5 days

b. 10 days

c. 22 days

d. 30 days

Solution. Ans-213: (b) 10 days


Ref.: Read the text below
Sol :
Corpus luteum controls the ovarian cycle
Leuteotrophic support maintains the corpus luteum.
Luteolysis terminates its life of 10-18 days and induces a new cycle.

Correct Answer. b

Copyright 2014 Delhi Academy of Medical Sciences, All Rights Reserved. 84/121
(214). Artificial rupture of membrane is contraindicated in?

a. Heart disease

b. Hydraminos

c. Pregnancy induced hypertension

d. Intrauterine death

Solution. Ans-214: (b) Hydraminos


Ref.: Read the text below
Sol :
Artificial rupture of membranes may lead to sudden decompression and result in APH (Antepartum hemorrhage) in hydramnios.

Correct Answer. b

(215). All are used in treating dysmenorrheal, except:

a. Bromocriptine

b. Ibuprofen

c. Mefenamic caid

d. Norethisterone and ethanyl estradiol

Solution. Ans-215: (a) Bromocriptine


Ref.: Read the text below
Sol :
Drugs used in dysmenorrhoea are
Prostaglandin synthetase inhibitors-mefenamic acid, ibuprofen, indomethacin.
OCPs
Dydrogesterone

Correct Answer. a

(216). All are Emergency post coital contraceptives, except :

a. IUD

b. Danazol

c. Mifeprestone

d. None

Solution. ans-216: (b) Danazol


Ref.: Read the text below
Sol :
Post coital/emergency contraceptives are
1. Levonorgesterol
2. Ethinly estradiol+norgesterol
3. Conjugated estrogen
4. Ethinyl estradiol
5. Mifepristone
6. Copper IUCDs

Correct Answer. b

Copyright 2014 Delhi Academy of Medical Sciences, All Rights Reserved. 85/121
(217). The most consistent sign in disturbed ectopic pregnancy is?

a. Pain

b. Vaginal bleeding

c. Fainting

d. Vomiting

Solution. Ans-217: (a) Pain


Ref.: Read the text below
Sol :
Classic triad of symptoms in ruptured ectopic pregnancy is abdominal pain (100%),Preceding amenorrhoea (75%), Vaginal bleeding (70%)

Correct Answer. a

(218). Post menopausal bleeding is vaginal bleeding occurring?

a. After 6 months of amenorrhea in menopausal age

b. After 3 months of amenorrhea in menopausal age

c. After 1 year of amenorrhea in menopausal age

d. After 1 and years of amenorrhea in menopausal age

Solution. Ans-218: (c) After 1 year of amenorrhea in menopausal age


Ref.: Read the text below
Sol :
Post menopausal bleeding is vaginal bleeding occurring after 1 yr of onset of amenorrhoea in menopausal women.
It should be promptly evaluated by endometrial sampling.

Correct Answer. c

(219). Ventouse is contraindicated in all, except:

a. Foetal distress

b. Face presentation

c. Transverse lie

d. Deep Transverse arrest

Solution. Ans-219: (a) Foetal distress


Ref.: Read the text below
Sol :
Contraindications of ventouse application :
1. Non vertex presentation
2. Preterm fetus
3. Suspected aponuerotic haemorrhage
4. Fetal macrosomia

Correct Answer. a

Copyright 2014 Delhi Academy of Medical Sciences, All Rights Reserved. 86/121
(220). Aschheim-Zondek tests is false positive in the following, except:

a. Cancer cervix

b. Hydatidiform mole

c. Ectopic pregnancy

d. Chorionic epithelioma

Solution. Ans-220: (a) Cancer cervix


Ref.: Read the text below
Sol :
Aschheim-Zondek test is an obsolete test for increased hCG levels as in normal pregnancy, ectopic pregnancy, Hydatidiform mole etc.
The tests are usually based on changes in the ovaries of an animal injected with the urine of a pregnant woman.
Included are the Aschheim Zondek test (using mice or rats) and the Friedman test (using virigin rabbits).
Both the Friedman and the Aschheim-Zondek tests have an accuracy of about 98% but the former has supplanted the latter because of the speed of completion of the test.
Whereas the Aschheim Zondek test requires 96 hours, the Friedman can be completed in 48 hours.

Correct Answer. a

(221). Commonest cause of pyometra is?

a. Carcinoma endometrium

b. Carcinoma cervix

c. Carcinoma vagina

d. Uterine myoma

Solution. Ans-221: (a) Carcinoma endometrium


Ref.: Read the text below
Sol :
Causes of pyometra are :
1. Ca endometrium
2. Endocervical ca
3. Senile endometritis
4. Infected hematometra
5. Tubercular Endometritis

Correct Answer. a

(222). Surgery for mitral stenosis during pregnancy is ideally done at?

a. 8 weeks

b. 14 weeks

c. 28 weeks

d. 32 weeks

Solution. Ans-222: (b) 14 weeks


Ref.: Read the text below
Sol :
nd
Cardiac surgery should be postponed until after delivery but if required can be performed safely in 2 trimester.

Correct Answer. b

Copyright 2014 Delhi Academy of Medical Sciences, All Rights Reserved. 87/121
(223). What is the optimal time of the menstrual cycle when serum progesterone should be drawn to confirm the diagnosis phase deficiency?

a. Day 15

b. Day 18

c. Day 21

d. Day 25

Solution. Ans-223 : (c) Day 21


Ref.: Read the text below
Sol :
Luteal phase defect accounts for 4% cases of infertility.
There is failure to develop fully mature secretory endometrium during implantation window
It is diagnosed when there is a delay of 2 days or more in endometrial histology when compared to chronologic cycle day in two or more cycles.
Diagnosed by serum progesterone level and Endometrial biopsy.
Managed by progesterone by various routes.
No route of administration is superior.

Correct Answer. c

(224). Spinal anesthesia for cesarean section involves the injection of sufficient local anesthetic to achieve a:

a. T2 sensory level

b. T4 sensory level

c. L4 sensory level

d. L2 sensory level

Solution. Ans 224: (b) T4 sensory level


Reference Read the text below
Sol:
Spinal anesthesia
Spinal anesthesia for cesarean section involves the injection of sufficient local anesthetic to achieve a T4 sensory level.
Spinal anesthesia is chosen when the length of the procedure is known. If the duration is in question, a continuous technique is
preferred.
The major risks of spinal anesthesia include postdural puncture headache, bradycardia, hypotension, and limited duration of anesthesia.
Compared to epidural anesthesia, umbilical cord pH is lower for spinal anesthesia than for epidural anesthesia.
Hypotension from spinal anesthesia may be treated with either phenylephrine 100 g or ephedrine 10 mg.

Correct Answer. b

(225). Pudendal bock is usually administered during the :

a. First stage of labor

b. Second stage of labor

c. Third stage of labor

d. None.

Solution. Ans 225: (b) Second stage of labor


Reference Read the text below
Sol: Pudendal bock
Somatic pain of the second stage of labor results from distention of the pelvic floor, vagina, and perineum as the fetus descends through the birth canal.
These painful impulses are transmitted primarily through the pudendal nerve, which is derived from the anterior primary divisions of the sacral nerves S2 to S4.
Pudendal block involves the injection of local anesthetic below the ischial spines (the approximate location of the pudendal nerve). This block is administered during
the second stage of labor and is useful for vaginal delivery and outlet forceps. It is also helpful for suturing following delivery as it anesthetizes the perineum.

Correct Answer. b

Copyright 2014 Delhi Academy of Medical Sciences, All Rights Reserved. 88/121
(226). An increased evidence of adenocarcinoma of the small intestine has been established with which of the following conditions?

a. Peutz-Jegher Syndrome

b. Crohns disease

c. Simple tubular adenomas of the small intestine

d. Colon carcinoma

Solution. Ans 226: (b) Crohns disease


Reference: Read the text below
Sol:
The incidence of adenocarcinoma of the small bowel is surprisingly low when compared to that of colon carcinoma when considering the
vast length and surface area of the small intestine.
The relative infrequency of these tumors has limited our knowledge of pathogenic factors. It appears, however, that the polyp-to-cancer
sequence is not well established for simple tubular adenomas of the small intestine.
Peutz-Jegher Syndrome is an inherited syndrome which is associated with multiple small intestinal polyps. These polyps are hamartomas
with progression of dysplasia to carcinoma felt not to be a major concern.
The chronic inflammatory changes of Crohns disease appears to predispose to the development of adenocarcinoma, thereby increasing
the risk to 100 times that of the general population.
There is no known association of carcinoma of the colon and small intestine.

Correct Answer. b

(227). Which of the following features is consistent with a diagnosis of colonic inertia?

a. Alternating episodes of severe constipation and normal bowel activity

b. Total bowel transit time of 24 hours

c. Total bowel transit time of 48 hours

d. Total bowel transit time of 96 hours

Solution. Ans 227: (d) Total bowel transit time of 96 hours


Reference: Read the text below
Sol:
Colonic inertia, or slow transit constipation, is primarily a disease of young women.
Despite attempts at initiating bowel movements with fiber supplementation, large doses of laxatives and enemas, normal bowel
movement patterns are not established. The etiology of this condition is totally unknown, but a likely etiology is some aberration in the
neurochemical control of the colon, possibly within the enteric nerves.
Abnormalities within the neural elements of the myenteric plexus suggest that disturbances in neuromodulation of colonic motility may
play a role in some patients. The treatment of colonic inertia has proved difficult and many patients have required subtotal colectomy to
correct the severe constipation.
Diagnosis of this condition is usually achieved by assessing colonic transit with various radio-opaque markers. After ingestion of such
markers, sequential abdominal films are taken to assess movement of markers in each segment of the colon.
Total transit time in normal subjects averages about 35 hours. Total transit time in excess of 72 hours is clearly abnormal.

Correct Answer. d

Copyright 2014 Delhi Academy of Medical Sciences, All Rights Reserved. 89/121
(228). Which of the following statements regarding the myenteric plexus of the colon is incorrect?

a. The myenteric plexus is located between the longitudinal and circular layers of the bowel wall

b. The myenteric plexus contains only sensory neurons

c. The density of neurons with the colonic plexuses decreases along the length of the bowel

d. Neurons of the myenteric plexus control the motor function of the colon

Solution. Ans 228: (b) The myenteric plexus contains only sensory neurons
Reference: Read the text below
Sol:
Similar to other regions of the gut, two groups of plexuses exist within the wall of the colon. The submucosal plexus is located between
the muscularis mucosa and the circular muscle layer of the muscularis propria.
Sandwiched between the circular muscle and the outer longitudinal muscle is the myenteric plexus. The myenteric plexus appears to be
intimately involved with colonic motility. The plexus is composed of ganglia and clusters of nerve cell bodies that are linked together by
bundles of nerve processes.
Although the myenteric plexus is well developed throughout the entire length of the colon, the density of nerve cells is greatest in the
proximal one third of the colon, similar to the density that exists throughout the small intestine.
The physiologic significance of this organization remains to be defined, but probably contributes to the different motility patters that
exist throughout the large bowel.

Correct Answer. b

(229). Which of the following ocular manifestations of ulcerative colitis does not respond to therapy with steroids or immunosuppressive
agents?

a. Iridis

b. Uveitis

c. Retrobulbar neuritis

d. Ulcerative panophthalmitis

Solution. Ans 229: (d) Ulcerative panophthalmitis


Reference: Read the text below
Sol:
A number of ocular manifestations of ulcerative colitis exist. Included in this group are iridis, uveitis, conjunctivitis, episcleritis, retinitis,
and retrobulbar neuritis.
With the exception of ulcerative panophthalmitis, ocular symptoms are closely related to disease activity and respond to therapy with
steroids or immunosuppressive agents.

Correct Answer. d

(230). Which of the following statements regarding the risk of cancer in the context of ulcerative colitis is correct?

a. After 10 years of active disease, the risk of cancer approximates 20% to 30%

b. After 10 years of active disease, the risk of cancer approximates 2% to 3%

c. The risk of colon cancer in ulcerative colitis is identical to controls

d. After 20 years of disease activity, the risk of colon cancer approximates 80%

Solution. Ans 230: (b) After 10 years of active disease, the risk of cancer approximates 2% to 3%
Reference: Read the text below
Sol:
Significant dysplasia or suspected colon cancer is a clear indication for colectomy in patients with ulcerative colitis.
Earlier studies have suggested that the risk of cancer is relatively low for the first ten years after the onset of disease activity (roughly
2% to 3%).
The incidence of colon cancer then begins to climb at a rate of 1% to 2% per year. By the time the patient has had ulcerative colitis for 20
years, the risk of colon cancer approximates 20%.
Many epidemiologists believe that earlier studies overestimated the risk of malignancy due to referral bias and the imperfection of
retrospective surveys performed in tertiary referral hospitals.

Correct Answer. b

Copyright 2014 Delhi Academy of Medical Sciences, All Rights Reserved. 90/121
(231). Identify the pedigree drawn below

a. Autosomal Recessive

b. Autosomal Dominant

c. X linked recessive

d. X linked dominant

Solution. Ans-231: (c) X linked recessive


Ref.: Read the text below
Sol :
Hemophilia, Duchenne muscular dystrophy, Becker muscular dystrophy, and Lesch-Nyhan syndrome are relatively rare in most
populations, but because of advances in molecular genetics they receive attention in the media. More common traits, such as glucose--
-phosphate dehydrogenase deficience or color blindness, may occur frequently enough in some populations to produce a few affected
females. However, their effect on individuals is rarely life threatening and medical intervention is not needed. Pedigree shows one typical
inheritance pattern for a rare X-linked recessive disease.

Correct Answer. c

Copyright 2014 Delhi Academy of Medical Sciences, All Rights Reserved. 91/121
(232). Noonan syndrome may have all except

a. Hypertrophic cardiomyopathy

b. Cryptorchidism

c. Infertility in females

d. AD transmission

Solution. Ans-232: (c) Infertility in females


Ref.: Read the text below
Sol :
The term Noonan syndrome has been applied to males and females with normal karyotypes who have certain phenotypic features that
occur also in females with Turner syndrome
Noonan syndrome occurs in 1 : 10002500 live births. The disorder is autosomal dominant with variable expression. Sporadic and
autosomal recessive occurrence has been reported.
Missense mutations in PTPN11The cardiac defect is most often pulmonary valvular stenosis, hypertrophic cardiomyopathy, or atrial
septal defect.Males frequently have cryptorchidism and small testes; they may be hypogonadal or normal.
Puberty is delayed 2 yr; adult height is achieved by the end of the 2nd decade and usually reaches the lowest limit of the normal
population.

Correct Answer. c

Copyright 2014 Delhi Academy of Medical Sciences, All Rights Reserved. 92/121
(233). Bidextrous grasp is achieved at

a. 3 months

b. 4 months

c. 5 months

d. 6 months

Solution. Ans-233: (c) 5 months


Ref.: Read the text below
Sol :
DEVELOPMENT MILESTONES (QUICK REVIEW):
Gross Motor:
3 months Head holding
5 months Sitting with support
6 months Prone to supine, supine to prone
8 months Sitting without support
9 months Crawling
10 months Creeping
1 year Walking few steps independently
2 yr walk up stairs with two feet at each step
3 yr Up stairs with one foot at each step, rides tricycle
4 yrs Hops on one foot
5 yr Skips on two feet
Fine motor
12 weeks Grasp reflex disappears
4 months goes for object
5 months Bidextrous grasp
7 months Transfer object, palmar grasp
9 months Pincer grasp
13 months Casting
15 months Self feed with a spoon
18 months Self feed with a cup
tower of 4 cubes
24 months turns pages of a book one at time
6-7 cubes
3 yrs makes circle, tower of 10 cubes
4 yrs Square, + sign
5 yrs Triangle x sign
Social
2 months Social smile
3 months recognizes mother
6-7 months smiles at mirror image
9 months waves bye-bye
Stranger anxiety
15 months hugs parents
18 months kisses parents
2 yrs Dry by day
3 yrs dry by night
Dress/undress himself
Language
1 month head turns to sound
3 months cooing
6 months mono syllabus (ma,ba)
9 months Bisyllabus (ma-ma, baba)
1 yr 2 words with meaning
18 months vocabulary of ten words
2 yrs simple sentences with 2 words
3 yrs tells age and sex, uses pronouns,handedness
4 yrs tells story
5 yrs knows colors

Correct Answer. c

Copyright 2014 Delhi Academy of Medical Sciences, All Rights Reserved. 93/121
(234). CFTR gene is on

a. 7p

b. 7q

c. 17p

d. 17q

Solution. Ans-234: (b) 7q


Ref.: Read the text below
Sol :
CF is the major cause of severe chronic lung disease in children and is responsible for most exocrine pancreatic insufficiency in early life.
It is also responsible for many cases of salt depletion, nasal polyposis, pansinusitis, rectal prolapse, pancreatitis, cholelithiasis, and
insulin-dependent hyperglycemia.
CF occurs most frequently in white populations of northern Europe, North America, and Australia/New Zealand. The prevalence varies by
report but, in general, approximates 1/3,500 live births.
CF is inherited as an autosomal recessive trait.
The CF gene codes for a protein of 1,480 amino acids called the CF transmembrane regulator (CFTR). CFTR is expressed largely in
epithelial cells of airways, the gastrointestinal tract (including the pancreas and biliary system), the sweat glands, and the genitourinary
system.
CFTR has ion channel and regulatory functions that are perturbed variably by the different mutations.
More than 1,500 CFTR polymorphisms are associated with the CF syndrome.
The most prevalent mutation of CFTR is the deletion of a single phenylalanine residue at amino acid 508 (F508). This mutation is
responsible for the high incidence of CF in northern European populations and is considerably less frequent in other populations, such as
those of southern Europe and Israel.
Approximately 50% of individuals with CF who are of northern European ancestry are homozygous for F508, and >70% carry at least
one F508 gene.
The membranes of CF epithelial cells are unable to secrete chloride ions in response to cyclic adenosine monophosphate
(cAMP)mediated signals and, at least in the respiratory tract, excessive amounts of sodium are absorbed through these membranes
Another puzzle is the propensity for P. aeruginosa to undergo mucoid transformation in the CF airways. The complex polysaccharide
produced by these organisms generates a biofilm that provides a hypoxic environment and thereby protects Pseudomonas against
antimicrobial agents.

Correct Answer. b

(235). What is the preferred drug for Infantile spasms in children

a. ACTH

b. Phenytoin

c. Valproate

d. Vigabatrin

Solution. Ans-235: (a) ACTH


Ref.: Read the text below
Sol :
Infantile spasms usually begin between the ages of 4 and 8 mo and are characterized by brief symmetric contractions of the neck, trunk,
and extremities. There are at least three types of infantile spasms: flexor, extensor, and mixed.
The EEG that is most commonly associated with infantile spasms is referred to as hypsarrhythmia, which consists of a chaotic pattern of
high-voltage, bilaterally asynchronous, slow-wave activity or a modified hypsarrhythmia pattern.
Infantile spasms are typically classified into two groups:cryptogenic and symptomatic.
Prenatal and perinatal factors include hypoxic-ischemic encephalopathy with periventricular leukomalacia, congenital infections, inborn
errors of metabolism, neurocutaneous syndromes such as tuberous sclerosis, cytoarchitectural abnormalities including lissencephaly and
schizencephaly, and prematurity. Postnatal conditions include CNS infections, head trauma (especially subdural hematoma and
intraventricular hemorrhage), and hypoxic-ischemic encephalopathy.
Infants with cryptogenic infantile spasms have a good prognosis, whereas those with the symptomatic type have an 8090% risk of
mental retardation.
One hypothesis implicates corticotropin-releasing hormone (CRH), a putative neurotransmitter, metabolized in the inferior olive.
Exogenous ACTH and glucocorticoids suppress CRH synthesis, which may account for their effectiveness in treating infantile spasms. In
Europe and Canada, infantile spasms associated with TS are treated with vigabatrin (rather than adrenocorticotropic hormone) with
good results. Vigabatrin is not available in the United States.

Correct Answer. a

Copyright 2014 Delhi Academy of Medical Sciences, All Rights Reserved. 94/121
(236). All are true about Tetrology of Fallot except :

a. Right sided aortic arch seen in 85% cases

b. Associated with CATCH-22

c. Persistence of a left superior vena cava is seen

d. Ventricular septal defect (VSD)

Solution. Ans-236: (a) Right sided aortic arch seen in 85% cases
Ref.: Read the text below
Sol :
Tetrology of Fallot
Primary defect is an anterior deviation of the infundibular septum (the muscular septurm that separates the aortic and pulmonary
outflows).
The consequences of this deviation are :
Obstruction to right ventricular outflow (pulmonary stenosis)
Ventricular septal defect (VSD)
Dextroposition of the aorta with override of the ventricular septum
Right ventricular hypertrophy

Correct Answer. a

(237). The commonest cause of Aseptic Meningitis in children is?

a. Enterovirus

b. Measles

c. Mumps

d. AIDS

Solution. Ans-237: (a) Enterovirus


Ref.: Read the text below
Sol :
Enteroviruses are the most common cause of aseptic meningitis in chidren and adults
Mumps is the most common cause of aseptic meningitis in an unimmunized population
The most common vector transmitted cause of aseptic meningitis is st. Louis encephalitis virus, a flavivirus of Arbrovirus group.

Correct Answer. a

Copyright 2014 Delhi Academy of Medical Sciences, All Rights Reserved. 95/121
(238). Most common cause of upper GI bleed in children is?

a. NSAIDs

b. Varices

c. Acute gastritis

d. Ingested blood

Solution. Ans-238: (c) Acute gastritis


Ref.: Read the text below
Sol :
Most common causes of upper GI bleeding in children are due to mucosal lesions like acute gastritis followed by peptic ulcer disease.
Most common cause of dramatic or serious upper GI bleed in children is Esophageal Varices.
Age Wise Distribution of Upper GI bleed in children in order of Frequency

Swallowed blood > Gastritis > Peptic Ulcer > Idiopathic > Bleeding Diasthesis > Esophagitis > Intestinal Duplication >
Neonates
Vascular malformations

Infancy Gastritis > Esophagitis > Mallory Weiss Lesion > Pepitc Ulcer > Pyloric Stenosis > Vascular Malformation

Pre School Epistaxis > Gastritis > Esophagitis > Mallory Weis Tear > Toxin Ingestion > Peptic Ulcer

School Gastritis > Peptic Ulcer > Mallory Weiss > Stress Ulcers > Toxin Ingestion

Correct Answer. c

(239). Most common age group for bronchiolitis is?

<2 years>a.

b. 2-5 years

c. 5-10 years

d. > 10 years

Solution. Ans-239: (a) <2 years


Ref.: Read the text below
Sol :
Acute bronchiolitis is usually preceded by exposure to an older contact with a minor respiratory syndrome within the previous week.
The infant 1st develops a mild upper respiratory tract infection with sneezing and clear rhinorrhea.
This may be accompanied by diminished appetite and fever of 38.5-39C (101-102 F), although the temperature may range from
subnormal to markedly elevated.
Gradually, respiratory distress ensues, with paroxysmal wheezy cough, dyspnea, and irritability.
The infant is often tachypnoeic, which may interfere with feeding.
The child does not usually have other systemic complaints, such as diarrhea or vomiting.
Apnea may be more prominent than wheezing early in the course of the disease, particularly with very young infants (<2 mo old) or
former premature infants.)

Correct Answer. a

Copyright 2014 Delhi Academy of Medical Sciences, All Rights Reserved. 96/121
(240). The percentage rise in length of infant in first year of life is?

a. 20%

b. 30%

c. 40%

d. 50%

Solution. Ans-240: (d) 50%


Ref.: Read the text below
Sol :
By the 1st birthday, birth weight has tripled, length has increased by 50% and head circumference has increased by 10 cm.

Correct Answer. d

(241). A 15 months old child do all, except:

a. Feeds self with spoon

b. Says three words

c. Build tower of 2 blocks

d. Creeps upstairs

Solution. Ans-241: (a) Feeds self with spoon


Ref.: Read the text below
Sol :
Child feeds himself with spoon at 18 months of age, rest of the given options are attained at or before 15 months of age.

Correct Answer. a

(242). APGAR score of a child which indicates high mortality is?

a. 0-3

b. 2-7

c. 4-7

d. 5-10

Solution. Ans-242: (a) 0-3


Ref.: Read the text below
Sol :
An infant with a score of 0-3 requires immediate resuscitation.
The 1 min APGAR score may signal the need for immediate resuscitation and the 5-10-, 15-, and 20-min scores may indicate the
probability of successfully resuscitating an infant.
A low score may be due to a number of factors, including drug given to the mother during labor and immaturity.
The APGAR score and umbilical artery blood pH both predict neonatal death.
An APGAR score of 0-3 at 5 min is uncommon but is a better predictor of neonatal death (in both term and preterm infants) than an
umbilical artery pH of 7.0 or less; the presence of both variables increases the relative risk of neonatal mortality in term and preterm
infants.

Correct Answer. a

Copyright 2014 Delhi Academy of Medical Sciences, All Rights Reserved. 97/121
(243). Congestive heart failure in children is best diagnosed by?

a. Tachycardia and tender hepatomegaly

b. Increased JVP

c. Increased JVP + pedal edema

d. Hypotension

Solution. Ans-243: (c) Increased JVP + pedal edema


Ref.: Read the text below
Sol :
In children, the signs and symptoms of heart failure may be similar to those in adults and include fatigue, effort intolerance, anorexia,
abdominal pain, dyspnea, and cough.
Many children, however, especially adolescents, may have primarily abdominal symptoms and a surprising lack of respiratory
complaints.
Attention to the cardiovascular system may come only after an abdominal roentgenogram unexpectedly shows the lower end of an
enlarged heart.
The elevation in systemic venous pressure may be gauged by clinical assessment of jugular venous pressure and liver enlargement.
Orthopnea and basilar rales are variably present; edema is usually discernible in dependent potions of the body, or anasarca may be
present.
Cardiomegaly is invariably noted.
A gallop rhythm is common; when ventricular dilatation is advanced, the holosystolic murmur of mitral or tricuspid valve regurgitation
may be heard.

Correct Answer. c

(244). Which of the following is cause of pan systolic murmur?

a. MS, ASD

b. MS, TA

c. VSD, MR

d. ASD, TR

Solution. Ans-244: (c) VSD, MR


Ref.: Read the text below
Sol :
Holosystolic (pansystolic) murmurs start at S1 and extends up to S2. They are usually due to regurgitation in cases such as mitral
regurgitation, tricuspid regurgitation, or ventricular septal defect (VSD).[

Correct Answer. c

(245). Ossification in fetus starts at?

a. 3 weeks

b. 5 weeks

c. 9 weeks

d. 12 weeks

Solution. Ans-245: (c) 9 weeks


Ref.: Read the text below
Sol :
Ossification begins in fetus at around 9 weeks.

Correct Answer. c

Copyright 2014 Delhi Academy of Medical Sciences, All Rights Reserved. 98/121
(246). Proper use of the PRISM scoring system would include:

a. Decision-making in end-of-life issues for a chronically ill child

b. Withdrawal of support decisions for a child with multiple organ failure

c. Comparison of level of disease severity between treatment and control groups

d. Assessment of performance of a chemotherapy regimen

Solution. Ans 246: (c) Comparison of level of disease severity between treatment and control groups
Ref Read the text below
Sol:
The Pediatric Risk of Mortality score is based on 17 physiologic variables (vital and neurologic signs, acid-base,blood chemistries,
hematologic parameters) subdivided into 26 ranges and taking into consideration age (neonate, infant, child,adolescent).
It is best in predicting mortality for populations of patients and not for an individual PICU patient.
Decisionmaking at the end of life should never be based on an acute PRISM score, especially in a chronically ill child.
It has no relevance or reliability in non-PICU patients, such as those receiving chemotherapy.

Correct Answer. c

(247). A 6-month-old comatose infant with multiple broken bones in various stages of healing, bulging anterior fontanelle, and retinal
hemorrhages.Diagnosis is

a. Caput succedaneum

b. Subdural hemorrhage

c. Subarachnoid hemorrhage

d. Cephalohematoma

Solution. Ans 247: (2) Subdural hemorrhage.


Ref: Read the text below
Sol:
Subdural hematomas are commonly seen as part of the shaken baby syndrome.
This lesion occurs when the bridging cortical veins that drain the cerebral cortex have been ruptured, leading to a collection of blood
between the dura and the cerebral mantle.
Repeated trauma can lead to additional collections of blood. In many children, additional findings of abuse such as broken bones, bruises,
and retinal hemorrhages are found.

Correct Answer. b

(248). Which of the following features distinguishes paralytic polio from Guillain-Barr syndrome?

a. Pleocytosis is uncommon in paralytic polio

b. Paralysis is usually asymmetric in paralytic polio

c. The paralysis of polio is usually spastic

d. Sensory changes are common in paralytic polio

Solution. Ans 248: (b) Paralysis is usually asymmetric in paralytic polio


Ref Read the text below
Sol:
Paralytic polio is characterized by aseptic meningitis accompanied by asymmetric flaccid paralysis without sensory loss.
In Guillain-Barr syndrome, the paralysis is characteristically symmetric,and sensory changes (paresthesias) are common.
Pleocytosis is common in polio, whereas the cerebrospinal fluid in Guillain-Barr syndrome usually shows only elevated protein and
occasionally a few cells.

Correct Answer. b

Copyright 2014 Delhi Academy of Medical Sciences, All Rights Reserved. 99/121
(249). Most infants with localized neonatal herpes encephalitis become symptomatic at the age of:

a. 0-2 days

b. 2-8 days

c. 8-12 days

d. 13-21 days

Solution. Ans 249: (c) 8-12 days


Ref Read the text below
Sol:
Localized skin, eye, and mouth infection and also disseminated infection occur at a mean of 5-6 days post partum,whereas localized CNS
infection occurs later at a mean of 8-12 days post partum.

Correct Answer. c

(250). A 2-yr-old patient, previously admitted to the hospital for respiratory distress, is found apneic and pulseless in the early morning hours.
She is intubated and ventilated adequately, and an intra-osseous needle is placed, but she remains pulseless. The first medications you
should employ are:

a. Epinephrine and norepinephrine

b. Norepinephrine and atropine

c. Lidocaine and atropine

d. Epinephrine and atropine

Solution. Ans 250: (d) Epinephrine and atropine


Ref Read the text below
Sol:
By this time in an arrest, an ECG should be available to identify the rhythm.
These are the drugs recommended for asystole, but pulseless electrical activity (electrical mechanical dissociation) should lead one to
search for a treatable cause of poor pulses in the presence of a cardiac rhythm capable of effective mechanical cardiac activity which
should be able to support the cardiac output.

Correct Answer. d

(251). Most common cause of contact dermatitis in ear rings is due to?

a. Copper

b. Nickel

c. Iron

d. Brass

Solution. Ans-251: (b) Nickel


Ref.: Read the text below
Sol :
There are two main types of contact dermatitis: irritant contact dermatitis, which accounts for approximately 80% of cases, and allergic
contact dermatitis, which accounts for approximately 20% of cases
Irritant contact dermatitis is a nonimmune-mediated reaction caused by direct injury to the skin following exposure to an irritant
chemical or physical agent (eg, alkalis in soaps and solvents), whereas allergic contact dermatitis is a type IV delayed hypersensitivity
reaction that only affects patients who have been previously sensitized to an allergen (eg, nickel, poison ivy oleoresin, bacitracin,
neomycin)

Correct Answer. b

Copyright 2014 Delhi Academy of Medical Sciences, All Rights Reserved. 100/121
(252). Wavelength of UV-8 ray is?

a. 280 nm

b. 400 nm

c. 250 nm

d. 220 nm

Solution. Ans-252: (a) 280 nm


Ref.: Read the text below
Sol :

UV wavelengths

UV A 250 280 nm

UV B 280 320 nm

UV C 320 400 nm

Correct Answer. a

(253). Salt & Pepper appearance of skin in is seen in?

a. SLE

b. Scleroderma

c. Morphea

d. Sjogrens syndrome

Solution. Ans-253: (b) Scleroderma


Ref.: Read the text below
Sol :
Salt and pepper change in skin is seen in diffuse systemic sclerosis

Correct Answer. b

(254). Circinate balanitis is caused by?

a. Spondyloarthritis

b. Reactive arthritis

c. Ankylosing spondylitis

d. SLE

Solution. Ans-254: (b) Reactive arthritis


Ref.: Read the text below
Sol :
Balanitis circinata (also known as circinate balanitis) is a dermatologic manifestation of reactive arthritis (formerly Reiter's Syndrome)
comprising a serpiginous annular dermatitis of the glans penis.
Reiter's syndrome is characterized by nongonococcal urethritis, conjunctivitis and arthritis

Correct Answer. b

Copyright 2014 Delhi Academy of Medical Sciences, All Rights Reserved. 101/121
(255). Painful vascular tumour at nail bed is?

a. Pyogenic granuloma

b. Kaposis sarcoma

c. Angiosarcoma

d. Glomus tumor

Solution. Ans-255: (d) Glomus tumor


Ref.: Read the text below
Sol :
Glomus tumours are the most characteristic of vascular nail bed toumours.
There is pain, which may be spontaneous or evoked by mild trauma or temperature change.
Nail plate changes depend on the location o the tumour.
Matrix tumours cause splitting and distortion of the nail plate.
Nail bed lesions are most likely to appear as bluish or red foci, 1-5 mm in diameter, beneath the nail.

Correct Answer. d

(256). Highly pruritic lesions all except :

a. Dermatitis herpetiformis

b. Lichen planus

c. Cutaneous vasculitis

d. Psoriasis

Solution. Ans-256: (c) Cutaneous vasculitis


Ref.: Read the text below
Sol :
Pruritis is a major feature and seen in excessive amounts in :
Psoriasis
Lichen planus
Dermatitis herpetiformis
Urticaria
Scabies
Eczema

Correct Answer. c

(257). Following is a premalignant condition?

a. Seborrhic keratosis

b. Actinic keratosis

c. Melanoacanthoma

d. Trichelememmal cyst

Solution. Ans-257: (b) Actinic keratosis


Ref.: Read the text below
Sol :
Actinic keratosis (also called "solar keratosis and "senile keratosis";abbreviated as "AK") is a pre-cancerous patch of thick, scaly, or
crusty skin.
These growths are more common in fair-skinned people and those who are frequently in the sun.
They usually form when skin gets damaged by ultraviolet (UV) radiation from the sun or indoor tanning beds. AKs are considered
potentially pre-cancerous; left untreated, they may turn into a type of cancer called squamous cell carcinoma

Correct Answer. b

Copyright 2014 Delhi Academy of Medical Sciences, All Rights Reserved. 102/121
(258). Berkeley membrane is seen in?

a. Psoriasis

b. Pemphigu

c. Tinea capitis

d. Pityriasis rubra

Solution. Ans-258: (a) Psoriasis


Ref.: Read the text below
Sol :
Psoriasis is characterised by well circumscribed, sharply demarcated erythematous papules and/or plaques.
These are covered by dry, brittle, silvery or grayish white, loosely adherent micaceous scales.
The scales are diposed in lamellar fashion
Occasionally, a white blanching ring is seen around psoriatic lesions, called Woronoffs ring.
On grattage, silvery white cells come off in layers. After their removal, a characteristic coherence is observed, as if one scratches on a
wax candle (candle grease sign).
On further grattage, a thin peel like membrane, Berkleys membrane, is seen, which comes off as a whole.

Correct Answer. a

(259). Bulla spread signs is seen in?

a. Herpes gestationalis

b. Bullous pemphigoid

c. Pemphigus vulgaris

d. Herpes simplex

Solution. Ans-259: (c) Pemphigus vulgaris


Ref.: Read the text below
Sol :
In the traditional bulla spread sign or Lutz sign, the margin of an intact bulla is first marked by a pen.
Slow, careful and unidirectional pressure applied by a finger to the bulla causes peripheral extension of the bulla beyond the marked
margin.
The bulla thus extended has an irregular angulated border in pemphigus vulgaris, while a regular rounded border is observed in bullous
pemphigoid or other subepidermal blistering disorders.
The sign may also be elicited on a burst blister if a substantial portion of the roof is intact.
The Asboe-Hansen sign is a variation of the bulla spread sign. It applies to smaller, intact, tense bullae where the pressure is applied to
the centre of the blister.
This sign is positive in all varieties or pemphigus and many cases of subepidermal blisters, including bullous pemphigoid, dermatitis
herpetiformis, epidermolysis bullosa acquisita, cicatricial pemphigoid, dystrophic epidermolysis bullosa, stevens Johnson syndrome and
toxic epidermal necrolysis.
Due to fragility of the roof of the blister it is usually negative in Hailey Hailey disease and staphylococcal scalded skin syndrome.

Correct Answer. c

(260). Lesions of pityriasis rosea are distributed mostly on the?

a. Face

b. Trunk

c. Scalp

d. Palms and soles

Solution. Ans-260: (b) Trunk


Ref.: Read the text below
Sol :
Lesions of Pityriasis rosea are usually confined to the upper trunk

Correct Answer. b

Copyright 2014 Delhi Academy of Medical Sciences, All Rights Reserved. 103/121
(261). Complication of total spinal anesthesia are all except :

a. Hypotension

b. Tachycardia

c. Respiratory depression

d. Unconsciousness

Solution. Ans-261: (b) Tachycardia


Ref.: Read the text below
Sol :
Total spinal anesthesia is seen when there is inadvertent intrathecal or subarachnoid injection while attempting an epidural/caudal
anesthesia.
Features of total spinal anesthesia :
Marked hypotension
Apnea
Dilated pupils
Respiratory depression
Bradycardia
Unconsciousness

Correct Answer. b

(262). Colour of ethylene cylinder is?

a. Blue

b. Grey

c. Orange

d. Purple

Solution. Ans-262: (d) Purple


Ref.: Read the text below
Sol :
Colour of ethylene cylinders is violet or purple.

Correct Answer. d

(263). Boyle apparatus was first developed in?

a. 1912

b. 1887

c. 1917

d. 1939

Solution. Ans-263: (c) 1917


Ref.: Read the text below
Sol :
The original concept of Boyle's machine was invented by the British anaesthetist Henry Boyle (18751941) in 1917.
Prior to this time, anaesthetists often carried all their equipment with them, but the development of heavy, bulky cylinder storage and
increasingly elaborate airway equipment meant that this was no longer practical for most circumstances.

Correct Answer. c

Copyright 2014 Delhi Academy of Medical Sciences, All Rights Reserved. 104/121
(264). Intracranial pressure is increased by which of the following inducing agent?

a. Thiopentone

b. Ketamine

c. Etomidate

d. Propofol

Solution. Ans-264: (b) Ketamine


Ref.: Read the text below
Sol :
Ketamine
Side effects
Cardiovascular: Arrythmias, bradycardia or tachycardia, hyper- or hypotension
Central nervous system: Ketamine is traditionally avoided in people with or at risk of intracranial hypertension (ICP) due to concerns
about ketamine causing increased intracranial pressure. It does not increase ICP more than opioids
Dermatologic: Transient erythema, transient morbilliform rash
Gastrointestinal: Anorexia, nausea, increased salivation, vomiting
Local: Pain or exanthema of the injection site
Neuromuscular and skeletal: Increased skeletal muscle tone (tonic-clonic movements)
Ocular: Diplopia, increased intraocular pressure, nystagmus
Respiratory: Airway obstruction, apnea, increased bronchial secretions, respiratory depression, laryngospasm
Other: Anaphylaxis, dependence, emergence reaction

Correct Answer. b

(265). Which of the following non-depolarising muscle relaxant is excreted maximally through kidney?

a. Gallamine

b. Pancuronium

c. Vecuronium

d. Rocuronium

Solution. Ans-265: (a) Gallamine


Ref.: Read the text below
Sol :
Gallamine
Less potent than curare ( 1/5 ).
Metabolized mainly by kidney 100%
Long duration of action.
Tachycardia due to :
Atropine like action.
Release of NA from adrenergic nerve endings

Correct Answer. a

(266). Respiratory irritation is seen with?

a. Ether

b. Halothane

c. Trichloroethlyene

d. Cyclopropane

Solution. Ans-266: (a) Ether


Ref.: Read the text below
Sol :
Ether causes respiratory irritation leading to laryngospasm and increased secretions, so atropine pre-medication is necessary.

Correct Answer. a

Copyright 2014 Delhi Academy of Medical Sciences, All Rights Reserved. 105/121
(267). In anaesthetic practice, preoperative digitalization is considered in order to prevent?

a. Intracardiac failure

b. Supraventricular arrhythmias

c. Post operative cardiac failure

d. All of the above

Solution. Ans-267: (d) All of the above


Ref.: Read the text below
Sol :
Although the role of pre operative digitalis is controversial due to narrow therapeutic index. It is used to prevent
Atrial fibrillation
Post operative CHF
Supraventricular ventricular tachycardia

Correct Answer. d

(268). The dosage of morphine used for preanaesthetic purposes in an adult patient is?

a. 5 mg IM

b. 10 mg IM

c. 20 mg IM

d. 30 mg IV

Solution. Ans-268: (b) 10 mg IM


Ref.: Read the text below
Sol :
Dose of morphine for pre anesthetic medication is 8-10 mg IM
Dose of meperidine for same indication is 50-100 mg IM

Correct Answer. b

(269). To counter central anticholinergic effect, drug used is?

a. Physostigmine

b. Atropine

c. Neostigmine

d. Hyoscine

Solution. Ans-269: (a) Physostigmine


Ref.: Read the text below
Sol :
Because physostigmine is a tertiary amine (and thus does not have hydrogen bond, making is more hydrophobic), it can cross the blood-
brain barrier, and physostigmine salicylate is used to treat the central nervous system effects of atropine, scopolamine and other
anticholinergic drug overdoses.

Correct Answer. a

Copyright 2014 Delhi Academy of Medical Sciences, All Rights Reserved. 106/121
(270). An adult trachea has a diameter of?

a. 0.5 to 1 cm

b. 1.2 to 1.6 cm

c. 2.5 to 3 cm

d. 3 to 3.5 cm

Solution. Ans-270: (b) 1.2 to 1.6 cm


Ref.: Read the text below
Sol :

Dimensions of Trachea

Tracheal Parameter Men Women

External diameter sagitally 1.8 cm 1.4 cm

External Diameter coronally 2.3 cm 2.0 cm

Thickness of wall of trachea 3 mm 3 mm

Correct Answer. b

(271). Frog eye appearance in ultrasound is seen in?

a. Anencephaly

b. Acardia

c. Downs syndrome

d. Pataus syndrome

Solution. Ans-271: (a) Anencephaly


Ref.: Read the text below
Sol :
The prominent, bulging eyes give the anencephalic fetus a typical ultrasound appearance designated frogs eyes.
A frog eye or mickey mouse appearance may be seen when seen in the coronal plane due to absent cranial bone/brain and bulging
orbits.

Correct Answer. a

(272). Which is not used in Radiotherapy?

a. Linear accelerator

b. Cobalt unit

c. Technetium 99

d. Caesium unit

Solution. Ans-272: (c) Technetium 99


Ref.: Read the text below
Sol :
Technetium 99 is used in radionucleotide scanning not radiotherapy external beam therapy (EBT), also called external radiation therapy,
is a method for delivering a beam or several beams of high energy x-rays to a patients tumor.
Beams are generated outside the patient, usually by a linear accelerator and are targeted at the tumor site.

Correct Answer. c

Copyright 2014 Delhi Academy of Medical Sciences, All Rights Reserved. 107/121
(273). Half life of Radio-Iodine is?

a. 72 hours

b. 8 days

c. 8 weeks

d. 6 months

Solution. Ans-273: (b) 8 days


Ref.: Read the text below
Sol :
Iodine 131 is a radioisotope with a very short half-life of 8.02 days, making it highly radioactive. Frequently used in small doses in thyroid
cancers therapies, it is also one of the most feared fission products when accidentally released into the environment.

Correct Answer. b

(274). In intestinal pseudo obstruction, what is done?

a. Double contrast X-ray taken

b. Colonoscopic decompression

c. Surgery

d. None of these

Solution. Ans-274: (b) Colonoscopic decompression


Ref.: Read the text below
Sol :
Use of colonoscopy, decompression tube placement in the ascending colon, and cecostomy should be reserved for patients who do not
respond to neostigmine administration.
Endoscopic decompression has lower mortaliy as compared to surgery.
In the presence of peritoneal signs or perforation, surgery is the appropriate first intervention.

Correct Answer. b

(275). Pleural effusion without evident lung infiltration is most often due to?

a. Viral pleurisy

b. Lymphoma

c. Carcinoma

d. Tuberculosis

Solution. Ans-275: (a) Viral pleurisy


Ref.: Read the text below
Sol :
Numerous viral infections have been associated with pleural effusion.
Effusions are usually small in size & accompany parenchymal infiltrates.
Of note, viral pleurisy without radiographic infiltrates has been shown to produce pleural effusions.
Isolated pleural effusion can occur as a sole manifestation of AIDS related lymphoma without pulmonary parenchymal abnormality and
lymphadenopathy.

Correct Answer. a

Copyright 2014 Delhi Academy of Medical Sciences, All Rights Reserved. 108/121
(276). Investigation of choice for remnant spleen is?

a. Tc99 pertechnetium scan

b. Tc99 labelled heat labeled RBC scan

c. Gallamium

d. CECT

Solution. Ans-276: (b) Tc99 labelled heat labeled RBC scan


Ref.: Read the text below
Sol :
Tc99m-tagged heat-damaged RBC scan (Tc-99m-DRBC) with autologous erythrocytes remains gold-standard of imaging, being capable of
specifically proving splenic tissue

Correct Answer. b

(277). The gold standard of investigation is a case of acute cholecystitis is?

a. Ultrasound

b. ERCP

c. OCG

d. Radionuclide imaging

Solution. Ans-277: (d) Radionuclide imaging


Ref.: Read the text below
Sol :
The hepatobiliary imindiacetic acid (HIDA) scan remains the gold standard test for acute cholecystitis.
But initial investigation of choice is USG.
Ultrasound is the most useful radiographic test for diagnosing acute cholecystitis, with sensitivity and specificity of 85% and 95%
respectively.
Biliary radionuclide scanning (HIDA scan) may be of help in the atypical case.
A normal HIDA scan excludes acute cholecystitis.
However, when the patient is fasting for more than 5 days, HIDA scan is much less helpful, with a 40% false positive rate

Correct Answer. d

(278). On radiography widened duodenal C loop with irregular mucosal pattern on upper gastrointestinal barium series is most likely due to?

a. Chronic pancreatitis

b. Carcinoma head of pancreas

c. Duodenal ulcer

d. Duodenal ileus

Solution. Ans-278: (b) Carcinoma head of pancreas


Ref.: Read the text below
Sol :
Widening of duodenal C loop due to Ca head of pancreas is called Antral Pad sign
The antral pad sign is seen on a radiograph of the upper gastrointestinal tract obtained with orally administered contrast material.
The sign refers to an extrinsic impression or indentation on the posteroinferior aspect of the antrum.

Correct Answer. b

Copyright 2014 Delhi Academy of Medical Sciences, All Rights Reserved. 109/121
(279). X-ray of closed pneumothorax is done at?

a. Full expiration

b. Full inspiration

c. Mid inspiration

d. Between inspiration and expiration

Solution. Ans-279: (a) Full expiration


Ref.: Read the text below
Sol :
Although chest radiographs are routinely taken at full inspiration, an expiratory film may be helpful under appropriate circumstances.
For example, a small pneumothorax is often difficult or impossible to be seen on a routine inspiratory PA film.
On expiration, the volume of thorax & lungs within it is reduced, but the amount of air in the pleural sac remains essentially unchanged.
Also, when a film is taken in expiration, the lungs appear denser because of the blood containing vessels are crowded into a smaller
space.
Because the blackness of pneumothorax does not change, the density gradient between the pneumothorax & the lungs becomes larger,
making it easier to visualize the pneumothorax.

Correct Answer. a

Copyright 2014 Delhi Academy of Medical Sciences, All Rights Reserved. 110/121
(280). Paranoid psychosis observed with cocaine abuse can be explained by

a. Tolerance

b. Intoxication

c. Reverse tolerance

d. Withdrawal

Solution. Ans-280: (b) Intoxication


Ref: Harrison 18th ed.chapter 394,Kaplan & saddocks Synopsis of psychiatry 10th ed. page425,426
Sol :
Cocaine is a stimulant and a local anesthetic with potent vasoconstrictor properties. The reinforcing effects of cocaine appear to be
related to activation of dopaminergic neurons in the mesolimbic system. Cocaine increases synaptic concentrations of the monamine
neurotransmitters dopamine, norepinephrine, and serotonin by binding to transporter proteins in presynaptic neurons and blocking
reuptake.
Forms of the drug
In powder form, cocaine is known by such street names as "coke," "blow," "C," "flake," "snow" and "toot." It is most commonly inhaled or
"snorted." It may also be dissolved in water and injected.
Crack is a smokable form of cocaine that produces an immediate and more intense high. It comes in off-white chunks or chips called
"rocks." Little crumbs of crack are sometimes called "kibbles & bits."
In addition to their stand-alone use, both cocaine and crack are often mixed with other substances. Cocaine may be mixed with
methcathinone (a more recent drug of abuse, known as "cat," that is similar to methamphetamine) to create a "wildcat." A hollowed-out
cigar filled with a mixture of crack and marijuana is known as a "woolah." And either cocaine or crack used in conjunction with heroin is
called a "speed-ball." Cocaine used together with alcohol represents the most common fatal two-drug combination.
Cocaine produces a brief, dose-related stimulation and enhancement of mood and an increase in cardiac rate and blood pressure. Body
temperature usually increases following cocaine administration, and high doses of cocaine may induce lethal pyrexia or hypertension.
Because cocaine inhibits reuptake of catecholamines at adrenergic nerve endings, the drug potentiates sympathetic nervous system
activity. Cocaine has a short plasma half-life of approximately 4560 min. It is metabolized by plasma esterases, and cocaine metabolites
are excreted in urine. The very short duration of the euphorigenic effects of cocaine observed in chronic abusers is probably due to both
acute and chronic tolerance.
Cocaine abuse has four distinct phases-
Euphoria
Dysphoria
Halucination
Psychosis
1. The first phase is cocaine euphoria, characterized by intense pleasure which is accompanied by affective lability, hypervigilance,
hyperactivity and hypersexuality.
2. The second phase, which sometimes follows after a few hours of smoking, is a dysphoric state manifested by
considerable anxiety and smoking compulsion. Other affective changes can be observed in some persons as sadness, melancholy, apathy,
or aggressiveness..
3. The third phase is the cocaine hallucinosis, which can also develop after smoking several grams of coca paste and is marked by visual,
tactile, auditory, and olfactory hallucinations. The patient is generally very excited and has transitory delusional interpretations. The
hallucinosis is evanescent, it may last for two or three days and then fade gradually, if the individual has discontinued to smoke or
receives parenteral neuroleptics.
4. The fourth phase is the cocaine psychosis. It may appear after days or weeks of frequent or continued paste smoking. It is
characterized by a marked agitation with hypervigilance and defined paranoid delusions of persecution, damage, death or spouse
unfaithfulness. Generally it is accompanied by auditory and olfactory hallucinations.
Treatment: Cocaine Overdose
Treatment of cocaine overdose is a medical emergency that is best managed in an intensive care unit. Cocaine toxicity produces a
hyperadrenergic state characterized by hypertension, tachycardia, tonic-clonic seizures, dyspnea, and ventricular arrhythmias.
Intravenous diazepam in doses up to 0.5 mg/kg administered over an 8-h period has been shown to be effective for control of seizures.
Ventricular arrhythmias have been managed successfully by administration of 0.51 mg of propranolol IV. Since many instances of
cocaine-related mortality have been associated with concurrent use of other illicit drugs (particularly heroin).
Cocaine withdrawl
If a person who is dependent on cocaine suddenly stops taking it, or significantly cuts down the amount they are using, they can
experience withdrawal symptoms.
Cocaine withdrawal generally occurs in three phases: the crash; the withdrawal; and the extinction.
The crash, which usually occurs in the first few days, describes the withdrawal symptoms experienced immediately after the person stops
using cocaine.
The withdrawal phase may last up to 10 weeks. During this phase, people experience severe cravings for cocaine.
The extinction phase may last indefinitely. It involves intermittent cravings for cocaine. These generally occur in response to people,
places or objects that are conditioned cues and provoke memories of taking the drug. These cravings may surface months or years after
cocaine use has stopped.
Reverse Tolerance-
As the tolerance to cocaine increases, the addict feels he is becoming less sensitive to the drugs effects. While the addict is becoming
less sensitive to the pleasurable effect of the drug, he is at the same time becoming more likely to develop epileptic seizures, delusions,
and psychotic behavior.

Correct Answer. b

Copyright 2014 Delhi Academy of Medical Sciences, All Rights Reserved. 111/121
(281). A 35-year female has been diagnosed with obsessive compulsive disorder (OCD) and she washes her hands 40 times a day. Which would
be the best CBT technique for her treatment?

a. Thought stopping

b. Response prevention

c. Relaxation

d. Exposure

Solution. Ans-281: (d) Exposure


Ref: Harrison 18th ed.chapter 391
Sol :
Obsessive-Compulsive Disorder
Obsessive-compulsive disorder (OCD) is characterized by obsessive thoughts and compulsive behaviors that impair everyday functioning.
Fears of contamination and germs are common, as are handwashing, counting behaviors, and having to check and recheck such actions
as whether a door is locked. The degree to which the disorder is disruptive for the individual varies, but in all cases obsessive-compulsive
activities take up >1 hour per day and are undertaken to relieve the anxiety triggered by the core fear. Patients often conceal their
symptoms, usually because they are embarrassed by the content of their thoughts or the nature of their actions.
A genomewide association study (GWAS) reported linkage to chromosome 2p23.2; however, no susceptibility gene for OCD has been
identified to date. Family studies show an aggregation of OCD with Tourette's disorder, and both are more common in males and in first-
born children.
Treatment: Obsessive-Compulsive Disorder
Clomipramine, fluoxetine, fluvoxamine, and sertraline are approved for the treatment of OCD. Clomipramine is a TCA that is often
tolerated poorly owing to anticholinergic and sedative side effects at the doses required to treat the illness (25250 mg/d); its efficacy in
OCD is unrelated to its antidepressant activity. Fluoxetine (560 mg/d), fluvoxamine (25300 mg/d), and sertraline (50150 mg/d) are as
effective as clomipramine and have a more benign side effect profile. Only 5060% of patients with OCD show adequate improvement
with pharmacotherapy alone. In treatment-resistant cases, augmentation with other serotonergic agents such as buspirone, or with a
neuroleptic or benzodiazepine may be beneficial and in severe cases deep brain stimulation has been found to be effective. When a
therapeutic response is achieved, long-duration maintenance therapy is usually indicated.
For many individuals, particularly those with time-consuming compulsions, behavior therapy will result in as much improvement as that
afforded by medication. Effective techniques include the gradual increase in exposure to stressful situations, maintenance of a diary to
clarify stressors, and homework assignments that substitute new activities for compulsive behaviors.

Correct Answer. d

(282). All the following are currently used as psychologic tests of personality functioning except the

a. Rorschach Test

b. Thematic Apperception Test

c. Word Association Test

d. Bender Gestalt Test

Solution. Ans 282: (d) Bender Gestalt Test


Ref:Read the text below
Sol:
When properly administered and interpreted by an experienced clinician, psychologic tests offer a significant contribution to
understanding psychiatric illness.
These tests can be powerful tools in clarifying issues of psychiatric diagnosis, psychodynamics, and patient management.
Standardized psychologic tests provide a fairly objective means for comparing behavior with available normative data representative of a
larger reference group.
Psychologic tests of personality functioning include the Thematic ApperceptionTest, Word Association Test, Sentence Completion Test,
and Rorschach Test. The Bender Gestalt Test is a test of visuomotor coordination.

Correct Answer. d

Copyright 2014 Delhi Academy of Medical Sciences, All Rights Reserved. 112/121
(283). Anorexia nervosa can be described by all the following statements except

a. Prognosis is more favorable in younger adolescents than in older adolescents

b. Boys develop anorexia nervosa much less often than do girls

c. Affected persons typically believe they remain overweight despite marked weight loss

d. Affected persons typically are rebellious and delinquent

Solution. Ans 283: (d) Affected persons typically are rebellious and delinquent
Ref:Read the text below
Sol:
Anorexia nervosa, a disorder predominantly but not exclusively affecting females, is characterized by obsessional weight loss and
disordered body image.
Other behavioral features include food binges, selfinduced vomiting, and abuse of laxatives.
Affected persons typically are described as pleasant, polite overachievers. Mortality rate is as high as 20%, with death caused by
dehydration, electrolyte imbalances, and other manifestations of starvation.
The prognosis is better if affected persons are younger and if intervention is begun before weight loss has become marked

Correct Answer. d

(284). The MNaghten rule is associated in American jurisprudence with

a. Protocols for civil commitment

b. The insanity defense

c. The right to psychiatric treatment

d. Maintaining confidentiality

Solution. Ans 284: (b) The insanity defense


Ref:Read the text below
Sol:
Daniel MNaghten, a disturbed Scottish woodcutter, attempted to kill the British Prime Minister.
He mistook the Prime Ministers secretary for the Prime Minister and shot and killed him.
In the subsequent trial,MNaghtens counsel argued that his client suffered from a mental illness and therefore did not know the
difference between right and wrong and could not comprehend the nature or consequences of his actions.
Although modified over time, these criteria for an insanity defense have become widely adopted in the United States.

Correct Answer. b

(285). The unconscious feelings that arise within a psychotherapist during psychotherapy are described by the term

a. Projection

b. Countertransference

c. Acting out

d. Identification

Solution. Ans 285: (b) Countertransference


Ref:Read the text below
Sol:
During the course of psychotherapy, patients develop feelings toward the therapist that recreate feelings they have had toward
significant persons in their life, a phenomenon called transference.
Identification of these feelings and the behaviors they provoke is crucial, especially in insight-oriented psychotherapy.
However, during the course of conducting psychotherapy, therapists, too, develop feelings and reactions as memories of situations from
their past are evoked.
These countertransference feelings provide the therapist with valuable clues regarding how people in the patients life may have reacted
to the patient. Countertransference may also serve to pinpoint areas within the therapists own unconscious that may prejudice the
ability to offer nondirective, unbiased treatment.

Correct Answer. b

Copyright 2014 Delhi Academy of Medical Sciences, All Rights Reserved. 113/121
(286). Poor scholastic performance is not associated with

a. SLD

b. ADHD

c. PICA

d. Anxiety

Solution. Ans-286: (c) PICA


Ref: Kaplan & saddocks Synopsis of psychiatry 10th ed. Page 1228-1230
Sol :
In PICA presenting complaints is either nutritional deficiencies, lead complications, or persistantly taking something in mouth.
ADHD,Anxiety & specific learning disorder are associated with decreased scholastic performance.
Pica is characterized by an appetite for substances largely non-nutritive (such as clay or chalk). For these actions to be considered pica,
they must persist for more than one month at an age where eating such objects is considered developmentally inappropriate. There are
different variations of pica, as it can be from a cultural tradition, acquired taste or a neurological mechanism such as an iron deficiency,
or chemical imbalance.
Diagnosis
There is no single test that confirms pica. However, because pica can occur in people who have lower than normal nutrient levels and
poor nutrition (malnutrition), the health care provider should test blood levels of iron and zinc. Hemoglobin can also be checked to test
for anemia. Lead levels should always be checked in children who may have eaten paint or objects covered in lead-paint dust. The health
care provider should test for infection if the person has been eating contaminated soil or animal waste.
DSM IV criteria for PICA
A. Persistent eating of nonnutritive substances for a period of at least 1 month
B. Does not meet the criteria for either autistic disorder, schizophrenia, or Kleine-Levin syndrome.
C. The eating behavior is not part of a culturally sanctioned practice
D. If the eating behavior occurs exclusively during the course of another mental disorder (e.g., mental retardation, pervasive
developmental disorder, schizophrenia), it is sufficiently severe to warrant independent clinical attention.

Correct Answer. c

(287). Delusions are not likely to be a part of the clinical picture of

a. Dementia

b. Schizophrenia

c. Depression

d. Conversion disorder

Solution. Ans-287: (d) Conversion disorder


Ref: Kaplan & saddocks Synopsis of psychiatry 10th ed. Page 638-639
Sol :
The definition of a delusion is a false,unshakeable idea or belief held with extraordinary conviction and subjective certainty.Delusion are
common symptom of arrange of psychiatric illnesss including Schizophrenia,depression,bipolar disorder & dementia.
DSM IV criteria for Conversion disorder-
A. One or more symptoms or deficits affecting voluntary motor or sensory function that suggest a neurological or other general medical
condition.
B. Psychological factors are judged to be associated with the symptom or deficit because the initiation or exacerbation of the symptom or
deficit is preceded by conflicts or other stressors.
C. The symptom or deficit is not intentionally produced or feigned (as infactitious Disorder or Malingering).
D. The symptom or deficit cannot, after appropriate investigation, be fully explained by a general medical condition, or by the direct
effects of substance, or as a culturally sanctioned behavior or experience.
E. The symptom or deficit causes clinically significant distress or impairment in social, occupational, or other important areas of
functioning or warrants medical evaluation.
F. The symptom or deficit is not limited to pain or sexual dysfunction, does not occur exclusively during the course of Somatization
Disorder, and is not better accounted for by another mental disorder.
Specify type of symptom or deficit:
With Motor Symptom or Deficit
With Sensory Symptom or Deficit
With Seizures or Convulsions
With Mixed Presentation

Correct Answer. d

Copyright 2014 Delhi Academy of Medical Sciences, All Rights Reserved. 114/121
(288). Repetitive transcranial magnetic brain stimulation (rTMS) is approved by US FDA for the treatment of

a. Depressive disorder

b. Obsessive compulsive disorder

c. Acute psychosis

d. Resistant schizophrenia

Solution. Ans-288 : (a) Depressive disorder


Ref: Harrison 18th ed.chapter 391
Sol :
Treatment of Depression-
The most effective intervention for achieving remission and preventing relapse is medication, but combined treatment, incorporating
psychotherapy to help the patient cope with decreased self-esteem and demoralization, improves outcome.
Electroconvulsive therapy is at least as effective as medication, but its use is reserved for treatment-resistant cases and delusional
depressions. Transcranial magnetic stimulation (TMS) is approved for treatment-resistant depression and has been shown to have
efficacy in several controlled trials. Vagus nerve stimulation (VNS) has also recently been approved for treatment-resistant depression,
but its degree of efficacy is controversial. Deep brain stimulation is another treatment that is being used experimentally in treatment
resistant cases.
Transcranial magnetic stimulation (TMS) is a noninvasive method to cause depolarization or hyperpolarization in the neurons of the
brain. TMS uses electromagnetic induction to induce weak electric currents using a rapidly changing magnetic field; this can cause
activity in specific or general parts of the brain with minimal discomfort, allowing the functioning and interconnections of the brain to be
studied. A variant of TMS, repetitive transcranial magnetic stimulation (rTMS), has been tested as a treatment tool for various
neurological and psychiatric disorders including migraines, strokes, Parkinson's disease, dystonia, tinnitus, depression and auditory
hallucinations

Correct Answer. a

(289). A 30-year lady presents with pain and tenderness in index finger just under the nail. She was unable to wash her hands with cold
water.Patient doesnot reveal any history of trauma or injury . What could be probable findings-

a. Sausage digits

b. Ridging of nail,discolouration & pin-head tenderness

c. Stiffness of whole hand

d. Hypersensitivity of finger

Solution. Ans-289: (b) Ridging of nail,discolouration & pin-head tenderness


Ref: Apleys system of orthopaedics & fracture,9th ed.page 221-222
Sol :
Diagnosis in above case is Glomus tumor
Glomus Tumour
This rare tumour usually occurs around fine peripheral neurovascular structures and specially in nail bed of fingers & toes.A young adult
presents with recurrent episodes of intense pain in fingertip.A small bluish nodule may be seen under the nail,the area is sensitive to cold
and exquisitely tender.X-rays sometime show erosion of underlying phalanx.Treatment is excision,the tumor never larger than a pea,is
easily shelled out of its fibrous capsule.

Correct Answer. b

Copyright 2014 Delhi Academy of Medical Sciences, All Rights Reserved. 115/121
(290). Which of the following is not included as a yellow flag sign for low back pain?

a. Radicular impingement

b. Systemic steroids

c. Social isolation

d. High functional limitation at 4 weeks/after 4weeks

Solution. Ans-290 : (a) Radicular impingement


Ref: BMJ 2003;326:535
Sol :
Red flag and yellow flag signs
Red flags are possible indicators of serious spinal pathology:
Thoracic pain
Fever and unexplained weight loss
Bladder or bowel dysfunction
History of carcinoma
Ill health or presence of other medical illness
Progressive neurological deficit
Disturbed gait, saddle anaesthesia
Age of onset <20 years or >55 years
Yellow flags are pyschosocial factors shown to be indicative of long term chronicity and disability:
A negative attitude that back pain is harmful or potentially severely disabling
Fear avoidance behaviour and reduced activity levels
An expectation that passive, rather than active, treatment will be beneficial
A tendency to depression, low morale, and social withdrawal
Social or financial problems

Correct Answer. a

(291). Non union is a common complication in all of the following fractures except

a. Fracture Neck of the femur

b. Fracture Scaphoid

c. Fracture lower third of Tibia

d. Trochanteric Fractures

Solution. Ans-291 : (d) Trochanteric Fractures


Ref.: Essential Orthopedics, RM Shenoy- 7, 92
Sol :
Trochanteric fractures heal faster and unite well and have less complications due to the rich blood supply in this area.

Correct Answer. d

Copyright 2014 Delhi Academy of Medical Sciences, All Rights Reserved. 116/121
(292). Distended bladder, incontinence of urine and priapism are

a. Signs of injury to the Urethra

b. Signs of injury to Cauda Equina

c. Signs of injury to pelvis

d. Signs of spinal cord injury

Solution. Ans-292 : (d) Signs of spinal cord injury


Ref.: Esential Orthopedics, RM Shenoy- 119
Sol :
In addition to a loss of sensation and motor function below the point of injury, individuals with spinal cord injuries will often experience
other complications of spinal cord injury :
Dysfunction of the bowel/bladder, including infections of the bladder, and anal incontinence.
Sexual function is also affected; priapism in males.
Injuries of the C-1, C-2 will often result in a loss of breathing, necessitating mechanical ventilators or phrenic nerve pacing.
Inability or reduced ability to regulate heart rate, blood pressure, sweating and hence body temperature.
Spasticity (increased reflexes and stiffness of the limbs).

Correct Answer. d

(293). Pain due to post-amputation neuroma is best treated by

a. Infrared therapy

b. Interference therapy

c. Ultrasound therapy

d. Stump bandaging

Solution. Ans-293 : (c) Ultrasound therapy


Ref: De Lisas Physical Medicine and Rehabilitation: Principles and Practice; 5/e, Vol II, pg 1304
Sol :
TREATMENT OF POST-AMPUTATION NEUROMA AND SCAR PAIN
Repeated injection of a local anaesthetic has proven to be an extremely useful technique.
This should be followed by appropriate physical therapy to the scar, usually ultrasound followed by stretching or deep massage of the
scar.
This type of treatment has provided permanent or prolonged pain relief for longer than 6 months in many patients.
When the local anaesthetics, with or without steroids, do not provide prolonged relief, other methods should be considered.
Cryoanalgesia using a cryoprobe and freezing the neuroma for 1 minute at 20OC has been used with good success.
The advantage of a cryoprobe lies in the fact that it is a physical method of blocking the nerve without further neuroma or neuritis.
Neurolytic agents such as phenol or alcohol have been used to relieve neuroma and scar pain.
However, incomplete block with these agents can result in neuritis, producing severe pain.
These neurolytic techniques should be used only after repeated injections of local anaesthetics produce consistent pain relief
proportional to the duration of action of the local anaesthetic agent.
Although surgical revision of the scar is often considered, it is not very successful when the scar cannot be stretched out and there is
significant nerve entrapment.

Correct Answer. c

(294). The ideal treatment of a 3 day old fracture neck of femur in a 50-year-old male would be

a. Compression screw fixation

b. POP hip spica

c. Hemireplacement arthroplasty

d. Total hip replacement

Solution. Ans-294 : (a) Compression screw fixation


Ref.: Essential Orthopedics, Maheswari- 115.
Sol :
The ideal treatment of a 3 day old fracture neck of femur in a 50-year-old male would be compression screw fixation (multiple AO
cancellous screws, SP Nail or DHS rare now).

Correct Answer. a

Copyright 2014 Delhi Academy of Medical Sciences, All Rights Reserved. 117/121
(295). Abduction and external rotation deformity at the hip may be seen in all of the following conditions except

a. Tuberculosis

b. Posterior dislocation

c. Poliomyelitis

d. Fracture neck femur

Solution. Ans-295 : (b) Posterior dislocation


Ref.: Essential Orthopedics, Maheswari- 111
Sol :
Posterior dislocation of the hip causes a characteristic deformity Flexion, Adduction, Internal Rotation.

Correct Answer. b

(296). A compartment syndrome in a leg can result from all of the following except

a. Edema of muscles due to injury

b. Fracture of hematoma within the compartment

c. Edema of muscles due to ischemia

d. Compound fracture

Solution. Ans-296 : (d) Compound fracture


Ref.: Essential Orthopedics, Maheswari, - 38, 1
Sol :
The limbs contain muscles in compartments enclosed by bones, fascia and interosseous membrance.
A rise in pressure within these compartments due to any reason may jeopardize the blood supply to the muscles and nerves within the compartment compartment
syndrome.

Causes of compartment syndrome :


Any injury leading to edema of muscles
Fracture hematoma within the compartment
Ischemia to the compartment, leading to muscle edema

Correct Answer. d

(297). Froments sign is diagnostic of the following nerve injury

a. Median

b. Ulnar

c. Radial

d. Musculocutaneous

Solution. Ans-297 : (b) Ulnar


Ref.: Essential Orthopedics- 10.7.
Sol :
Also called as the book test, Froments sign is positive in ulnar nerve injury and indicates paralysis of the adductor pollicis.

Correct Answer. b

Copyright 2014 Delhi Academy of Medical Sciences, All Rights Reserved. 118/121
(298). A 40 year old male presents with weakness & pain in his lower limbs.Patient revealed history of paralysis of both lower limbs during
childhood.Probable Diagnosis would be-

a. Post polio syndrome

b. Polymyositis

c. Muscular dystrophy

d. Neuropathy

Solution. Ans-298 : (a) Post polio syndrome


Ref: http://www.ninds.nih.gov/disorders/post_polio/detail_post_polio.htm
Sol :
Post-polio syndrome (PPS) is a condition that affects polio survivors years after recovery from an initial acute attack of the poliomyelitis
virus. Most often, polio survivors start to experience gradual new weakening in muscles that were previously affected by the polio
infection.
The most common symptoms include slowly progressive muscle weakness, fatigue (both generalized and muscular), and a gradual
decrease in the size of muscles (muscle atrophy).
Pain from joint degeneration and increasing skeletal deformities such as scoliosis (curvature of the spine) are common and may precede
the weakness and muscle atrophy. Some individuals experience only minor symptoms while others develop visible muscle weakness and
atrophy.

Correct Answer. a

(299). The normal AV nodal delay is

a. 0.16 sec

b. 0.13 sec

c. 0.10 sec

d. 0.03 sec

Solution. Ans-299: (c) 0.10 sec


Ref.: Ganongs Physiology, 23nd edn. Pg. 492
Sol :
Depolarization initiated in the SA node spread radially through the atria, then converges on the AV node.
Because conduction through the AV node is slow a delay of about 0.1 s (AV nodal delay) occurs before excitation spread to the ventricles.

Correct Answer. c

Copyright 2014 Delhi Academy of Medical Sciences, All Rights Reserved. 119/121
(300). Vasopressin secretion is increased by :

a. Increased effective osmotic pressure of plasma.

b. Decreased effective osmotic pressure of plasma.

c. Increased extracellular fluid volume

d. Alcohol

Solution. Ans-300: (a) increased effective osmotic pressure of plasma.


Ref.: Ganongs -. 667
Sol :

Correct Answer. a

Test Answer
1.(c) 2.(a) 3.(b) 4.(b) 5.(c) 6.(a) 7.(d) 8.(a) 9.(a) 10.(d)

11.(c) 12.(d) 13.(c) 14.(b) 15.(d) 16.(a) 17.(c) 18.(c) 19.(c) 20.(d)

21.(c) 22.(d) 23.(a) 24.(c) 25.(a) 26.(d) 27.(b) 28.(d) 29.(b) 30.(b)

31.(d) 32.(c) 33.(a) 34.(b) 35.(a) 36.(b) 37.(a) 38.(a) 39.(c) 40.(a)

41.(d) 42.(c) 43.(d) 44.(c) 45.(b) 46.(a) 47.(a) 48.(d) 49.(d) 50.(b)

51.(d) 52.(d) 53.(c) 54.(c) 55.(b) 56.(d) 57.(a) 58.(c) 59.(a) 60.(b)

61.(d) 62.(c) 63.(b) 64.(c) 65.(d) 66.(b) 67.(a) 68.(d) 69.(b) 70.(b)

71.(b) 72.(d) 73.(c) 74.(c) 75.(d) 76.(d) 77.(b) 78.(a) 79.(d) 80.(b)

81.(b) 82.(d) 83.(b) 84.(c) 85.(d) 86.(a) 87.(d) 88.(b) 89.(c) 90.(b)

91.(d) 92.(d) 93.(a) 94.(a) 95.(d) 96.(a) 97.(c) 98.(c) 99.(c) 100.(c)

101.(a) 102.(c) 103.(c) 104.(b) 105.(c) 106.(d) 107.(a) 108.(b) 109.(b) 110.(d)

111.(c) 112.(a) 113.(d) 114.(c) 115.(c) 116.(b) 117.(a) 118.(c) 119.(d) 120.(b)

121.(c) 122.(c) 123.(b) 124.(a) 125.(c) 126.(c) 127.(d) 128.(d) 129.(c) 130.(d)

131.(a) 132.(d) 133.(b) 134.(d) 135.(d) 136.(b) 137.(c) 138.(a) 139.(c) 140.(c)

141.(c) 142.(d) 143.(c) 144.(b) 145.(d) 146.(b) 147.(c) 148.(a) 149.(a) 150.(d)

151.(a) 152.(a) 153.(b) 154.(b) 155.(c) 156.(a) 157.(d) 158.(b) 159.(c) 160.(c)

161.(a) 162.(a) 163.(b) 164.(d) 165.(a) 166.(c) 167.(d) 168.(a) 169.(c) 170.(a)

171.(d) 172.(c) 173.(a) 174.(d) 175.(a) 176.(c) 177.(c) 178.(c) 179.(b) 180.(a)

181.(c) 182.(d) 183.(d) 184.(d) 185.(a) 186.(b) 187.(c) 188.(d) 189.(c) 190.(b)

191.(a) 192.(d) 193.(c) 194.(b) 195.(d) 196.(d) 197.(b) 198.(b) 199.(d) 200.(d)

201.(d) 202.(b) 203.(c) 204.(b) 205.(b) 206.(a) 207.(a) 208.(a) 209.(c) 210.(a)

211.(a) 212.(b) 213.(b) 214.(b) 215.(a) 216.(b) 217.(a) 218.(c) 219.(a) 220.(a)

221.(a) 222.(b) 223.(c) 224.(b) 225.(b) 226.(b) 227.(d) 228.(b) 229.(d) 230.(b)

Copyright 2014 Delhi Academy of Medical Sciences, All Rights Reserved. 120/121
231.(c) 232.(c) 233.(c) 234.(b) 235.(a) 236.(a) 237.(a) 238.(c) 239.(a) 240.(d)

241.(a) 242.(a) 243.(c) 244.(c) 245.(c) 246.(c) 247.(b) 248.(b) 249.(c) 250.(d)

251.(b) 252.(a) 253.(b) 254.(b) 255.(d) 256.(c) 257.(b) 258.(a) 259.(c) 260.(b)

261.(b) 262.(d) 263.(c) 264.(b) 265.(a) 266.(a) 267.(d) 268.(b) 269.(a) 270.(b)

271.(a) 272.(c) 273.(b) 274.(b) 275.(a) 276.(b) 277.(d) 278.(b) 279.(a) 280.(b)

281.(d) 282.(d) 283.(d) 284.(b) 285.(b) 286.(c) 287.(d) 288.(a) 289.(b) 290.(a)

291.(d) 292.(d) 293.(c) 294.(a) 295.(b) 296.(d) 297.(b) 298.(a) 299.(c) 300.(a)

Copyright 2014 Delhi Academy of Medical Sciences, All Rights Reserved. 121/121

Vous aimerez peut-être aussi